You are on page 1of 54

Examsguru.

Net – LASU POST UTME PAST QUESTIONS WITH SOLUTIONS 2005 - 2014

LAGOS STATE UNIVERSITY


POST UTME PAST QUESTIONS AND ANSWERS
All Departments 2005-2013
============================================================================

ENGLISH LANGUAGE 9. From the list of words lettered A to D choose the one
2013/2014 Past Questions with Solutions that is most nearly opposite in meaning to the
1. From the list of words lettered A to D choose the one underlined word and that will, at the same time,
that is most nearly opposite in meaning to the correctly fill the gap in the sentenced: Last night,
underlined word and that will, at the same time, Uncle Joe was blind drunk but this morning he is as
correctly fill the gap in the sentence: the ....................adjudge (a) alert (b) saintly (c) strong (d)
management has neither.... nor denied the rumour sober
that is was going to lay of some workers (a) 10. From the list of words lettered A to D choose the one
pronounced (b) confirmed (c) rejected (d) advertised that is mostly nearly opposite in meaning to the
2. From the list of words lettered A to D choose the one underlined word and that will, at the same time,
that is most nearly opposite in meaning to the correctly fill the gap in the sentence: What I have is
underlined word and that will, at the same time, only a...........of the famous painting; the original is in
correctly fill the gap in the sentence: Instead of ......... the National Museum (a) duplicate (b) model (c) fake
the issue, the speaker digressed to unimportant (d) drawing
matters (a) summarizing (b) describing (c) revealing 11 From the words lettered A to D, choose the word that
(d) addressing best completes the following sentence: If the jury’s
3 From the list of words lettered A to D choose the one verdict is “not guilty”, the defendant will be ................
that is most nearly opposite in meaning to the (a) acquitted (b) indicted (c) pardoned (d) forgiven
underlined word and that will, at the same time, 12 From the words lettered A to D, choose the word that
correctly fill the gap in the sentence: The telephone best completes the following sentence: When all
booths were all......... and I had to wait for more than the items on the agenda had been discussed, the
ten minutes for one to become vacant (a) locked (b) meeting was ..............
busy (c) occupied (d) ringing (a) adjourned (b) dismissed (c) postponed (d)
4. From the list of words lettered A to D choose the one dissolved
that is most nearly opposite in meaning to the 13. From the words lettered A to D, choose the word that
underlined word and that will, at the same time, best completes the following sentence: Because
correctly fill the gap in the sentence: While the first the new venture was .,............many businessmen
battalion halted its march at nightfall, the and women was into it (a) rich (b) manageable (c)
second.......to the barracks (a) returned (b) set out (c) lucrative (d) satisfying
proceeded (d) turned up 14 From the words lettered A to D, choose the word that
5 From the list of words lettered A to D choose the one best completes the following sentence: We save a
that is most nearly opposite in meaning to the ............of lions at the game reserve (a) pride (b)
underlined word and that will, at the same time, family (c) band (d) group
correctly fill the gap in the sentence: His father is an 15 From the words lettered A to D, choose the word that
............driver but he is only novice (a) enlightened (b) best completes the following sentence: Many
excellent (c) poor (d) experienced educationists feel that...............punishment should
6. From the list of words letter A to D choose the one still be retained in schools (a) capital (b) corporal (c)
that is most nearly opposite in meaning to the corporeta (d) physical
underlined word and that will, at the same time, 16 From the words lettered A to D, choose the word that
correctly fill the gap in the sentence: The arrival of best completes the following sentence: Freedom
the police at the scene................. of movement is a ...........right of every citizen (a)
Rather than mitigated tension (a) provoked (b) fundamental (b) negotiated (c) privileged (d)
heightened (c) created (d) prolonged respected
7. From the list of words lettered A to D choose the one 17 From the words lettered A to D, choose the word that
that is most nearly opposite in meaning to the best completes the following sentence: The
underlined word and that will, at the same time, river......towards the sea (a) meandered (b) rolled (c)
correctly fill the gap in the sentence: Roses are ran (d) trickled
common in May but........................ 18 From the words lettered A to D, choose the word that
in November (a) dead (b) strange (c) rare (d) extinct best completes the following sentence:
8. from the list of words letter A to D choose the one I.............that our team will win the match tomorrow
that is most nearly opposite in meaning to the (a) certify (b)conclude (c) announce (d) predict
underlined word and that will, at the same time, 19 From the words lettered A to D, choose the word that
correctly fill the gap in the sentence: A bank clerk best completes the following sentence: If found
should be able to tell easily the difference guilty, he will............. all the wealth he has amassed
between........... notes and counterfeit ones (a) crisp illegally (a) forgo (b) abandon (c) repay (d) forfeit
(b) good (c) correct (d) genuine

© 2014 www.examsguru.net | All Right Reserved Enquiries: call +234 (0) 802 652 9647 Page 1
Examsguru.Net – LASU POST UTME PAST QUESTIONS WITH SOLUTIONS 2005 - 2014
20 From the words lettered A to D, choose the word that used in the sentence: Amina is the one who initiated
best completes the following sentence: (a) the quarrel, not Nike! (a) started (b) stopped (c)
movement (b) flow (c) velocity (d) drive settled (d) fuelled
21. From options A to D, choose the interpretation that 32 From the words letter A to D, choose the word that is
you consider most appropriate for this sentence: nearest in meaning to the underlined word as it is
Ladies and gentlemen, the worst may be over, but used in the sentence: She was reprimanded by her
we are not out of the woods yet. This means that we boss for negligence (a) sacked (b) rebuked (c)
are (a) still in the forest (b) not yet ready to celebrate punished (d) surcharged
our success (c) likely to encounter more difficulties 33 From the words letter A to D, choose the word that is
(d) not sure of ourselves nearest in meaning to the underlined word as it is
22 From options A to D, choose the interpretation that used in the sentence: Why should I retract my
you consider most appropriate for this sentence: statement when it is nothing but the truth? (a) alter
When i leave this country it will be for good. This (b) deny (c) withdraw (d) condemn
means that i will (a) never come back (b) leave for 34 From the words letter A to D, choose the word that is
better conditions elsewhere (c) become an nearest in meaning to the underlined word as it is
adventurer (d) improvement ways. used in the sentence: The doctor diagnosed the
23 From options A to D, choose the interpretation that tumour as malignant (a) deadly (b) painful (c) trouble
you consider most appropriate for this sentence: (d) minor
When she confronted her husband, he hung his head 35 From the words letter A to D, choose the word that is
and didn’t answer her questions. This means that he nearest in meaning to the underlined word as it is
was (a) defiant (b) ashamed and embarrassed (c) used in the sentence: In tropical; Africa, mosquitoes
caught unawares (d) angry with her are ubiquitous (a) dangerous (b) harmful (c) trouble
24 From options A to D, choose the interpretation that (d) everywhere
you consider most appropriate for this sentence: All 36 From the words letter A to D, choose the word that is
our plans fell through at the last moment. This means nearest in meaning to the underlined word as it is
that we (a)were exposed (b) delayed (c) were used in the sentence: It soon became obvious that a
abandoned (d) failed confrontation was inevitable (a) disastrous (b)
25 From options A to D, choose the interpretation that unavoidable (c) desirable (d) imminent
you consider most appropriate for this sentence: 37 From the words letter A to D, choose the word that is
The police knew that the suspect was not being nearest in meaning to the underlined word as it is
straight with them. This means that the suspect was used in the sentence: They insisted that the omission
(a) not telling the truth of my named had been inadvertent (a) unintentional
(b) not friendly (c) afraid (d) timid (b) unconditional (c) improper (d) unfortunate
26 From options A to D, choose the interpretation that 38 From the words letter A to D, choose the word that is
you consider most appropriate for this sentence: nearest in meaning to the underlined word as it is
After his prison experience. Etim decided to go used in the sentence: The police announced that
straight. This means that he decided to (a) hold his they were yet to apprehend the criminals (a)
head up and walk with defiance (b) change his interrogated (b) charge (c) prosecute (d) arrest
religion (c) live an honest life (d) stop using drugs 39 From the words letter A to D, choose the word that is
27 From options A to D, choose the interpretation that nearest in meaning to the underlined word as it is
you consider most appropriate for this sentence: used in the sentence: He is easily recognizable
The teacher spoke with his tongue in his cheek when because of conspicuous birthmark on his face (a)
he said that lazy Ade was the best pupil. This means dark (b) beautiful (c) prominent (d) natural
that the teacher (a) did not mean what he said (b) 40 From the words letter A to D, choose the word that is
was disappointed (c) was angry (d) tired to nearest in meaning to the underlined word as it is
encourage used in the sentence: The fishing boat was imperilled
28 From options A to D, choose the interpretation that by high winds, but it management to reach port safely
you consider most appropriate for this sentence: (a) pushed (b) endangered (c) disturbed (d) blown
They did not know what to expect and so decided to 41 From options A to D, choose the word or group of
play it by it ear. This means that they decided to (a) words that best completes the following sentence:
listen attentively to everyone (b) act according to Surprisingly, Musa turned..........the job offer (a) away
circumstance (c) pretend to be deaf (d) prepare for (b) in (c) out (d) down
the worst 42 From options A to D, choose the word or group of
29 From options A to D, choose the interpretation that words that best completes the following sentence:
you consider most appropriate for this sentence: We have no choice.............to do what they demand
Ojo decided that the best course of action was to (a) except (b) than (c) unless (d) but
play along with them for time being. This means that 43 From options A to D, choose the word or group of
he decided to (a) join their them temporarily (b) words that best completes the following sentence:
entertain them for a while (c) pretend to a greed with The committee has submitted its report............ the
them (d) be playful with them students (a) in (b) for (c) about (d) on
30 From options A to D, choose the interpretation that 44 From options A to D, choose the word or group of
you consider most appropriate for this sentence: words that best completes the following sentence:
The conference was attended by people from all The chairman told members that he was
walks of life. This means that conference was open.........suggestions (a) to (b) for (c) about (d) on
attended by people (a) of all races (b) who travelled
widely (c) from all professions (d) from different parts 45 From options A to D, choose the word or group of
of the world words that best completes the following sentence:
31 From the words letter A to D, choose the word that is The policeman shot the escaping robber........the leg
nearest in meaning to the underlined word as it is (a) on (b) in (c) at (d) into

© 2014 www.examsguru.net | All Right Reserved Enquiries: call +234 (0) 802 652 9647 Page 2
Examsguru.Net – LASU POST UTME PAST QUESTIONS WITH SOLUTIONS 2005 - 2014
46 From options A to D, choose the word or group of strategic reasons (c) undisclosed reason (d) all of the
words that best completes the following sentence: above
For the management, your remarks amount to a 6 Ministers of local government and chieftaincy affairs
slap...............the face (a) on (b) upon (c) at (d) in were abolished in Nigeria by: (a) General Yakubu
47 From options A to D, choose the word or group of Gowon (b) General Murtala Mohammed (c) President
words that best completes the following sentence: Ibrahim Babangida (d) General Olusegun Obasanjo
She does not remember.......to anyone at the park (a) 7 Policy analysis, policy implementation and plan
to talk (b) to be talking (c) talking (d) to have talked setting are some of the functions of: (a) the
48 From options A to D, choose the word or group of legislature (b) the executive (c) the local government
words that best completes the following sentence: (d) the civil service
This portion is to be shared between ................(a) us 8 Public corporations can be controlled through: (a)
and them (b) we and them (c) us and they (d) we riots (b) public opinion (c) civil disobedience (d) none
and the of the above
49 From options A to D, choose the interpretation that
you consider most appropriate for this sentence: 9. The general supervision of a public corporation is
After his prison experience. Etim decided to go carried out by the (a) board of directors (b) board of
straight. This means that he decided to (a) hold his trustees (c) managing director (d) secretary of the
head up and walk with definance (b) change his board
religion (c) live an honest life (d) stop using drugs 10 The local government in Nigeria is created to:
50. From options A to D, choose the interpretation that (a)create more civil service jobs (b) encourage
you consider most appropriate for this sentence: competition and rivalry among communities (c) bring
The teacher spoke with his tongue in his cheek when the government nearer to the people (d) prevent the
he said that lazy Ade was the best pupil. This means creation of more states
that the teacher (a) did not mean what he said (b) 11 The civil service embraces all workers in: (a) all
was disappointed (c) was angry (d) tried to private corporations (b) public and private companies
encourage (c) government ministries (d) public corporations
51. From options A to D, choose the interpretation that 12 The effective operation of the civil service in Nigeria
you consider most appropriate for this sentence: is mostly hampered by: (a) inadequate training of
They did not know what to expect and so decided to personnel (b) corruption and inefficiency (c) debt
play it by it by ear. This means that they decided to burden and redundancy (d) poor infrastructure
(a) listen attentively to everyone (b) act according to 13 The Bureau of public enterprises is charged with the
circumstances (c) pretend to be deaf (d) prepare for responsibility for: (a) privatization and
the worst commercialization (b) generation revenue (c)
52. From options A to D, choose the interpretation that eradicating poverty (d) providing employment
you consider most appropriate for this sentence: opportunities
Ojo decided that the best course of action was to 14 Financial allocation to a local government t by the
play along with them for time being. This means that federal or a state government supplement the cost of
he decided to (a) join their them temporarily (b) a project is called: (a) revenue allocation (b)
entertain them for a while (c) pretend to agree with reimbursement (c) statutory allocation (d) matching
them (d) be playful with them grant
53 From options A to D, choose the interpretation that 15 A permanent civil service: (a) make continuity in
you consider most appropriate for this sentence: government possible (b) makes civil servants
The conference was attended by people from all arrogant (c) promotes ethnic domination (d)
walks of life. This means that conference was undemocratic
attended by people (a) of all races (b) who travelled 16 One form of control exercised over public
widely (c) from all professions (d) from different parts corporations is the requirement that their annual
of the world. reports be laid before: (a) parliament for scrutiny (b)
all the political parties (c) all the local authorities in
GENERAL PAPER the country (d) joint committee of the executive and
1 Which of the following groups fall into the civil parliament
service: (a) the police, the army and the air force n(b) 17. Anonymity of the civil service means that the civil
employees of NEPA, NNPC and NRCA (c) servant must (a) serve any government impartially
employees of ministries of finance, education and (b) be politically neutral (c) have job security (d)
transportation (d) all of the above
not receive the credit or blame for any good or bad
2 The most senior class in the Nigerian Civil Service is
a(a) higher technical class (b) and professional class policy
(c) and technical class (d) sub-clerical and 18 The local government reform of 1976 in Nigeria
manipulative class were designed to (a) Decentralize authority (b)
3 The recruitment or appointment of the permanent enlist grassroots support (c) achieve even
secretary is one of the duties of: (a) the federal public development (d) all of the above
service commission (b) the state civil service 19 Bye-laws made by local authorities ca be declared
commission (c) the national assembly
unconstitutional only by the (a) local government
4. In the organizational structure of the ministry or
government department, offices and positions are: a service commission (b) ministry of local
hierarchically arranged (b) diagonally arranged (c) government and chieftaincy affairs courts (d)
secretly arranged (d) haphazardly arranged attorney general
5. The government maintains monopoly over certain 20. Mass retrenchment of workers in the public and
services for: (a) selfish reasons (b) security and private sectors is most like to result in (a) political
stability (b) economic survival (c) high rate of
© 2014 www.examsguru.net | All Right Reserved Enquiries: call +234 (0) 802 652 9647 Page 3
Examsguru.Net – LASU POST UTME PAST QUESTIONS WITH SOLUTIONS 2005 - 2014
armed robbery, pilfering and political instability (d) 34 The idea of making the local government the third
electoral malpractices tier of government was initiated by (a) Abdusalam
21. Public corporations are established to (a) look after Abubkar regime (b) Alhaji Shedu Sahagari regim
the affairs of local authorities (b) coordinate the (c) Murtala Obasanjo regime (d) Ibrahim
activities of ministries (c) give advice to the Babangida regime
government 35. Before the 1976 local government reforms, one of
22 Which of the following is not a source of local the defective features of the local governments in
government revenue (a) state and federal Nigeria was that: (a) they had no functions to
government grants (b) licensing of cars and lorries perform (b) they had no legal personality (c) they
(c) market stall fees (d) returns on investment had no chairmen to pilot their affairs (d) they had
23 One of the major reasons for setting up public no political aspiration
corporations is to (a) maximize profit (b) compete 36. One of the major problems which spelt doom for
with private companies (c) provide essential Nigeria Airways was (a) Embezzlement of fund (b)
services (d) encourage corruption (c) lack of patriotism (d) all of the above
24 All of the following are functions of the civil service 37 The main cause of infrastructure decay in Nigeria
except (a) making laws (b) implementing policies is:
(c) preparing financing estimates (d) implementing (a) illiteracy (b) disobedience (c) lack of
edicts maintenance culture (d) political instability
25 One factors which militates against the effective 38 One of the measures that will enhance the status
functioning of the civil service is (a) delegated of the local government as a third tier of
legislation (b) political interference (c) judicial government is (a) the creation of more local
inference (d) political stability government areas (b) up-grading the local
26 Being the third tier of government, the local government to statehood (c) drafting of separate
government is therefore (a) subordinate to state constitution for local government (d) deduction of
and federal government (b) antagonistic to state local government share of federal allocation
and federal government (c) coordinate to state and directly from source
federal government (d) all of the above 39. To enhance the independence of the federal public
27 One major problem facing public corporations in service commission, members should (a) elected
Nigerians (a) political parties (b) excessive from a national party (b) take oath of celibacy (c)
patriotism (c) government interference (d) judicial neither belong to the legislative nor executive
manipulation branch of government (d) be appointed by the non-
28 Most of the reasons given for the establishment of aligned movement
public corporations in Nigeria are being 40 To be promoted from one grade level to another, a
contradicted by the current wave of (a) privatization staff must first (a) apply to the Nigeria export
and commercializarrion (b) legalization and promotion council (b) petition the civil service
nationalization (c) judicial and legislative commission (c) be in the president or governor’s
competence (d) rigging and electoral brouhaha list (d) be recommended to the public service
29 The recruitment, promotion and discipline of civil commission by his or her department head
servants in Nigeria is the responsibility of (a) board 41. to be entitled to pension in Nigeria, a staff must ()
of directors (b) civil service commission (c) the work for 55 years (b) work for at least 10 years
president (d) ministry of labour and productivity consecutive years (c) work for 65 years (d) attain
30 Engineers and architects in the civil service fall into the age of seventy
the (a) professional class (b) technical class (c) 42. The dismissal of a staff in the ministry for official
higher technical class (d) the manipulative class misconduct is the prerogative is (a) the permanent
31. The main functions of the administrative class of secretary (b) personnel manager (c) the minister
the civil service include (a) policy making (b) (d) the public service commission
implementation of government policies (c)
enactment of laws for the ministries (d) all of the
above 43 the greatest head-ache affecting revenue
32 The relationship between staffs of the civil service generation by NEPA is (a) debts owed it by
in the discharge of their duties is expected to be (a) government department’s and officials (b) refusal
personal and unofficial (b) official and non— of NEPA men to collect revenue (c) its instability to
personal (c) causal and inconsistent (d) illogical employ accountants (d) none of the above
and sporadic 44. Public corporation in Nigeria are subject to the
33 The first local government system adopted in control of (a) the judiciary (b) the minister in charge
Nigeria by the regional government was (a) the (c) public service commission
French professorial system (b) the Indian local 45 the public corporation is similar to the joint stock
government system (c) the Russian socialist company because (a) the chairmen is also the
system (d) the British council system managing director (b) their administrative centres

© 2014 www.examsguru.net | All Right Reserved Enquiries: call +234 (0) 802 652 9647 Page 4
Examsguru.Net – LASU POST UTME PAST QUESTIONS WITH SOLUTIONS 2005 - 2014
are far from their main factories (c) the two are 10. If un =2,6,10,… the sum of the terms of the
legal entities (d) they share the same market sequence is …………(a) 1104 (b) 1058 (c) 968
46 Which of the following statements both describes a (d) 1012
public corporation? (a) it is an organ of government
responsible for executing the policies of 11 In an arithmetic sequence with 25 terms, if the first
government (b) it is a local body that renders term is 60 and last term is -12 the ………(a) 3 (b) 2
services on local basis (c) it is a legal body (c) -3 (d) -2
established by an act of state to provide essential 12 In an arithmetic sequence with 25 terms, if the first
services. (d) it is a body owned by members of the term is 60 and last term is -12, the sum of the
public. terms of the sequence is …………….(a) 600 (b)
47 ‘Red tapism’ can be explained as (a) the 2400 (c) 1800
decentralized way of taking decision (b) a flexible 13 In an arithmetic sequence whose 4th term is 14 and
way by which government decisions are taken (c) whose 11th term is 70, the first and common
the rigid adherence to routines by civil servants (d) difference are
(a) a=-18, d=8 (b) a=-7,d-7 (c) a=10,d=8 (d)
management by objectives
a=14,d=7
48 An institution which seeks to redress people’s 14 In an arithmetic sequence whose 4th term is 14 and
grievances against abuse of administrative power whose 11th term is 70, the sum of the first 12 terms
is the (a) ombudsman (b) Judiciary (c) directorate of the sequence is ....................(a) 468 (b) 330 (c)
of public prosecution (d) judicial service 456 (d) 408
commission 15 Ann goes swimming regularly. She want to improve
49 A statutory corporation is under the supervision of her fitness, so she decides to swim 10 lengths in the
first session and increase the number of lengths she
(a) the chief justice (b) the commissioner of police
swims by 2 every session. When she reaches 50
(c) a minister (d) a local government chairman lengths in a session she will not increase the number
50 To which class of the civil service does the causal any further. After how many session does Ann swim
or manual labour force belong (a) the technical 50 lengths for the first time? (a) 21 (b) 22 (c) 20 (d)
class (b) the casual class (c) the manipulative class 19
(d) the higher technical class 16 Ann goes swimming regularly. She wants to improve
her fitness, so she decides to swim 10 lengths in the
first session and increase the number of lengths she
MATHEMATICS 2013/2014
swims by 2 every session. When she reaches 50
1. Which of the following points does not lie on the lie lengths in a session she will not increase the number
2y + 5x – 4 = 0? (a) (0,8,0) (b) (1,-0,5) (c) (0,2) (d) any further. After many sessions does she get her
(2,3) reward? (a) 15 (b) 25 (c) 16 (d) 24
2 A straight line has the equation 107 = 3x + 15. 17 Ann goes swimming regularly. She wants to improve
Which of the following is true? (a) the gradient is her fitness, so she decides to swim 10 lengths in the
first session and increase the number of lengths she
0,3 and they y-intercept is 1.5 (b) the gradient is 3
swims by 2 every session. When she reaches 50
and the y-intercept is 1.5 (c) the gradient is 15 and lengths in a session she will not increase the number
the y-intercept is 3 (d) the gradient is 1.5 and the y- any further. If Ann asks her friend Joy to come
intercept is 0.3 swimming with her. Joy starts coming at Sue’s 8th
3 P is the point (2,7) and Q is the point (6,3). What is session. Joy starts to swim 15 lengths and increases
the gradient of PQ? (a) 1 (b) -1 (c) 0.5 (d) -0.5 the number of the lengths by 5 each time. After how
4 P is the point (3,5). Q is the point (-1,9). R is the many of Joy’s sessions does she swim the same
number of lengths as Ann? (a) 10 (b) 5 (c) 11 (d) 4
midpoint of PQ. Which one of the following lines
18 A dealer increased the price of an item by 20%, then
does R lie on? (a) y=x+6 (b) y=x+8 (c) y=x-6 (d) increased the price of the same item by 30%. If x is
y=x-8 the original price, what is the price after the two
5 a straight line has a gradient of -2 and passes increases? (a) 1.5x (b) 1.56x (c) x + 0.5 (d) x + 0.56
through the point (4,1). What is the equation? (a) (e) x + 6
y+2x=6 (b) y=2x-6 (c) y+2x-9=0 (d) 2y=x-2 19 Two dice are thrown. What is the probability that the
6 The line y = 5x-3 and y=2x + 9 intersects at P. sum of the two numbers obtained is greater than 10?
(a) 1/12 (b) 1/36 (c) 1/6 (d) 1/4 (e) 1/2
What are the coordinates of P? (a) 2,7) (b) 2,13)
20. If x and y are two real numbers such that 3x + 2y=5
(x) 4,17) and 5x +4y=9, then 4x + 3y= (a) 0 (b) 2 (c) (d) 6 (e) 7
(d) -4,-23) 21 If m and n are positive integers are such that, which
7. A is the point (1,5), B is the point (4,7) and C is the of these values cannot be values of m and n?
point (5,2), Triangle ABC is (a) right-angle (b) (a) m=12 and n=18 (b) m=60 and n=90 (c) m=34 and
scalene with no right angle (c) equilateral (d) n=51 (d) m=7 and n=21 (e) m=102 and n=153
22 If 35 is the median of the data set including 21, 7, 45,
isosceles
th 33, 62 and x, then x= (a) 3b (b) 14 (c) 37 (d) 33 (e)
8. If un = 2,6,10…… the 12 term of the sequence is 48
………….(a) 46 (b) 50 (c) 48 (d) 44 23 The .............is the value you calculate when you
9 If un = 2,6,10 ….. the last term of the sequence is want the arithmetic average (a) mean (b) median (c)
90. The number of terms in the sequence is mode (d) all of the above
………..(a) 20 (b) 22 (c) 24 (d) 23

© 2014 www.examsguru.net | All Right Reserved Enquiries: call +234 (0) 802 652 9647 Page 5
Examsguru.Net – LASU POST UTME PAST QUESTIONS WITH SOLUTIONS 2005 - 2014
24 The ..............is often the preferred measure of central form (c) a table must be self sufficient (d) a table
tendency if the data are severely skewed (a) mean must be self explanatory (e) none of the above
(b) median (c) mode (d) range 41 Which of these is a measure of location? (a) mean
25 Which of the following is the formula for range (a) H + (b) standard deviation (c) variance (d) all of the
L (b) L x H (c) L – H (d) H - L above (e) none of the above
26 ................are used when you want to visually 42. Statistics is a set of tools whose proper use
examine the relationship between two quantitative will………..the decision maker (a) completely fill
variables (a) bar graphs (b) pie graphs (c) line graphs the needs of (b) encumber (c) aid (d) confuse (e)
(d) scatterplots none of the above
27 As general rule, the .............is the best measure of 45 A company employs 100 people, 65 of whom are
central tendency because it is more precise (a) mean men, 60 people including all the women are paid
(b) median (c) mode (d) range weekly. The number of the men that are paid
28 The most frequently occurring number in a set of weekly is (a) 35 (b) 40 (c) 25 (d) 30 (e) none of the
values is called the ..............(a) mean (b) median (c) above
mode (d) range 46 In a survey of villagers, it is found that 20% of the
29 A rectangular box with a square base and no top has people have visited Kano and 25% have visited
a volume of 500cm, the dimensions of the box that Port Harcourt. If 5% have been to both cities, then
require the least amount of material are (a) the percentage that have visited neither Kano nor
10x10x5cm (b) 4x5x5cm (c) 50x5x2cm (d) Port Harcourt is (a) 75% (b) 65% (c) 50% (d) 60%
10x50x1cm (e) 70%
30 The volume of the region bounded by the y-axis, y=4 47 If the sides of a triangle are and units, then the
and y=2 is (a) 192p/5 (b)52p/3 (c) 64p (d) 52p/5 (e) triangle is (a) equilateral triangle (b) well defined
192p/7 triangle (c) isosceles triangle (d) undefined triangle
31 A newspaper article reported that “children who (e) right-angled triangle
routinely compete in vigorous after school sportson 48 The length of a rectangle is three times its width, if
hazy days are three times more likely to get asthma the perimeter is 72cm, the width of the rectangle is
than their non-athletic peers” Of the following, which (a)6cm (b) 8cm (c) 9cm (d) 10cm (e) 11cm
is the most important additional information that 49 The capacity in litres of bucket 24cm in diameter at
would be useful before making decision about the top, 16cm in diameter at the bottom and 20cm
participation in school sports? (a) where was the deep is (a) 40 (b) 9.15 (c) 2.78 (d) 6.08 (e) 6.37
study conducted (b) How many students in the study 50 The area in cm2 of material required for a
participated in after-school sports? (c) what is the lampshade in the form of a frustum of a cone which
baseline risk for getting asthma? (d) who funded the top and bottom diameters are 20cm and 30cm
study? (e) none of the above respectively, and the vertical height is 12cm is (a)
32 The roof of a shelter is made from a piece of 1021 (b) 7200 (c) 3600 (d) 2400 (e) 2041
corrugated iron 2.3m long inclined at 18o to the 51 A frustum of a pyramid is 3cm square at the top
horizontal. How far from the wall does the roof stick and 6cm square at the bottom and is 5cm high.
out? (a) 0.1m (b) 2.2m (c) 1.1m (d) 1.2m (e) 2.1m The volume in cm3 of the frustum is (a) 15 (b) 150
33 A ladder 20m long rests against a vertical wall so that (c) 105 (d) 115 (e) 36
the foot of the ladder 9m from the wall. The height 52 If x =2, then (x-1) (2x-3) = (a) 1 (b) 3 (c) -2 (d) 0 (e)
(correct to 1 decimal place above the ground at -3
which the upper end of the ladder touches the wall is 53 What is sin 30? (a) ½ (b) 3/2 (c) 0.866 (d) 1 (e) 2
......................(a) 19.7m (b) 18.1m (c) 18.7m (d) 54 Given the list of numbers
17.1m (e) 17.9m {1,6,3,9,16,11,2,9,5,7,12,13,8}, what is the
34 A chord 6.6m long is 5.6m from the centre of a circle median? (a) 7 (b) 8 (c) 9 (d) 11 (e) 6
<C> The radius of the circle is (a) 3.2m (b) 6.3m (c) 55 What is the slope of the line that is perpendicular to
6.5m (d) 1.6m (e) 2.56m y – 2x =1? (a) 2 (b) -2 (c) ½ (d) -½ (e) 1
35 The heights in cm, of 10 children are 145, 163, 159, 56 What is the sum of the first 40 even positive
162, 167, 149, 150, 170 and 155. The mean height integers? (a)1,600 (b) 1,560 (c) 820 (d) 1,640
of the children is (a) 156cm (b) 158cm (c) 160cm (d) 57 If an item is purchased at $150 and sold at $165,
16 162cm (d) 159cm what percent of the original cost is the profit (a) 110
36 The heights in cm, of 10 children are 145, 163, 159, percent (b) 89 percent (c) 100 percent (d) 9
162, 167, 149, 150, 170 and 155. The standard percent (e) 10 percent
deviation of the heights of the children is (a) 5.5cm 58 What is the length of an arc of a circle with a radius
o
(b) 5.7cm (c) 6.5cm (d) 6.7cm (e) 7.7cm of 5 if it subtends an angle of 60 at the center? (a)
37 The express 10.011 two in decimal (a) 1.625 (b) 3.14 (b) 5.24 (c) 10.48 (d) 2.62 (e) 4.85
2.375 (c) 6.375 (d) 6.75 (e) 2.75 59 If the distance between a 13-foot ladder and a
38 A class of all possible subsets of space S is called vertical wall is 5 feet along the ground , how high
...........(a) universal set (b) alpha-field (c) sample can a person climb if the ladder is inclined against
space (d) probability the wall? (a) 18 feet (b) 65 feet (c) 13/5 (d) 8 feet
39 One of these is a demerit of a sample (a) it is (e) 12 feet
cheaper to enumerate a sample (b) it is faster to 60 The value of k so that the sequence k-1, k+3, 3k-1
survey a sample (c) Results obtained from sample forms an arithmetic progression is ………….(a) 6
are often times as informative as those from a (b) 5 (c) d (d) 3 (e) 2
censor (d) all of the above (e) none of the above 61 If sec u=1,414, sin u= (a) 0.5 (b) 0.71 (c) 0.86 (d)
40. One of these is not a desirable feature of a good 1,414 (e) 0.67
statistical table (a) a table must reveal salient 62 If (2x-y) =4 then (6x-3y)=…………….(a) 15 (b) 12
features of data () a table must clearly (c) 18 (d) none
communicate information in a neat and concise

© 2014 www.examsguru.net | All Right Reserved Enquiries: call +234 (0) 802 652 9647 Page 6
Examsguru.Net – LASU POST UTME PAST QUESTIONS WITH SOLUTIONS 2005 - 2014
63 The simple interest on a certain sum of money for 14 A 14 D
three years is 225 and the compound interest on 15 B 15 A
the same sum at the same rate for 2 years is 153 16 A 16 A
then the principal invested is (a) 1500 (b) 2250 (c) 17 A 17 D
3000 (d) 1875 (e) 2000 18 D 18 D
64 The probability of getting a number greater than 4 19 A 19 C
in a single throw of a die is (a) ½ (b) 2/3 (c) ¼ (d) 20 C 20 C
1/3 (e) 1/6 21 C 21 D
65 After allowing a discount of 11.11% a trader still 22 A 22 B
makes a gain of 14.28%, at what percentage of 23 B 23 C
interest dose the trader mark his goods? (a) 28.5% 24 D 24 A
(b) 35% (c) 22.11% (d) 20% (e) none 25 A 25 B
66 The population f a town was 160,000 three years 26 C 26 C
ago. If it increased by 3%, 2.5% and 5% 27 A 27 C
respectively in the last three years, then the 28 B 28 A
present population of the town is (a) 177,000 (b) 29 C 29 B
177,366 (c) 177,461, (177,596, (e) 177,564 30 C 30 A
67 Average age of students in an adult school is 40 31 A 31 B
years. 120 new students whose average age is 32 32 B 32 B
joined the school. As a result, the average age is 33 C 33 D
decreased by 4 years. The number of students of 34 A 34 C
the school after joining of the new students is (a) 35 D 35 B
1200 (b) 120 (c) 360 (d) 240 (e) 300 36 B 36 D
68 A man owns 2/3 of the market research bureau 37 A 37 C
business and sells ¼ of his shares for $75000. the 38 D 38 D
value of the business is (a) $150000 (b) $130000 39 C 39 C
(c) $ 240000 (d) $ 340000 (e) none 40 B 40 D
69 The cost of 16 packets of salt each weighing 900 41 D 41 B
gram is $28. What will be the cost of 27 packets, if 42 D 42 D
each packet weighs 1 kg? (a) $ 52.50 (b) $56.00 43 D 43 A
(c) $58.50 (d) 54.50 44 A 44 C
70 Chukwuma and Chinwe have two children. The 45 A 45 C
probability that the first child is a girl child is 50%. 46 D 46 C
The probability that the second child is a girl is also 47 C 47 C
50%. Chukwuu and Chinwe revealed they have 48 A 48 A
daughter. The probability that their other child is 49 C 49 C
also a girl is ………. 50 A 50 C
(a) ½ (b) 1/3 (c) ¼ (d) 1/5 (e) 1 51 B
71 A three digit number consists of 9,5 and one more 52 C
number. When these digits are reversed and then 53 C
subtracted from the original number the answer
yielded will be consisting the same digits arranged MATHEMATICS
yet in a different order. The missing digit is (a) 1 (b) 1 A -
c (c) d (e) 6 2 A
72. If x=2, then (x-1) (2x-3) = (a) 1 (b) 3 (c) -1 (d) 11
3 B
73 Line m has a negative slope and negative y-
intercept. Line n is parallel to line m and has a 4 D
positively intercept. The x-intercept of n must be 5 C
(a) negative and greater than the x-intercept of m 6 C
(b) negative and less than the x-intercept (c) zero 7 A
positive and greater than the x-intercept of m (e) 8 A
positive and less than the x-interception 9 D
10 A
ANSWERS
11 C
ENGLISH LANGUAGE GENERAL PAPER 12 A
1 B 1 C 13 C
2 D 2 B
3 C 3 C
4 C 4 A
5 D 5 B
14 D
6 B 6 A
7 C 7 D 15 A
8 D 8 B 16 C
9 D 9 C 17 D
10 A 10 C 18 D
11 A 11 C 19 A
12 A 12 B
20 A
13 C 13 A

© 2014 www.examsguru.net | All Right Reserved Enquiries: call +234 (0) 802 652 9647 Page 7
Examsguru.Net – LASU POST UTME PAST QUESTIONS WITH SOLUTIONS 2005 - 2014
21 D 3 A solid sphere of radius x cm is placed in a cylinder
22 C of radius 2x cm and height 2x cm. The cylinder is
23 A then filled with water to the brim and the solid gently
24 B withdrawn. Find the volume of the water in the
3 3
25 D cylinder in cm (a) 28/3X
3 3 3
26 B (b) 24X (c) 20/3X (d) 8X
27 A 4 The earth rotates on its own axis once in 24hrs.
28 C What is the speed in km/hr of a place whose latitude
29 A o 4
is 30 5 (Take 2R to be equal to 4 x 10 km) (a)
30 C 2,140km/hr (b) 1,443km/hr (c) 1,200km/hr (d)
31 C 1,000km/hr
32 E 5. The minor sector of a circle of diameter 3.6cm
33 E o
subtends angle 35 at the center. What is the
34 A perimeter of the sector? (a) 5.8cm ( b) 4.7cm (c)
35 B 2.9cm (d) 1.1cm
36 E 6.
o
A regular polygon of (2k + 1) sides has 140 as the
37 B size of each interior angle. Find k (a) 4 (b) 4½ (c) 8
38 A (d) 8½
39 C 7. Solve the following simultaneous equation
40 E 2 2
X + Y = 10, x + y = 58
41 D (a) x = 7, y=3 or x=3,y=7 (b) x= -7, y= -3 or x= -3,
42 C y= -7
43 E (c) x= -7, y=3 or x=3, y= -7 (d) x=7, y= -3 or x = -3,
44 C y=7
45 C 8 A man is x years old which is son is y years old.
46 E The sum of their ages is twice the difference of their
47 E ages. If the product of their ages is 675, find the
48 C age of the man
49 D (a) 40years (b) 42years (c) 55years (d) 45years
50 D 9 Let the universal set U = [1, 2, 3, 4, 5, 6]
51 C o
A = {1,2,3,} and B {2,4,6} then A n B is (a) {2} (b)
52 C (1,3}
53 A (c) {4,6} (d) {1,2,3,5}
54 B 10 Simplify 8 + 50 as far as possible
55 A 7
56 D 2 2
11 Factorize fully: px – py + qy = qy
3 2

57 E 2
(a) (p q) (x + y) (x – y) (b) (p-q) (x + y )
2

58 - 2
(c) (p + q) (x –y) (x + y) (d) (p-q) (x + y )
2

59 E 12 Evaluate log3 9-log27 3+ log 8 9


60 C 1
(a) 6 /3 (b) 5½ (c) 9 (d) 5 /3
2

61 B 13 In a class of 30 students, there are 10 who wear


62 B spectacles and 16 girls. There are 8 boys who do
63 - not wear spectacles. How many girls wear
64 D spectacles?
66 B (a) 3 (b) 4 (c) 5 (d) 6
67 B 14 Solve the equation log2 x-log2 (x-1) = 2
68 A (a) 2 (b) 1 1/8 (c) 11/99 (d) no solution
69 C 2 2
15 (x-2) is a factor of x 3x +kx + 14, the value of k is
70 A (a) -5 (b) -2 (c) 2 (d) -3
MATHEMATICS - 2011/2012 2
16 Factorize the polynomial x – 7x + 6
1. A solid is made up of a hemisphere of radius x cm, (a) (x-3) (x-1) (x + 2) (b) (x-3) (x – 1) (x – 2)
2
and a cone of height x cm of the same radius as (c) (x + 3) (x + 1) (x + 2) (d) (x – 1 (x – 6)
the hemisphere. What is the volume of the 17 y is inversely proportional to the square of x. When
3 3
composite solid? (a) 2/3X (b) 15/3X (c) x =3, then y =4. Find the constant of proportionality
12/3X (d) X
3 3
(a) 48 (b) 4/9 (c) 2.25 (d) 36
2 What is the difference in the local time between two 18 The solution to the inequality 5 2x > 11 – 4x is
o
places in latitude 55 N if they are located at (a) x > 3 (b) x>3 (c) x>1 (d) x<1
o
longitudes 8oW and 18 E respectively. (a) 60mins 19 If x 2 = x 3 , find the value of x
(b) 80mins (c) 98mins (d) 104mins 1 5 2 5
(a) 2 (b) -2 (c) 0 (d) 1

© 2014 www.examsguru.net | All Right Reserved Enquiries: call +234 (0) 802 652 9647 Page 8
Examsguru.Net – LASU POST UTME PAST QUESTIONS WITH SOLUTIONS 2005 - 2014
32 Change 671nine to base 8
20 The determinant o f 1 2 0 is (a) 550eight (b) 540eight (c) 671eight (d) 1046 eight
3 2 1 33 Write 14/5 – 3 in the form a 5 + b 3, where a and
4 2 1 b are rational (a) 7 (b) 7 5 + 53 (c) 7 5 – 7 3
(d) 7 3 + 7 5
(a) 0 (b) 2 (c) 02 (d) 1 34 In the relation logx y=z, write x in terms of y and z
y y z/y t/y
(a) x = z (b) x = z (c) x = z (d) d x=z
21 In fig. 1, O is the centre of the circle 35 Solve the equation x+ 7 = x-5
o
<AOB = 130 Find <ACB (a) 9 (b) 5, -7 (c) 2 (d) 2,9
O 36 Let y = 4x + 3/2x – 5, write x as a function of y
(a) x=2y-3/5y+3 (b) x=5y-3/2y+4
(c) x=5y+3/2y-4 (d) x=(5y +3) (2y-4)
37 The second and fifth term of a geometric
A B
progression are 6 and -48, respectively. The first
term is (a) -3 (b) 3 (c) 12 (d) -12
C
38 Find the positive solution of the equation log (x + 2)
(a) 115 (b) 135 (c) 79 (d) 65 + log (x + 4) = 1 (a) 6 (b) 0 (c) 2 (d) 1
22. Fig. 2 shows circle of radius 4cm, the area of the 39 N72,000 is invested at 80% simple interest. After
2 2 2
shaded segment is (a) 4cm (b) 4 8cm (c) 84cm how many years has it reached N87,840?
2
(d) 2 -4cm (a) 2 ¼ years (b) 2 years (c) 3years (d) 2½
40 Suppose that p is the probability that an event
occurs, and that q is the probability that the event
does not occur. Which of the following is true? (a)
p=q (b) p+q=1 (c) pq=1
Fig. 2 41 Suppose x and y are positive numbers for which
x>y. Which of the following is not true?
23 Fig.3 shows a pyramid on top of a cuboid. The 2 2
(a) x >y (b) x<-y (c) 1/x>1/y (d) 3x>2y
height of the cuboid is Hem, the height of the 42 Fig. 4 shows a trapezium. The height is 8m, one the
2
pyramid is b, cm, and the square base of both parallel side is 10m and the area is 104m
shapes has side s cm, find the volume of the shape.
8cm
Fig. 4

(a) 16cm (b) 10m (c) 13m (d) 10.4m


3 2
43 Find the remainder when x 3x + 4x – 7 is divided
(x + 2)
2 3 2 3 2 2
(a) s (H + h)cm (b) s (H t h)cm (c) ½ s (2H + h) (a) -3 (b) -7 (c) -35 (d) x = 5x + 14
3 2 3
cm 44 if dy/dx = 6x + 15x and y=7 when x=2, find y
3 5 2
(a) 2x + 3x + 7 (b) 12x + 60x – 497
2 3
24 If y = sin (x + 7), then dy is (c) 12x + 60x – 7 (d) 2x + 3x5 – 105
2 2
(a) 2x cos = (x + 7) (b) (2x + 7) cos (x + 7) 45 The long hand minute of a clock is 7cm long. What
2
(c) -2 cos (x + 7) (d) 2x cos x distance does to the tip of the minutes move 1¼
25 The line y = kx 3 is perpendicular to the line 2y + 3x hours? (take  =22/7) (a) 33cm (b) 44cm (c) 55cm
= 7. the value of k is (a) -1/3 (b) -2/3 (c) 2/2 (d) 2/3 (d) 65cm
26 The midpoint of the line segment joining (-1, 3) and
(5,7) is (a) (3,5) (b) (3,2) (c) (2,5) (d) (1,6) Question 46 and 47 refers to the points A(-2,3) and
27 The solution of the inequality x2 + 3x 10 < o is B(4,5)
(a) -2<x<5 (b) x<-5 pr x>2 (c) 2<x<5 (d) -5<x<5 46 The distance /AB/is: (a) 10units (b) 8units (c)
o
28 A binary operation is defined by a b = a + b 3. the 40units
identity is (a) 3 (b) -3 (c) 1 (d) 0 (d) 14units
o
29 A binary operation is defined by a y = xy x + y, the 47 The midpoint of AB is (a) (3,-4 (b) (-11) (c) (1,-1) (d)
o o
value of (3 4) 5 is (a) 81 () 61 (c) 57 (d) 73 (-3,4)
30 Find the difference between mean and the median 48 Find the sum to infinity of the series
of the numbers 1, 2, 3, 4, 5, 7, 9 and 10 (a) 0 (b) 1/9 ½ ¼ + 2/5 – 1/16 + …………
(c) 5 (d) 4/9 (a) 1 (b) 1/3 (c) 2/3 (d) 2
31 There are eight men and nine women on a 49 Find the solution set for the set (x-2) (x-1) > 0
committee. In how many ways can a subcommittee (a) x>2 (b) x<2 (c) x<1 (d) x<1 or x>2
two men and three women be chosen? (a) 2,352 50 The solution set of the inequality /2x + /<10 is
(b) 112 (c) 6,188 (d) 28,224 (a) (-3,2) (b) (-5,2) (c) -8,2) (d) (-3,8)
© 2014 www.examsguru.net | All Right Reserved Enquiries: call +234 (0) 802 652 9647 Page 9
Examsguru.Net – LASU POST UTME PAST QUESTIONS WITH SOLUTIONS 2005 - 2014
th
51 Write the 7 term of the sequence {1+(-1)”} 75 If 2m + 3m/4m=5m=2, then 5m + n/2m + n is equal
(a) 0 (b) 1 (c) 2 (d) -1 to:
52 If x, 2x + 1, 3x a form an A.P. find a (a) 2 (b) -2 (c) 1 (a) 71/32 (b) 32/71 (c) 2 () 4
2
(d) -1 76 if 2x px + 6 = (2x 6) (x-1), then p is equal to
53 The fifth term of the sequence 1, 21, 51, (a) -8 (b) 4 (c) 8 (d) -4
91………….is 77 Which of the following is not a quadratic
(a) 131 (b) 141 (c) 151 (d) 161 expression?
2 2 2
54 Let X = (a, b, c, d) which statement is correct (a) x -5 (b) x(1-2x) (c) x(1 + x ) (c) x -3/8x + 1
(a) { a } x (b) {a, b) x (c) b x (d) n(x)=4 78 In fig. 5 below, RST is a tangent to the circle centre
55 The distance from the points (3,2) to the line 3y + 2x O. it touches the circle at S.U and V are at the ends
+ 5=0 is (a) 5/13 (b) 15/13 (c) 7/ 23 (d) 7/ 5 of a diameter and <SUV = 49o. Find < RSU.
56 Find the slope of the line which is perpendicular to
S T
the line 3x + 5y + 17 = 0 (a) 5/3 (b) -3/5 (c) -5/3 (d)
17/5
57 Find the intercept on the x and y axis respectively of R V
U
the line 3x-2y + 6=0 (a) (3,2) (b) (2,3) (c) 3,2) (d) - O
2,3)
2 o o o o
58 If f (x + 2) = 3x – 2x + 5, find f (1) (a) 8 (b) 10 (c) 6 (a) 48 (b) 138 (c) 42 (d) 90
o
(d) 3 79 The bearing of A from B is 280 . Find the bearing of
2 o o o o
59 If a and b are the root of the equation 2x – 3x – 9 = B from A (a) 80 N (b) 100 (c) 100 (d) 90
2 2–
0, find a + b (a) -1/3 (b) 2/3 (c) 3 (d) 1/3
60 The nth term of a sequence is given by Un = 2 + use frequency table below to answer question 80 to
3Un-1 while the U4 = 36 + U3, find the third term of 82
the sequence.(a) 1 (b) 5 (c) 17 (d) 51 X 0 1 2 3
2
61 If a and b are the roots of the equation 2x – 5x + 6 Frequency 20 18 7 5
2 2
= 0 find a + b (a) ½ (b) 3 (c) -5/2 (d) ¼
62 If x-2 and x+1 are factor of equation x2 + px2- 80 Calculate the mean of x. (a) 1.5 (b) 0.47 (c) 0.94 (d)
4xq=0, determine p, q (a) -3,12 (b) 3,-12 (c) -3,-12 1
(d) -1,0 81 What is the median of x? (a) 0 (b) 1 (c) 25.5 (d) 0.94
63 If x + 25/(x+1)(x-2)=P/5+2+q/5-2,find P 82. What is the range of x? (a) 15 (b) 0 to 3 (c) 5 to 20
(a) 3 (b) 2 (c) 1 (d) -8 (d) 3
64 A 16m ladder is placed against house so that its
base is 8m from the house. What angle does the 83 OAB is a sector of a circle of radius 8cm and centre
o o
ladder makes with the ground? (a) 65 (b) 60 (c) 0 in fig. 6 below. The length of the arc AB is 8cm.
o o
34 (d) 10 Find the area of the sector.
A
o
65 Find the trigonometric function value of cos (315 )
(a) 3/2 (b) 2 (c) ½ (d) undefined O 8cm
o
66 Convert-320 to radian measure. Give answer
using 3, 14 for  (a) 32 (b) 2.57 (c) -5.58 (d) 1.31 B
2
67 Solve for sec x 2/4 sec x =2/2
2 2 2 2
(a) 3 + 41/9 (b) 3-41/5 (c) 2/5 (d) 2 +22/5 (a) 32cm (b) 64cm (c) 30cm (d) 60cm
68 Given that tan θ is in the second quadrant. Find sin
2θ 84 In fig.7 below, find the value of x
o o o o
(a) -2/35 (b) 30.78 (c) -24/25 (d) 18 (a) 141 (b) 90 (c) 97 (d) 112
69 Find arc sin 0.2334 in degrees, using tables (a) 2.91 x
o 1 o 1 o 1
(b) 21 , 30 (b) 13 , 31 (d) 13 , 40
70 Find the components of this vector u + v, where u= o
Fig. 7 112
(3,7) and v=(4,2) (a) (7,-5) (b) (u,-6) (c) (2,5) (d)
3,+5)
71
2
Simplify (4-x2 ) (2+x)-½ (a) (2+x) (2-x) (b) (2-x) 7
(2+x)
(c) (2-x)/ (2-x) (d) (2 +x)/ (2-x) 85 What value of K makes the expression p2-18p + k a
72 Simplify (2-3) (a) (10-93 (b) 8-33 (c) 26-153 (d) perfect square? (a) -9 (b) 9 (c) -81 (d) 81
86 For what value of x is the function y=7/x+3 not
26-93
defined
73 The sum of an infinite geometric progression is 23/7
(a) 7 (b) 0 (c) -3 (d) 3
and the first term is 4. what is the common ratio? 3 2
87 Evaluate (4x10 ) x-(6x10 ), giving your answer in
(a) -23/4 (b) -3/4 (c) ¾ (d) 23/4 4 6
standard form (a) 2400000 (b) 24x10 (c) 2.4 x10
74 Evaluate x3 + 1/2x2 x 1 when x =-1 3
(d) 4.6 x 10
(a) 0 (b) -1 (c) 1 (d) 1/3

© 2014 www.examsguru.net | All Right Reserved Enquiries: call +234 (0) 802 652 9647 Page 10
Examsguru.Net – LASU POST UTME PAST QUESTIONS WITH SOLUTIONS 2005 - 2014
88. In fig. 8 below, O is the centre of the circle, AC=6cm 2011/2012 ENGLISH LANGUAGE
and BC=8cm. Find the circumference of the circle.
Read passage I and II, carefully and answer the
A question
B
O that follow.

C PASSAGE I
One of the most potent elements in body language is eye
(a) 10cm (b) 5cm (c) 47cm (d) 10cm Behaviour. You shift your eyes, meet another person’s
gaze or fail to meet it – and produce an effect out of all
89 In fig.9 below, O is the centre of the circle and proportion to the muscular effort you have made. When
<ACB=130o, Find <DOB two people look searchingly into each other’s eyes,
o o o o
(a) 100 (b) 130 (c) 80 (d) 25 emotions are heightened and the relationship tipped
D toward greater intimacy.
O
In normal conversation, each eye contact lasts only about
a second before one or both individuals look away.
A 130o
B
Because the longer meeting of the eyes is rare, it is
C weighted with significance when it happens and can
generate a special kind of human-to-human awareness.
90 Two ships leave the same port: one ship sails for Most of the time, a lingering look is interpreted as a sign
o
300km on a bearing of 340 ; the other sails for of attraction and this should be scrupulously avoided
o
400km on a bearing of 250 . The distance between except in appropriate
ships is circumstances. A young woman once complained, ‘that
(a) 700km (b) 100km (c) 500km (d) 200km man makes me so uncomfortable, half the time when I
91 A shopkeeper sold an item for N3,600, making a glance at him he’s already looking at me and he keeps
profit of 20%. Find the original cost of the item (a) right on looking.
N2,800 (b) N3,000 (c) 4,500 (d) 4,320
92 A flag pole of height 2.5m casts a shadow of length Proper street behaviour requires a balance of attention
4m. Calculate the angle of elevation of the sum, and intention. You are supposed to look at a passer-by
o o o
correct to the nearest degree (a) 32 (b) 58 (c) 39 just enough to show that you are aware of his presence.
o
(d) 51 If you look too little, you appear haughty or furtive; too
x+1 2x-1
93 If 4 x 8 = 16, find x. (a) 4 (b) 9 ½ (c) 5/9 (d) -1 much and you are inquisitive. Usually what happens is
94 Evaluate 22three x 102three, leaving your answer in that people eye each other until they are about eight feet
base 3. apart, at which point both east down their eyes.
(a) 88three (b) 1021three (c) 10021three (d) 2244three
95 8% of a certain sum of money is N320. What is 10% Much of eye behaviour is so subtle that we react to it only
of the sum? (a) N400 (b) N256 (c) N4,000 (d) 800 on the intuitive level. This has been demonstrated in
96 The number is selected at random from the set elaborate experiments. Subjects sit and talk in the
(3,0,5, 4, 5, 2/5). What is the probability the psychologist’s laboratory innocent of the fact that their
number is rational? eye behaviour is being observed from behind a one-way
(a) 2/5 (b) 3/5 (c) 1/5 (d) 4/5 vision screen. In one fairly typical experiment, subjects
2
97 The area of a circle is 154cm . Find its were included to cheat while performing a task, then
circumstance. (take =22/7) (a) 7cm (b) 14cm (c) were interviewed and observed. It was found that those
308cm (d) 44cm who had cheated met the interviewer eyes less often
98 Two dice are thrown together, what is the probability than was normal, an indication that ‘shifty eyes’ can
of getting a sum of 5 (a) 1/6 (b) 5/16 (c) 1/8 (d) 1/12 actually be a tip-off to an attempt to deceive.
99 in Fig.10 below, the acute angle of the
o
parallelogram is 45 , one side is 8cm and the area is However, none of the ‘facts’ of eye behaviour are cut and
2
24cm , find the other side. dried, for there are variations between individuals.
People use their eyes differently and spend different
o
45 amounts of time looking at others. Besides, pattern of
eye behaviour is precisely predictable in any normal
(a) 12cm (b) 10cm (c) 6cm (d) 4cm conversion.
100 three times the tens digit of a two digit number is 2
greater than the unit digit. When the digits are Adapted from McQuade (1969), Thinking in Writing,
interchanged the new number is 36 more than the p.167
original number. What is the original number? (a) 1 The young woman in the passage was uncomfortable
35 (b) 37 (c) 15 (d) 28 because (a) the man appeared to e showing interest
in her by his prolonged eye contact (b) the man’s short

© 2014 www.examsguru.net | All Right Reserved Enquiries: call +234 (0) 802 652 9647 Page 11
Examsguru.Net – LASU POST UTME PAST QUESTIONS WITH SOLUTIONS 2005 - 2014
and sharp gazes would tip the relationship toward breast-feeding is one of the basic cares which many
greater intimacy (c) strangers who made the man’s children of this age are 11 [A. tantalized with B. denied of
type of eye contact were likely to be dangerous (d) C. left with D. spared of]. This psychiatrist argues that
she was allergic to eye contact breast-feeding extends into the world outside the womb,
2. The reason given for the non-finality of research a liquid bond with the inside of the mother’s body; a bond
results on eye contact is (a) lack of concord and 12 [A. close to B. the same as C. unrelated to D.
foresight among psychologists who carry out research irrelevant to] that which the baby had with the placenta
on eye movement inside the uterus. Rhythmic rocking to and from is 13 [A.
(b) lack of uniformity in eye behaviour and the an elongation B. a demonstration C. a continuation D. a
variability of contact situation (c) that some subjects stretching] of the movement that the child experienced
cheat during experiments, thus invalidating research before it was born. As for the baby’s 14 [A. squeezing
findings (d) that research findings on eye contact have against B. separation from C. likeness for D. pressure
not been subjected to further empirical test against] its mother’s body, it reminds the child of the
…….15. [A. reassuring B. uncomfortable C. amusing D.
unpleasant] pressure of the uterus, and enables it to 16
3 From the expression proper street behaviour requires [A. unearth B. rediscover C. learn D. explore] the rhythms
a balance of attention and intention, it can be of its mother’s breathing and heartbeat.
conclude that
(a) even among strangers, attitudes and purposes In each of question 17 to 26, select the option that best
can be deduced from eye behaviour (b) before explains the information conveyed in the sentence.
walking the streets one must balance one’s shifty
eyes (c) when intimate groups meet in the streets, 17 If the rain hadn’t fallen, we wouldn’t have missed the
eye contact is usually longer (d) there is a balance match (a) the rain is falling, so we will miss the
between those with longer eye contact and those match (b) the rain fell, so we didn’t watch the match
with shorter eye contact (c) the fell, so we watched the watch (d) the rain
4 Which of the following can be concluded from the didn’t fall, so we didn’t watch the match
opening paragraph? (a) the effects of eye contact are 18 The new headmaster hoped that his men would pull
always overwhelming (b) eye behaviour is one of the together (a) he expected that the men would
non-verbal ways of communicating (c) eye movement cooperate with him (b) he thought that the men
is the most potent means of expressing intimacy (d) would compose themselves at work (c) he was
people are always offended by searching eye contact certain that they would resign on masse (d) he was
5 From the findings of the research described in the certain that their condition would improve under him
passage, one can reason that (a) innocent people 19 If the trader paid in full, his order was not pruned
maintain longer eye contact than the guilty ones (b) down (a) the trader who made full payment did not
guilty people make Jess frequent gazes during have his order reduced (b) the trader who made
interrogation (d) a feeling of guilt is occasioned by some payment did not have his order delayed (c)
‘shifty eyes’ unless the trader paid in full his order would be
rejected (d) as the trader did not increase his order,
PASSAGE II he did not need to pay in full
Use the passage below to answer question 6 to 16. The 20. The men were not pawns in someone else’s political
passage has gaps numbered 16 to 26. Immediately game (a) the action they executed was their idea (b)
following each gap, four options are provided. Choose the men used someone else’s plan (c) the were
the most appropriate option for each gap. used by someone’s political game (d) they loved
playing political games
Two thirds of children in …………..6 (A: industrialized B. 21 The crisis ended as suddenly as it began (a) the
socialized C. technological D. modernized} societies no crisis had suddenly begun (b) the crisis will not end
longer have family life. They are virtually abandoned to suddenly (c) the crisis stopped almost immediately
child minders from a very tender age. The………..7 [A. (d) the crisis will stop immediately
disregard B. indifference C. alienation D. inattention] 22 One thing I will not be complaining about in my new
from their mothers brings suffering and makes it job is a lack of excitement (a) the job is bad (b) the
impossible for them to achieve a healthy social life. The 8 job is exciting (c) the job is not too exciting (d) the
[A. development B. increase C. appreciation D. inflation] job has been previously done
in the number of suicides, the rates of drug addiction and 23 The governor parried all the questions put to him by
………….9 [A. delinquency B. irresponsibility C. the journalist (a) the governor answered all the
Satanism D. truancy] among young people may be to a questions brilliantly (b) the governor evaded all the
large extent, due to these premature separations which question (c) the governor failed all the questions (d)
take place before sufficient time has 10 [A. materialized the governor mastered all the questions
B. occurred C. surfaced D. elapsed] for attachment to 24 If I visited England, I might go to Manchester city (a)
develop. “this is one of the causes of psychosis in when I go to England, I could go to Manchester city
children today’ says a psychiatrist who believes that (b) whenever I visit England, I must go to

© 2014 www.examsguru.net | All Right Reserved Enquiries: call +234 (0) 802 652 9647 Page 12
Examsguru.Net – LASU POST UTME PAST QUESTIONS WITH SOLUTIONS 2005 - 2014
Manchester city (c) I did not go to England and 40 We made a pile in the business deal. (a) lost a lot of
could not go to Manchester city (d) I could not visit money (b) earned a lot of money (c) broke even (d)
Manchester city because I did not want to go to cut corners
England.
25 The man puts his foot down whenever he is 2011/2012 GENERAL PAPER
convinced of his action (a) he desires to assert his Answer the following questions by choosing one of the
will in the situation (b) he makes his mark wherever options
he goes (c) his attitude demonstrates someone who 1. A low land between two hills is called (a) island (b)
like to oppress others around him (d) he valley (c) plateau (d) lake (e) mountain
demonstrates firmness of character. 2. Which of these animals is not a member of dog
family
In each of questions 26 to 40, choose the option nearest (a) jackal (b) German Shephered (c) tiger (d)
in Yorkshire terrier (e) Rottweiler
meaning to the word or phrase in italic 3. The process in which the food we eat is broken
down into substance that can be used by the body
26 No wonder Dekemi later become a detective; she is called (a) digestion (b) circulation (c) tissue (d)
has been very observant (a) curious (b) perceptive respiration (e) excretion
(c) inductive (d) inquisitive 4 A frightening dream is called (a) blush (b) nightmare
27 Nigeria has been playing a vital role in the political (c) night anguish (d) night fear (e) none of the above
and economic development of Africa (a) creditable 5. Which of these is not a type of eagle? (a) short-toed
(b) crucial (c) respectable (d) laudable (b) booted (c) bonelli (d) all of the above (e) none of
28 Emeka’s painting was so realistic that it could the above
almost have been a photograph (a) picturesque (a) 6 What was the former name of present Zimbabwe?
concrete (c) lively (d) authentic (a) Rhodesia (b) Yamashoma (c) Orange republic
29 Courteously, Ade stood back to let his teacher go (d) Brazzaville
first through the door (a) patiently (b) politely (c) 7. A system of government based on the ideological
carefully (c) calmly belief of people concentration of national resources
30 Many people used to live in property (a) instability in the hands of the state is called (a) fanaticism (b)
(b) want (c) difficulty (d) the slums egalitarianism (c) communism (d) democracy (e)
31. Accountability is certainly a desirable quality in a utopianism
politician (a) respectability (b) courage (c) 8. An instrument used in finding ways and navigation
responsibility (d) diligence is called (a) map (b) GPRS (c) compass (d)
32 The common practice among some media compound (e) none of the above
practitioners is to be sensational in their reporting 9 A computer is made of (a) hardware (b) software (c)
(a) prevalent (b) rampant (c) ordinary (d) cogent none of the above (d) A & B (e) A only
33 That fateful decision changed the company’s 10. Which of these continents is the coldest in the
outlook in many ways (a) wonderful (b) disastrous world? (a) Asia (b) Africa (c) Antarctic (d) Europe (e)
(c) an internal (d) an external America
34 The accident victim received a superficial wound 11 What is the name of the world’s highest mountain?
from the crash (a) a serious (b) a painless (c) an (a) Mount Kilomajaro (b) Mount Everest (c) Mount
internal (d) an external Cameroon (d) Mountain Nkoyo (e) none of the
35 the boxer fizzled out just in the sixth round (a) above
knocked out his opponent (b) showed off his talents 12 What name is the parliament of Nigeria called (a)
(c) became tired but fought on spiritedly (d) house of representative (b) senate (c) state house
surrendered rather disappointingly of assembly (e) national assembly (e) all of the
36 The manager described Mfon as a man of above
vehement character (a) weak and uninterested (b) 13 The parliament of the United States of America is
strong and insistent (c) troublesome and noisy (d) called (a) house of parliament (b) national assembly
clever and helpful (c) congress (d) assembly of law makers (e) none of
37 Obi’s dog is old but still lively (a) attractive (b) the above
howling (c) barking (d) frisky 14 Cote d’ Ivoire is formerly known as? (a)
38. His taciturnity amazed everyone in the court during Yamassokou (b) Ivory Coast (c) Gold Coas (d)
the legal tussle (a) obliviousness (b) reticence (c) Rhode Coast (e) Diamond Haven
sensibility (d) pervasiveness 15 The process of preservation, protection and wise
39 The greatness of the creator of the universe is use of natural resources is called (a) fermentation
always inexpressible to many adherents of certain (b) preservation (c) conservation (d) ecology (e)
faiths (a) inevitable (b) wonderful (c) ineffable (c) zoning
unbearable 16 Ballet and Tango are types of (a) song (b) dancing
(c) cycling (d) swimming (e) horse ridding

© 2014 www.examsguru.net | All Right Reserved Enquiries: call +234 (0) 802 652 9647 Page 13
Examsguru.Net – LASU POST UTME PAST QUESTIONS WITH SOLUTIONS 2005 - 2014
17 The first person to develop atomic bomb was (a) 2012/2013 MATHEMATICS PRACTICE
Albert Einstein (b( Charles De Gaulle (c) Thomas 1. Calculate 3310, 1442 (a) 1313 (b) 2131 (c) 4302 (d)
Jefferson (d) T.S. Elliot (e) Plato 1103
18 The process by which the people in a country is 2. Convert 3.1415926 to 5 decimal places (a) 3.14160
given an opportunity to elect, choose or reject new (b) 3.14159 (c) 0.31415 (d) 14200
government is known as (a) referendum (b) 3 The length of a notebook 15cm was measured as
plebiscite (c) election (d) ratification (e) none of the 16.8cm. calculate the percentage error to 2
above significant figures (a) 12.00% (b) 11.00% (c) 10.71%
19. In which continent is Mount Everest? (a) Asia (b) (d) 0.12%
North America (c) South America (d) Africa (e) 4 A worker’s present salary is N24.000 per annum.
Europe His annual increment is 10% of his basic salary.
20. Which of these people is not an explorer? (a) Vasco What would be his annual salary at the beginning of
Da Gama (b) Christopher Columbus (c) Ferdinand the third year?
Magellan (d) David Livingstone (e) none of the (a) N28.800 (b) N29.040 (c) N31.200 (d) N31.944
above 5 Express the product of 0.0014 and 0.011 in
4 -3
21. What is the difference between 2:45am and standard form (a) 1.54 x 10 (b) 1.54 x 10 (c) 1.54
-4 -5
12:32pm (a) 9Hrs 47min (b) 14hrs 10min (c) 5hra x 10 (d) 1.54 x 10
1 3
25min (d) 10hrs 17min 6 Evaluate (81)¾ - (27) /3 /3 x 2
22 2¾ - 2½ x ½ = (a) 4/3 (b) 14/3 (c) 9/8 (d) 3/2 (a) 27 (b) 1 (c) 1/3 (d) 1
3
23 2 scores plus 4 dozens multiplied by 14 equals? (a) 7 Find the value of (16) /2 + log10 0.0001 + log2 32
118 (b) 1232 (c) 1882 (d) 1432 (a) 0.065 (b) 0.650 (c) 5.500 (d) 65.000
24 33½ of 100 equal? (a) 33½ (b) 30 (c) 3 (d0 33 8 Simplify 12 -  3
25 1800 multiplied by what number will give you 12 - 3
100800 (a) 56 (b) 28 (c) 41 (d) 38 (a) 1/3 (b) 0 (c) 2 (d) 1
26 5.8 x 6.1 x 9.8 = (a) 3324 (b) 1591 (c) 2831 (d) 9. Four members of a social first eleven cricket term
2419 are also members of the first fourteen rugby team.
27 A farmer has 41 bags of oranges. Each bag How many boys play for at least one of the two
contains 59 oranges each. How many oranges does teams?
the farmer have? (a) 25 (b) 21 (c) 16 (d) 3
(a) 3324 (b) 1591 (c) 2831 (d) 2419 10. If S = (xx2 = 9, x>4), then S is equal to (a) 0 (b) (0)
28 If 16 of the same book weight 4kg. how much does (c) (?) (d) (%)
3
one book weigh? (a) 20g (b) 25g (c) 55g (d) 40g 11 If x 1 and x = 1 are both factors of the equator x +
2
29 What is 5% of N575? (a) 30.8 (b) 28.75 (c) 25.5 (d) px + qx + 6 = 0, evaluate p and q (a) 6,-1 (b) 6,-1
55.5 (c) 1, -1 (d) 6,-6
2
30 What is the difference between 500 multiplied by 12 Find a positive value of p is the expression 2x -px +
700 and 700 multiplied by 500? (a) 1000 (b) 100 (c) p leaves a remainder 6 when divided by x – p.
0 (d) 10000 (a) 1 (b) 2 (c) 3 (d) 4
31 A girl has 98 beads and all but 14 were lost. How 13 Find T in terms of K, Q and S if S = 2r (QT + K)
many beads did she loose? 2
(a) S - K (b) S2 - K
32 If 15% of a number is 175. What is the number 2r  Q
2
Q 4r  Q
2
Q
multiplied by 2? (a) 500 (b) 150 (c) 1000 (d) 800
th
33 A man was born on the 29 of February, 1980. How 2
(c) S - K (d) S
2
- K
many birthdays has he celebrated after his birth till 2r  Q
2
4r  Q
2

today? (a) 9 (b) 9 (c) 3 (d) 13 14 The graph of f(x)2 = x2 5x + 6 crosses the x-axis
35 17,31,51,68 (a) 75 (b) 82 (c) 90 (d) 85 (a) (-6,0), (-1,0) (b) -3, 0) (c) (-6,0), (1,0) (d) (2,0),
36 A man buys 6 books and 3 bags. If a book cost N17 (3,0)
and a bag cost N25. How much as spent? (a) N112 15
2
Factorize completely the expression abx + 6y 3ax
(b) N177 (c) N125 (d) N150 2byx
37 It takes 15 minutes to fill 125 gallons with petrol (a) (ax – 2y) (bx 3) (b) bx + 3) (2y – ax) (c) (bx + 3)
from a tanker. How long will it takes to fill gallons? (ax – 2y) (d) ax – 2y (ax – b)
(a) 92min (b) 45min (c) 87 min (d) 102min 16 Solve the following inequality (x-3) (x-4) < 0
38 If it takes 15 men 6½ days to build a house, how (a) 3 < x < 4 (b) 3< x <4 (c) 3<z<4 (d) 3<x< 4
many houses can they build in 45 days? (a) 3 days 17
th
The 4 term of an A.P is 13 while the 10 term is
th

(b) 7days (c) 8 days (d) 5days st


31. Find the 21 term, (a) 175 (b) 85 (c) 64 (d) 45
39 If it takes a boy 5 minutes to run 1 km, how long 18 Simplify
2
x –1
would it take him to run 2½ km? (a) 10½ min (b) 2
X3 – 2x – x – 2
15min (c) 12½ (d) 11½min (a) 1 (b) x – 1 (c) x – 1 (d) 1
40 How many bottles are in a dozen crates containing X+2 x+1 x+2 x–2
24 bottles each? 19 Express 5x 12/(x-2) (x-3) in partial fractions
(a) 2 - 3 (b) 2 + 3

© 2014 www.examsguru.net | All Right Reserved Enquiries: call +234 (0) 802 652 9647 Page 14
Examsguru.Net – LASU POST UTME PAST QUESTIONS WITH SOLUTIONS 2005 - 2014
X -2 x-3 x-2 x-3 36
(c) 2 - 3 (d) 5 - 4 Age in 37. 38. 39. 40. 41.
x-3 x-2 x-3 x-2
20 Which of the following binary operations is years 13 14 15 16 17
commutative in the set of integers? (a) a-b=a-2b (b)
No of 42. 3 43.10 44. 45. 46.15
a.b=a+b-ab
2
(c) a-b=a =b (d) a.b=a(b+1)/2 students 30 42
21 If a*b= + ab evaluate 2* (12*27)
(a) 12 (b) 9 (c) 6 (d) 2
22 Find the sum to infinity of the following sequence The frequency distribution above shows the ages of
1,9/10, (9/10)2, (9/10)3 students in a secondary school. In a pie chart
(a) 1/10 (b) 9/10 (c) 10/9 (d) 10 constructed to represent data, the angle
o o
23 Find the value of k if corresponding to the 15 years old is (a) 20 (b) 30
o o
-2 1 1 (c) 54 (d) 108
2 1 k = 23
1 3 1 French, 90

(a) 1 (b) 2 (c) (d) 4


Economic, 150
24 If X = 1 2 and Y = 2 1 find XY C.R.K, 30
0 3 4 3
History, 90
(a) 10 7 (b) 2 7 (c) 10 4 (d) 4 3 37
12 9 4 17 4 6 10 9
25
o
In a triangle XYZ, <YXZ = 44 and <XYZ = 112 ,
o The pie chart above shows the distribution of
calculate the acute angle between internal bisectors students in a secondary school class. If 30 students
o o
of <XYZ and <XZY (a) 12 (b) 56 (c) 68 (d) 78
o o offered French, how many offered C.R.K? (a) 25 (b)
26 Find the distance between two town P(45 N 30 W)
o o 15 (c) 10 (d) 8
o o
and Q(15 S, 3 W) if the radius of the earth is
38 The mean and the range of the set of number 1, 20,
7000km (?=22/7)
1.00, 0.90, 1.40, 0.80, 1.20 and 1.10 are in and r
(a) 1100/3km (b) 2200/3km (c) 22000/3km (d)
respectively. Find m + r (a) 1.11 (b) 1.65 (c) 1.85
11000/3km
(d) 2.45
27 Two perpendicular lines PQ and QR intersect at (1,
-1), if the equation of PQ is x-2y + 4 = 0, find the Class 1-3 4-6 7-9
equation of QR
(a) x-2+1=0 (b) 2x+y-3=0 (c) x-2y-3=0 (d) 2x + y- Frequency 5 8 5
1=0
28 P is on the locus of points equidistant from two 39 Find the standard deviation of the data using the
table above. (a) 5 (b) 6 (c) 5/3 (d) 5
given points X and Y, UV is a straight line through Y
o 40 The variance of the scores 1, 2, 3, 4, 5 is
parallel to the locus, if < PYU is 40 , find <XYP. (a) 1.2 (b) 1.4 (c) 2.0 (d) 3.0
o o o o
(a) 100 (b) 80 (c) 50 (d) 40
29 The base diameter of a cylinder is 14cm while the
height is 12cm, calculate the total surface area if the ENGLISH 2012/2013
2 1. From the following options select the one opposite in
cylinder has both a base and a top (a) 836cm (b)
2 2
528cm (c) 308cm (d) 154cm
2 meaning to the underlined word or group of words in
a sentence.
30 A school boy lying on the ground 30, away from the
foot of a water tank tower observes that the angle of The young engineer is good at terminating other
o
elevator of a top of the tank is 60 , calculate the people’s projects but has not been capable of
height of the water tank. (a) 18.2m (b) 12.2m (c) ............any of his own. (a) integrating (b) finishing (c)
6.2m (d) 3.2m completing (d) initiating (e) organizing
o
31 QRS is a triangle with QS=12m, <RQS=30 and
o
<QRS=45 (a) 18.2m (b) 12.2m (c) 6.2m (d) 3.2m 2. From the following options select the one opposite in
meaning to the underlined word or group of words in
32 The derivative of cosec x is (a) tan x cosec x (b) cot
sentence.
x cosec x (c) tan x sec x (d) cot x sec
2
33 For what value of x is the tangent to the curve y=x - The manager who expected to be given respect was
4x+3 parallel to the axis? (a) 3 (b) 2 (c) 1 (d) 0 treated with......... (a) dignity (b) scorn (c) shame (d)
34 Two variables x and y are such that dy/dx=4x-3 and cruelty (e) disloyalty
y=5 when x=2, find y in terms of x.
2 2 2
(a) 2x -3x+5 (b) 2x -3x+3 (c) 2x -3x (d) 4 3. From the following options select the one opposite in
2 meaning to the underlined word or group of words in
35 Find the area bounded by the curve y=3x -2x+1, the sentence.
ordinates x=1 and x=3 and the x-axis
(a) 24 (b) 22 (c) 21 (d) 20

© 2014 www.examsguru.net | All Right Reserved Enquiries: call +234 (0) 802 652 9647 Page 15
Examsguru.Net – LASU POST UTME PAST QUESTIONS WITH SOLUTIONS 2005 - 2014
Those who had invitation cards were admitted to the The play was so interesting that the ..............clapped
party while those who had none were ........... for quite a long time at the end (a) spectators (b)
(a) Barred (b) repelled (c) expelled (d) compelled watchers (c) congregation (d) people (e) audience
(e) restricted
12 From the options choose the word that best
4. From the following options select the one opposite in completes each of the following sentences.
meaning to the underlined word or group of words in
sentence. The building.................because of weak structural
foundation (a) tumbled (b) succumbed (c) collapsed
Too many theories will not help us: we need to (d) caved (e) crumbled
be.........
(a) Bookish (b) hypothetical (c) antithetical (d) 13 From the options choose the word that best
shrewd completes each of the following sentences.
(e) practical
The magazine was ..............by the government for an
5 From the following options select the one opposite in offensive publication (a) prescribed (b) proscribed (c)
meaning to the underlined word or group of words in suspended (d) condemned (e) persecuted
sentence.
14 From the options choose the word that best
Nobody expects him to show ..........for his children completes each of the following sentences.
but he certainly bestows too much affection on them
(a) love (b) concern (c) intimacy (d) devotion (e) Many people reacted to the brutal murder of the
hatred popular journalist with strong...............(a) indignation
(b) demonstration (c) mobilization (d) condemnation
6 From the following options select the one opposite in (e) accusation
meaning to the underlined word or group of words in
sentence. 15 From the options choose the word that best
completes each of the following sentences.
The challenger was crude and inexperienced
incorrect to the champion who was .............(a) great The governor frowned at the ...............which slowed
(b) exposed (c) celebrate (d) refined (e) strong down the implementation of policy decisions (a)
bureaucracy (b) autocracy (c) opposition (d)
7 From the following options select the one opposite in convention
meaning to the underlined word or group of words in
sentence. 16 From the options choose the word that best
completes each of the following sentences.
What should have been a source of motivation for
him proved a terrible source of .............(a) failure (b) The doctor ............the illness as yellow fever not
harm (c) discourage (d) uncertainty (e) disturbance malaria (a) prescribed (b) examined (c) discovered
(d) announced (e) diagnosed
8 From the following options select the one opposite in
meaning to the underlined word or group of words in 17 From the options choose the word that best
sentence. completes each of the following sentences.

Ade is very conservative in his political ideals Since the writer did not indicate his name, the editor
whereas Bala expresses.................views always (a) decided not to publish such ...........article (a) a
radical (b) heretical (c) conventional (d) fanatical (e) discourteous (b) an anonymous (c) a scandalous (d)
impractical a libellous (e) a cowardly

9 From the following options select the one opposite in 18 After the following sentence, a list of possible
meaning to the underlined word or group of words in interpretation of all or part of the sentence is given.
sentence. Choose the interpretation that you consider most
appropriate for each sentence.
The government has been spending more money on
preventive rather than on .............medicine (a) Mrs. Alabi was not around when her case was called
modern (b) protective (c) diagnostic (d) traditional (e) and none of her colleagues was willing to hold brief
curative for her. This means that: (a) her colleagues were
afraid (b) she was not trusted (c) her colleagues did
not want to represent her (d) she had quarrel with her
10 From the options choose the word that best colleagues.
completes each of the following sentences.
19 After the following sentence, a list of possible
The fisherman threw a stone into the river and this interpretation of all or part of the sentence is given.
caused a ..................(a) sprinkle (b) sparkle (c) Choose the interpretation that you consider most
splash (d) spring (e) storm appropriate for each sentence.

11 From the options choose the word that best


completes each of the following sentences.

© 2014 www.examsguru.net | All Right Reserved Enquiries: call +234 (0) 802 652 9647 Page 16
Examsguru.Net – LASU POST UTME PAST QUESTIONS WITH SOLUTIONS 2005 - 2014
Telling lies has become second nature to Kunle. Choose the interpretation that you consider most
This means that Kunle (a) rarely tells lies (b) always appropriate for each sentence.
tells lies (c) tells lies only on second thoughts (d)
learnt to tell lies from childhood Okon is quite hard working but his problem is that he
has too many irons in the fire. This means that Okon
20 After the following sentence, a list of possible (a) is a blacksmith (b) takes on more problems that
interpretation of all or part of the sentence is given. he can cope with (d) is a very successful
Choose the interpretation that you consider most businessman
appropriate for each sentence.

For us to accomplish the task successfully, all hand


must be on check. This means that everybody. 27 From the words or group of words below the
(a) Should push with his hands (b) will have travel by following sentence, choose the word or group of
sea words that is nearest in meaning to the underlined
(c) must cooperate (d) should take a rest before group of words as used in the sentence.
continuing the task
It takes a great deal of stamina to run the marathon
21 After the following sentence, a list of possible race (a) courage (b) determination (c) Energy (d)
interpretation of all or part of the sentence is given. intelligence (e) elevenses
Choose the interpretation that you consider most
appropriate for each sentence. 28 From the words or group of words below the
following sentence, choose the word or group of
The wedding was attended by the cream of the words that is nearest in meaning to the underlined
society. This means that the wedding was attended group of words as used in the sentence.
by
(a) Everybody in the society (b) only those who were But for the principal actor the play would have been
invited (c) very important people (d) only the dull
educated people (a) important (b) head (c) master (d) famous (e) main

22 After the following sentence, a list of possible 29 From the words or group of words below the
interpretation of all or part of the sentence is given. following sentence, choose the word or group of
Choose the interpretation that you consider most words that is nearest in meaning to the underlined
appropriate for each sentence. group of words as used in the sentence.

I want you to keep an eye on that child. This means An open car has no protection against the element
that i want you to: (a) disciplined the child (b) keep (a) weather (b) emergency (c) molecules (d) atoms
the child busy all the time (c) punish the child if he (e) atmosphere
misbehave (d) watch the child all the time
30 From the words or group of words below the
23 After the following sentence, a list of possible following sentence, choose the word or group of
interpretation of all or part of the sentence is given. words that is nearest in meaning to the underlined
Choose the interpretation that you consider most group of words as used in the sentence.
appropriate for each sentence.
It appears Sade’s success at the last examination He was reluctant to grant my request (a) disposed (b)
has gone to her head. This means that Sade is (a) delighted (c) reticent (d) unwilling (e) agreeable
conceited (b) too happy (c) crazt (d) impudent
31 From the words or group of words below the
24 After the following sentence, a list of possible following sentence, choose the word or group of
interpretation of all or part of the sentence is given. words that is nearest in meaning to the underlined
Choose the interpretation that you consider most group of words as used in the sentence.
appropriate for each sentence.
The detective was perplexed when clues in the
Little Sheila is always as lively as a kitten. This murder case pointed to at least a dozen different
means that Sheila is always (a) wearing bright suspects (a) surprised (b) confused (c) excited (d)
dresses (b) very realistic in what she does (c) moving discouraged (e) disappointed
quickly like a kitten (d) Gay and cheerful
32 From the words or group of words below the
25 After the following sentence, a list of possible following sentence, choose the word or group of
interpretation of all or part of the sentence is given. words that is nearest in meaning to the underlined
Choose the interpretation that you consider most group of words as used in the sentence.
appropriate for each sentence.
The military Governor called for a concerted effort in
This masquerade appears once in a blue moon. This solving the problems of the state (a) a dramatic (b)
means that the masquerade appears (a) on very rare an agitated (c) a join (d) a directed (e) an unfailing
occasions (b) when the moon is blue (c) whenever a
special request is made (d) once a month 33 From the words or group of words below the
following sentence, choose the word or group of
26 After the following sentence, a list of possible words that is nearest in meaning to the underlined
interpretation of all or part of the sentence is given. group of words as used in the sentence.

© 2014 www.examsguru.net | All Right Reserved Enquiries: call +234 (0) 802 652 9647 Page 17
Examsguru.Net – LASU POST UTME PAST QUESTIONS WITH SOLUTIONS 2005 - 2014
aggressor (c) took the provocation with calmness (d)
My financial situation is so precarious very soon i was found guilty of committing the offence
may be insolvent (a) borrowing (b) stealing (c)
soluble (d) dependent (e) bankrupt 40 After the following sentence, a list of possible
interpretations of the sentence is given. Choose the
34 From the words or group of words below the interpretation that you consider most appropriate for
following sentence, choose the word or group of each sentence.
words that is nearest in meaning to the underlined
group of words as used in the sentence. The members of staff told the principal that they took
The Chairman is of the opinion that accepting the exception to his comments. This means that the
proposal would be inimical to the objectives of the members of staff. (a) approved of the principal’s
association (a) harmful (b) relevant (c) irrelevant (d) comments (b) objected to the principals comments
indispensable (e) helpful (c) wanted to be exempted from the principal
comments (d) warned the principal about his
35 From the words or group of words below the comments
following sentence, choose the word or group of
words that is nearest in meaning to the underlined 41 After the following sentence, a list of possible
group of words as used in the sentence. interpretations of the sentence is given. Choose the
interpretation that you consider most appropriate for
each sentence.
The famous politician was noted for his programmatic
approach to issues of national interest (a) idealistic Okon is too quite for my liking. This means that ekon
(b) romantic (c) compromising (d) practical (e) (a) is like because he is very quite (b) likes people
inconsistent who are quite (c) is not like because of his extreme
quietness (d) behaves in a very queer and suspicious
36 From the words or group of words below the manner
following sentence, choose the word or group of
words that is nearest in meaning to the underlined 42 After the following sentence, a list of possible
group of words as used in the sentence. interpretations of the sentence is given. Choose the
interpretation that you consider most appropriate for
Kunle is very pessimistic about our chance of each sentence.
success (a) sad (b) despondent (c) unconvinced (d)
worried (e) concerned

37 After the following sentence, a list of possible The teacher said all but one of the sixty candidates
interpretations of the sentence is given. Choose the scored below a credit grade. This means that (a) all
interpretation that you consider most appropriate for the candidate scored below a credit grade (b) only
each sentence. one of the candidates scored a credit grade (c) only
one of the candidates scored below a credit grade (d)
If Kunle hadn’t been transferred to Kano his wife all the candidates scored a credit grade
would not have resigned her teaching job, this means
that 43 From the words or groups of words below, choose
(a) Kunle was not transferred yet his wife resigned the words or group of words that best complete each
her job of the following sentences.
(b) Kunle was transferred and his wife thought of
resigning her job (c) Kunle was transferred yet his He was charged with complicity............. the abortive
wife had to resign her job (d) Kunle’s wife resigned coup (a) in (b) for (c) about (d) on
though her husband transferred.
44 From the words or groups of words below, choose
38 After the following sentence, a list of possible the words or group of words that best complete each
interpretations of the sentence is given. Choose the of the following sentences.
interpretation that you consider most appropriate for
each sentence. He was convicted ..................stealing some bags of
cocoa (a) with (b) in (c) of (d) for
I’ll call at your house this evening if it doesn’t train.
This means that i (a) will visit you this evening if it 45 From the words or groups of words below, choose
rains (b) will only phone if it rains (c) will sleep in your the words or group of words that best complete each
house this evening if it rains (d) may not visit you this of the following sentences.
evening if it rains
Mary said she was acting.................the instructions
39 After the following sentence, a list of possible of the principal (a) by (b) on (c) for (d) through
interpretations of the sentence is given. Choose the
interpretation that you consider most appropriate for 46 From the words or groups of words below, choose
each sentence. the words or group of words that best complete each
of the following sentences.
Though Ibrahim was provoked, he ought to have
controlled himself. This means that Ibrahim (a) ...................normal circumstances, it is rare to get all
should have exercise some restraint (b) was the registered voters to vote (a) on (b) by (c) in (d) under

© 2014 www.examsguru.net | All Right Reserved Enquiries: call +234 (0) 802 652 9647 Page 18
Examsguru.Net – LASU POST UTME PAST QUESTIONS WITH SOLUTIONS 2005 - 2014
47 From the words or groups of words below, choose 55 From the words or groups of words below, choose
the words or group of words that best complete each the
of the following sentences. words or group of words that best complete each of
the following sentences.
Lekan has tried to live................ to his parents’
expectations (a) over (b) on (c) through (d) up ……………..of what he said made no sense
(a) much (b) majority (c) plenty (d) many
48 From the words or groups of words below, choose
the words or group of words that best complete each 56 From the words or groups of words below, choose
of the following sentences. the
words or group of words that best complete each of
The minister’s daughter boasted she would have her the following sentences.
way and she really................. (a) had (b) has (c)
have (d) did That isn’t your book, it is ……………(a) Johns’ (b)
John’s (c) John own (d) for John’s
49 From the words or groups of words below, choose
the 57 From the words or groups of words below, choose
words or group of words that best complete each of the
the following sentences. words or group of words that best complete each of
the following sentences.
As at yesterday only nomination papers in respect of
three candidates…………(a) have been filled (b) are Nana Sawara is a queen and demands to be
being filled (c) been filed (d) has been filled treated………….(a) like that (b) like so (c) as such (d)
that much
50 From the words or groups of words below, choose
the 58 From the words or groups of words below, choose
words or group of words that best complete each of the
the following sentences. words or group of words that best complete each of
the following sentences.
They embraced passionately as if they………seen
each other for years (a) had (b) had not (c) have (d) It is pretty difficult to choose…………….those three
have not boys (a) between (b) over (c) against (d) among

51 From the words or groups of words below, choose 59 From the words or groups of words below, choose
the the
words or group of words that best complete each of words or group of words that best complete each of
the following sentences. the following sentences.
…………… of the five boys was able to show me the
You are not sure where you place the way to the zoo. (a) none (b) neither (c) any (d) some
book…………….?
(a) isn’t It (b) weren’t you (c) are you (d) didn’t you 60 From the words or groups of words below, choose
the
52 From the words or groups of words below, choose words or group of words that best complete each of
the the following sentences.
words or group of words that best complete each of
the following sentences. We will leave ……….you are ready. (a) as whenever
(c) while (d) until

61 From the words or groups of words below, choose


We arrived when the hall ……….arranged, so we the
stood outside (a) has been (b) is being (c) was being words or group of words that best complete each of
(d) had been the following sentences.

53 From the words or groups of words below, choose This …………….be David’s handwriting. I know his
the handwriting well enough (a) may (b) will (c) ought (d)
words or group of words that best complete each of can’t
the following sentences.
62 From the words or groups of words below, choose
She doesn’t’ know you like your food hot…………..? the
(a) does she (b) shouldn’t she (c) didn’t she (d) words or group of words that best complete each of
wasn’t it the following sentences.

54 From the words or groups of words below, choose The book is not in his locker and it’s not in that
the words or group of words that best complete each of cupboard……..(a) neither (b) again (c) either (d) even
the following sentences.
63 From the words or groups of words below, choose
It’s certain we shall travel by train…………………? the
(a) shan’t we (b) shall we (c) don’t we (d) isn’t it words or group of words that best complete each of
the following sentences.

© 2014 www.examsguru.net | All Right Reserved Enquiries: call +234 (0) 802 652 9647 Page 19
Examsguru.Net – LASU POST UTME PAST QUESTIONS WITH SOLUTIONS 2005 - 2014
words or group of words that best complete each of
………..things she had in the room were thrown out the following sentences.
(a) so few (b) the few (c) all few (d) very few
He has not been seen by his parents………. Seven
64 From the words or groups of words below, choose days
the (a) since (b) for (c) getting to (d) for last
words or group of words that best complete each of
the following sentences. 72 From the words or groups of words below, choose
the
The party hardly started….the lights went off. (a) than words or group of words that best complete each of
(b) then (c) as (d) when the following sentences.

65 From the words or groups of words below, choose He has not been seen by his parents…………….last
the month days (a) since (b) for (c) getting to (d) since
words or group of words that best complete each of over
the following sentences.
73 From the words or groups of words below, choose
There wee live five boys…………. the
(a) of which two were beaten (b) whom two were words or group of words that best complete each of
beaten the following sentences.
(c) two of whom were beaten (d) of whom of them
were beaten

66 From the words or groups of words below, choose


the Please, let me ……..you in your car to the railway
words or group of words that best complete each of stations
the following sentences. (a) go with (b) move with (c) follow (d) ride

This is the man………….told me the story (a) whom I 74 From the words or groups of words below, choose
said (b) who I said (c) I said that he (d) who I said he the
words or group of words that best complete each of
67 From the words or groups of words below, choose the following sentences.
the
words or group of words that best complete each of The ………….came here last wee (a) handsome tall
the following sentences. young man (b) young tall handsome man (c) tall
handsome young man (d) young handsome tall man
I feel you are none………for experiencing that shock
(a) the worse (b) worse (c) the worst (d) worst 75 From the words or groups of words below, choose
the
68 From the words or groups of words below, choose words or group of words that best complete each of
the the following sentences.
words or group of words that best complete each of
the following sentences. The activities marking the golden jubilee celebrations
of the dub were………..with a party (a) rounded off
Many students find Mathematics ………than English (b) rounded oven (c) rounded through (d) rounded up
(a) difficult (b) too difficult (c) very difficult (d) more
difficult 2012/2013 GENERAL PAPER
Answer the following questions by choosing one of the
69 From the words or groups of words below, choose options
the
words or group of words that best complete each of 1 The ‘motto’ of the LAGOS STATE UNIVERSITY is
the following sentences. (a) learning & morals (b) in truth we stand (c) in deed
and in truth (d) dignity and labour (e) none of the
Tunde is the ……..qualified person for that job. (a) above
more 2 Which of the following is not the name of the
(b) most (c) more than (d) many more incumbent President of Nigeria? (a) Goodluck (b)
Jonathan (c) Azikiwe (d) Ebele (e) Nwerem
3 the name of an organisation that provides customers
70 From the words or groups of words below, choose with access to the internet is (a) Cyber café (b)
the internet service provide (c) Microsoft service provider
words or group of words that best complete each of (d) HTML editor
the following sentences. 4. Nigeria became a Republic in (a) 1960 (b) 1966 (c)
1963 (d) 1959
It is such a bad place …………. I will never dream of 5 What is the name of the current Vice-Chancellor of
going there another time (a) therefore (b thus (c) that the LAGOS STATE UNIVERSITY ? (a) Prof. Tolu
(d) then odugbemi (b) Prof. Babafemi Sofoluwe (c) Prof. Oye
Ibidapo-Obe (d) Prof. Adetokunbo Sofoluwe (e) Prof.
71 From the words or groups of words below, choose Nurudeen Aho
the

© 2014 www.examsguru.net | All Right Reserved Enquiries: call +234 (0) 802 652 9647 Page 20
Examsguru.Net – LASU POST UTME PAST QUESTIONS WITH SOLUTIONS 2005 - 2014
6 The 2011 Presidential election in Nigeria was held on 24 Who invented the telephone? (a) Isaac Newton (b)
(a) April 6, 2011 (b) April 9, 2011, (c) April 16, 2011 Graham Douglas (c) Isaac Holloway (d) None of the
(d) April 26, 2011 (e) none of the above above
7 Which of the following was not a candidate in the 25 Electric bulb was invented by (a) John Bulb (b)
2011 presidential election in Nigeria? (a) Nuhu Thomas Watts (c) Mary Bulb (d) None of the above
Ribadu (b) Mohammed Buhari (c) none of the above 26 LAGOS STATE UNIVERSITY is situated in (a) Yaba
8 The chairman of the economic and financial crimes Local Government (b) Lagos Mainland Local
commission in Nigeria is (a) mallam nuhu ribadu (b) Government (c) Bariga Local Government (d) Lagos
Mrs. Farida Waziri (c) Mrs. Dora Akunyilu (d) Alh. Island Local Government
Mohammed Buhari (e) none of the above 27 The General Overseer of the Redeemed Christian
9 Lagos is a place in (a) Nigeria only (b) Portugal only Church of God in Nigeria is (a) Pastor Adebayo
(c) Spain only (d) Nigeria & Portugal (e) Nigeria & Kumuyi (b) Pastor Enoch Adeboye (c) Pastor Wale
Spain Odukoya (d) Pastor Benson Idahosa (e) none of the
10 The name of the wife of president Barack Obama of above
America is (a) Michelle Obama (b) Roseline Obama 28 Which of the following is not a Pentecostal church?
(c) Hilary Obama (d) Palin Obama (e) none of the (a) Redeemed Christian Church of God (b) Deeper
above Life Ministry (c) Foursquare Gospel Church (d)
11 President Barack Obama of the United State of Mountain of Fire and Miracle (e) The Catholic Church
America is 29 The clock called Big-B en can be found in (a) Los-
th th
(a) the 44 president of America (b) the 47 Angeles (b) Vancouver (c) London (d) Paris (e)
th
president of America (c) the 50 president of America Toronto
th
(d) the 99 president of America (e) the 101th 30 Which of the following is not in the national anthem of
president of America Nigeria? (a) one nation bound in freedom (b) arise o
12 Goals scored in an off-side position in a football compatriot (c) to serve our fatherland (d) with love
match attracts (a) a penalty from the referee (b) and strength and hope (e) none of the above
penalty by the award of one goal to the opponent (c) 31 Which of the following African leaders is not a Nobel
nullification (d) addition of one goal to the total goals Prize winner? (a) Archibong Desmond Tutu (b) Mr.
scored by the scorer’s side (e) none of the above Nelson Mandella (c) Prof. Wole Soyinka (d) Prof.
13 Mammar Gaddafi was the president of (a) Syria (b) Chinua Achebe (e) none of the above
Libya (c) Egypt (d) Malaysia (e) Yemen 32 In human respiration, oxygen is inhaled, which of the
14 9/11 refers to (a) September 9th, 2001 series of following is exhaled? (a) carbon-monoxide (b)
coordinated suicide attack on Japan (b) September oxygen (c) carbon-dioxide (e) sodium chloride (e)
9th, 2001 series of coordinated suicide attack on the none of the above
United States of America (c) September 9th, 2011 33 The smallest nation in the world is (a) Vatican city (b)
series of coordinated suicide attack on the United Malaysia (c) Madagascar (d) Zimbabwe (e) Italy
States of America (d) September 11th, 2009 series of 34 Who won the 2011 gubernatorial election in Lagos
coordinated suicide attack on the United States of State? (a) Mr. Babatunde Raji Fashola (b) Alhaji Bola
America (e) September 11th, 2001 series of Ahmed Tinubu (c) Dr. Ade Dosumu (d) chief J. K.
coordinated suicide attack on the United States of Randle
America (e) Mrs. Oluremi Tinubu
15 The total number of votes scored by the winner of the 35 The meaning of CNN, the American television station
2011 presidential election in Nigeria was (a) is (a) central national news (b) central news network
30,379,159 (b) 22,495,187 (c) 12,214,853 (d) (c) cable national news (d) central network news
15,379,159 (e) 12,495,187 36 The current chairman of INEC is (a) Prof. Maurice
16 INTERNET means (a) internal network (b) e-mail (c) Iwu (b) Prof. Humprey Nwosu (c) Prof. Rasheed
international network (d) web-site (e) none of the Muniru Jega (d) Prof. Humprey Iwu (e) Prof. Atahiru
above Jega
17 Which of the following cities has ever been the 37 The winner of the 2011 African Youth Championship,
capital of the Federal Republic of Nigeria? (a) Lagos U-20 football fiesta held in south Africa is (a) Nigeria
(b) Calabar (c) Lokoja (d) Abuja (e) all of the above (b) Mali (c) Cameroon (d) Egypt (e) none of the
18 The presidential election which was annulled in above
Nigeria was held in (a) 1993 (b) 1994 (c) 1983 (d) 38 Which of the following is not a political party in
1984 Nigeria? (a) Accord party (b) action congress of
19 `What is the name of the immediate past speaker of Nigeria (c) peoples democratic party (d) labour party
the Nigerian House of Representative? (a) David (e) republican party
Mark (b) Pius Anyim (c) Dimeji Longe (d) Dimeji 39 The capital of cape Verde is (a) cape-town (b) Praia
Bankole (e) none of the above (c) Rhodesia (d) new-Orleans (e) none of the above
20 The administrative capital of South Africa is (a) 40 Brazil is a country in (a) Africa (b) America (c) Europe
Johnnesburg (b) Durbar (c) Pretoria (d) Soweto (e) (d) Asia (e) Australia
None of the above 41 The current Governor of the central bank of Nigeria is
21 Which African country was called the “Gold Coast”? (a) Sanusi Lamido (b) Chukwuma Soludo (c) Dora
(a) Cote d’ evoir (b) Ghana (c) Sierra-Leone (d) Akunyuli (d) Erastus Akingbola (e) none of the above
Liberia (e) Nigeria 42 With of the following died while in office as a Nigerian
22 In which ethnic group in Nigeria is the “talking drum” ruler (a) Alhaji Umaru Yar’Adua (b) General Sanni
commonly used? (a) Igbo (b) Hausa (c) Fulani (d) Abacha (c) General Murtala Muhammed (d) all of the
Yoruba (e) Ibibio above (e) none of the above
23 Which of the following is not a member of the oil- 43 The current prime minister of Britain is (a) Mr. John
producing states in Nigeria? (a) Ondo (b) Bayelsa (c) Major (b) Mr. Tony Blair (c) Mr. David Cameroon (d)
Delta (d) Lagos (e) none of the above Mr. Gordon Brown (e) Mr. James Brown

© 2014 www.examsguru.net | All Right Reserved Enquiries: call +234 (0) 802 652 9647 Page 21
Examsguru.Net – LASU POST UTME PAST QUESTIONS WITH SOLUTIONS 2005 - 2014
rd
44 Prince William and Kate Middleton were married at 65 The most numerous ethnic group in the northern 2/3
th
Westminster Abbey, London on (a) 11 April, 2011 of Nigeria is (a) Nupe (b) Tiv (c) Hausa-Fulani (d)
th st nd
(b) 29 April, 2011 (c) 1 May, 2011 (d) 2 May, Kanuri
2011(e) none of the above 66. What is the full meaning of INEC? (a) Independent
45 Which of the following is not a search engine? (a) Nigerian Electoral Commission (b) Independent
Google (b) Excite (c) Alta Vista (d) Yahoo (e) none of National Electrical Commission (c) Independent
the above Nigerian Education Commission (d) Independent
46 Which of the following is not current telecom service National Electoral Commission
provide in Nigeria? (a) Starcoms (b) Airtel (c) MTN 67 The last gubernatorial election in Nigeria took place
(d) Celtel (e) none of the above on (a) April 26, 2011 (b) April 26, 2010 (c) April 23,
47 www used in web address on internet means (a) 2011 (d) April 16, 2011 (e) none of the above
worldwide web (b) world wave web (c) world wide 68 Current Nigerian president Goodluck Jonathan was
web (d) world wave web (e) worldwide wave governor of (a) Borno State (b) Ebonyi State (c)
48 The first prime minister of Nigeria is (a) Dr. Nnamdi Bayelsa State (d) Cross River State (e) Anambra
Azikiwe (b) Alhaji Tafawa Balewa (c) Lord State
Mackpherson (d) Lord Lugard (e) Alhaji Shehu 69 Senate president of Nigeria from May 2007 – May
Shagari 2011 was (a) Dimeji Bankole (b) David Mark (c)
49 The blue buses using special lanes in Lagos are Alhaji Aliko dangote (d)d Bukola Saraki
commonly referred to as BRT buses. What does 70 Nigeria’s presidential villa is located in (a) Asokoro,
BRT means? (a) Babatunde Raji Transport (b( Abuja (b) Dodan Barracks, Lagos (c) Obasanjo
Babatunde Raji Transit (c) Bus Rapid Transit (d) Blue Farms, Otta (d) Aso Rock, Abuja
Rapid Transport (e) none of the above 71 Nigeria flag consists of (a) 3 blocks of green, white
50 HALTI is a country in (a) America (b) Africa (c) Asia and green (b) 3 horizontal bands of green, white and
(d) Europe (e) none of the above green (c) 3 vertical bands of green, white and green
51 The first Nigerians to win Olympic medal is (a) (d) 3 horizontal bands of green, white and yellow
Chioma Ajunwa (b) Nojeem Maiyegun (c) Fidelis 72 Who named Nigeria (a) Mungo Park (b) Lord Lugard
Okoro (d) Maitama Sule (e) none of the above (c) Laura Shaw (d) Charles Darwin (e) none of the
52 The first Nigerian to win the heavy weight boxing above
championship of the world is (a) Simon Joe (b) 73 Nigeria’s national anthem is called (a) I pledge to
Hogan Kid Bassey (c) Simon Peter (d) Bash Ali (e) Nigeria my country (b) Arise O compatriots (c)
none of the above Nigeria, we hail thee (d) none of the above
53 The first television station was established in Nigeria 74 The main accreditation body that enforces uniform
in (a) 1959 (b) 1960 (c) 1961 (d) 1962 (e) 1963 standard and sets admissions capacity of every
54 Which of the following two statements does not university in Nigeria is (a) the Nigerian Universities
explain the word “surfing”? I. A sport in which the Commission (b) The national universities commission
individual rides on the crest or along tunnel of a wave (c) the federal ministry of education (d) the joint
II. Exploration of the web (www) (a) I only (b) II only admission matriculation board
(c) I & II (d) neither nor II 75 Nigeria is organized into (a) 36 states including the
55 The headquarters of the United Nations is in (a) PCT, Abuja (b) 37 states (c) 38 states (d) states and
Addis-Ababa (b) New York (c) Washington (d) the FCT, Abuja
th
London (e) Lagos 76 Nigeria accounts for (a) 1/4 of Africa’s people (b)
th
56 The head of the commonwealth of nation is (a) 1/6 of Africa’s people (c) 1/10 of Africa’s people (d)
rd
Queen Elizabeth of England (b) Mr. Ban-ki Mon (c) 1/3 of Africa’s people
Mr. Kofi Anan (d) Mr. Boutros Boutros-Ghali 77 Nigeria has (a) over 200 ethnic groups (b) over 300
57 The LAGOS STATE UNIVERSITY was established ethnic groups (c) over 400 ethnic groups (d) about
in (a) 1962 (b) 1960 (c) 1961 (d) 1959 (e) none of three ethnic groups
the above 78 The most widely used Nigerian languages are (a)
58 Where is the Sahara Desert? (a) America (b) Asia (c) Fulani, Yoruba and Igbo (b) Hausa, Yoruba and Igbo
Australia (d) Europe (e) Africa (c) Igbo, Yoruba and Pidgin (d) Pidgin, Yoruba and
59 Which of the following is not a newspaper in Hausa
circulation in Nigeria? (a) The Guardian (b) The 79 Major ethnic groups in the south-eastern Nigeria
punch (c) the new Nigeria (d) the nation (e) Le Figaro include (a) Igbo, Kanuri and Efik (b) Igbo, Yoruba
60 Which country will host the 2018 FIFA World Cup and Ijaw (c) Igbo, Efik and Ibibio (d) Igbo Ijaw and Tiv
Football? (a) Russia (b) France (c) Qatar (d) Nigeria 80 The following is not an ethnic group in the south east
(e) South-Africa of Nigeria (a) Efik (b) Annang (c) Ijaw (d) Ibibio (e)
61 ...............is the most populous African nation (a) none of the above
Kenya (b) South African (c) Nigeria (d) Ghana (e) 81 The number “419” refers to the article of (a) the
Libya American criminal code dealing with internet fraud (b)
62 Nigeria’s national telephone code is (a) 080 (b) 234 the Nigeria criminal code dealing with internet fraud
(c) 233 (d) 009 (c) the Russian criminal code dealing with internet
63 Nigerian population is currently estimated to be about fraud (d) the Nigeria criminal code dealing with fraud
(a) 100million (b) 140million (c) 190million (d) 82 One of the following is not a Nigerian music (a) Fuji
200million (b) Yo-Pop (c) Afrojuj (d) Highlife (e) Gbedu
64 Who is the current vice-president of Nigeria? (a) 83 Albert Einesein, Sir Isaac Newton and Charles
Goodluck Jonathan (b) Alh. Shehu Shagari (c) Alh. Darwin have something in common (a) they are all
Namadi Samba (d) Alh. Jubril Aminu (e) none of the famous poets (b) they are all famous historians (c)
above they are all famous artists (d) they are all famous
scientist

© 2014 www.examsguru.net | All Right Reserved Enquiries: call +234 (0) 802 652 9647 Page 22
Examsguru.Net – LASU POST UTME PAST QUESTIONS WITH SOLUTIONS 2005 - 2014
84 Nigeria ranks ..................in African’s GDP (a) First 5 C
(b) second (c) third (d) fourth (e) fifth 6 D
85 The Nigeria civil was from (a) 1967-1968 (b) 1967- 7 C
1970 (C) 1967-1971 (d) 1971 8 C
86 The flag of Nigeria was first used in (a) 1959 (b) 1960 9 D
(c) 1961 (d) 1970 (e) 1971 10 B
87 Which of the following is not true about computers? 11 D
(a) process information electronically to produce data 12 A
(b) computers are programmable so we can tell them 13 B
what to do (c) computers also include small 14 A
programmed systems found in some appliances such 15 B
as phones, microwave ovens and video records (d) 16 C
computers consists of hardware and software 17 C
88 ....................is used to read pages set up on the 18 A
internet (a) (a) Google (b) satellite communication (c) 19 B
web browser (d) video conferencing (e) e-mail 20 B
89 Which of the following is not a feature of the e-mail? 21 C
(a) broad communication is needed (b) very cheap to 22 C
send (c) very quick delivery (d) can be sent at any 23 C
time (e) none of the above 24 B
90 Which of the following methods of communication 25 D
may be inconvenienced by time zone? (a) post (b) 26 A
fax (c) e-mail (d)mobile telephone 27 D
91 All but one of the following are actions for dealing 28 C
with viruses in your computer (a) scanning of 29 B
incoming mails (b) changing passwords regularly (c) 30 A
using filtering software to prevent downloads of 31 C
computer programs (d) not opening suspicious e-mail 32 D
or attachments 33 C
34 C
92 Search engines on the internet help to search for 35 A
information by (a) looking for heading of web 36 A
documents (b) looking for words inside web 37 B
documents (c) search other search engines (d) none 38 C
of the above (e) all of the above 39 D
93 The public sector in Nigeria does not include (a) the 40 B
army (b) the central bank of Nigeria (c) the business
owned by individual members of the public (d) the ENGLISH 2012/2013
police (e) the state government universities 1. A 1 D 41 C
th
94 May 29 in Nigeria is celebrated as (a) independence 2. B 2 B 42 B
day (b) mother’s day (c) democracy day (d) hanging 3. A 3 A 43 A
over day 4. B 4 E 44 C
95 The Corporate Affairs Commission in Nigeria is 5 D 5 E 45 B
responsible for (a) taxation of companies in Nigeria 6 D`` 6 D 46 E
96 The government spends taxes on (a) provision of 7 D 7 C 47 D
health services (b) education (c) defence 1961 (d) 8 B 8 A 48 D
waste disposal (e) all of the above 9 A 9 E 49 C
97 One of the following countries is not a republic (a) 10 D 10 C 50 B
USA (b) Nigeria (c) China (d) UK 11 B 11 E 51 C
98 The millennium development goals include (a) 12 A 12 D 52 A
reduce those suffering from hunger and poverty (b) 13 C 13 B 53 A
ensure all children get primary education (c) stop the 14 A 14 A 54 D
spread of aids (d) make sure girls get as much 15 A 15 A 55 A
education as boys (e) all of the above 16 B 16 E 56 B
99 ………….is a meeting of eight rich countries including 17 B 17 B 57 C
(a) USA, UK, Canada, South Africa and France (b) 18 A 18 C 58 D
UK, Germany, USA, Japan and Brazil (c) China, 19 A 19 B 59 A
USA, UK, Canada and Germany (d) UK, USA, Italy, 20 A 20 C 60 B
France, Germany (e) UK, USA, Canada, Japan, 21 C 21 C 61 D
Brazil 22 B 22 D 62 C
100 The 9/11 attack killed nearly 3,000 people and 23 B 23 A 63 B
destroyed the (a) IMF office (b) World bank (c) World 24 C 24 D 64 D
trade centre (d) wall street 25 D 25 A 65 C
26 B 26 B 66 A
MATHEMATICS - 2012/2013 PRACTICE 27 B 27 C 67 A
1 B 28 A 28 E 68 D
2 A 29 B 29 A 69 B
3 D 30 B 30 D 70 C
4 C 31 C 31 B 71 B

© 2014 www.examsguru.net | All Right Reserved Enquiries: call +234 (0) 802 652 9647 Page 23
Examsguru.Net – LASU POST UTME PAST QUESTIONS WITH SOLUTIONS 2005 - 2014
32 B 32 C 72 A 25 D
33 B 33 E 73 A 26 C
34 D 34 A 74 C 27 D
35 D 35 D 75 A 18 A
36 C 36 C 29 C
37 D 37 C 30 D
38 B 38 D 31 A
39 B 39 A 32 D
40 B 40 B 33 C
34 B
GENERAL PAPER 35 A
1 C 31 D 61 C 91 B 36 C
2 E 32 C 62 B 92 E 37 A
3 B 33 A 63 B 93 C 38 D
4 C 34 A 64 C 94 C 39 A
5 D 35 D 65 C 95 B 40 B
6 C 36 E 66 D 96 E 41 C
7 D 37 A 67 A 97 D 42 A
8 E 38 E 68 C 98 E 43 C
9 A 39 B 69 B 99 D 44 D
10 A 40 B 70 D 100 C 45 C
11 A 41 A 71 C 46 A
12 C 42 D 72 C 1 B 47 C
13 B 43 C 73 B 2 C 48 B
14 E 44 B 74 B 3 A 49 D
15 A 45 E 75 D 4 B 50 C
16 C 46 D 76 A 5 E 51 A
17 E 47 A 77 C 6 A 52 B
18 A 48 B 78 B 7 C 53 B
19 D 49 C 79 C 8 C 54 D
20 C 50 E 80 E 9 D 55 A
21 B 51 B 81 D 10 C 57 D
22 D 52 B 82 B 11 B 58 C
23 D 53 A 83 D 12 D 59 D
24 D 54 D 84 C 13 C 60 C
25 B 55 B 85 B 14 B 61 D
26 B 56 A 86 B 15 C 62 D
27 B 57 A 87 B 16 B 63 D
28 E 58 E 88 C 17 A 64 B
29 C 59 E 89 E 18 C 65 C
30 E 60 A 90 A 19 A 66 C
20 E 67 A
21 – 40 Do it yourself 68 C
69 C
2011/2012 MATHEMATICS 70 A
1. D 86 C 71 B
2 D 87 B 72 C
3 C 88 A 73 B
4 B 89 C 74 A
5 B 90 C 75 A
6 B 91 B 76 C
7 A 92 A 77 C
8 D 93 C 78 C
9 B 94 C 79 B
10 A 95 A 80 C
11 C 96 C 81 B
12 D 97 D 82 D
13 D 98 C 83 A
14 B 99 C 84 C
15 D 100 B 85 D
16 B
17 D ENGLISH LANGUAGE 2010/2011
18 B Read the passage carefully and answer the questions that
19 C follows.
20 A
21 A The banking sector consolidation, which ended in
22 B December 2005, came with the realization that those who
23 C understood the market merely plunged into it, notched
24 A premium stocks and left the directors of those firms to

© 2014 www.examsguru.net | All Right Reserved Enquiries: call +234 (0) 802 652 9647 Page 24
Examsguru.Net – LASU POST UTME PAST QUESTIONS WITH SOLUTIONS 2005 - 2014
grapple with the challenges of a hostile business 10 The first graduation ceremony of the university was
environment. Many investors who, hitherto, had staked attended by men from all …………..of life (a) works
their funds on high risk projects, which offers little or no (b) areas (c) walks (d) parts
profit, therefore, found solace in their discovery of the
stock market as a potential ENGLISH LANGUAGE 2009/2010
as cow. With the continuous fall in the prices of stocks in Read the passage carefully and answer the questions that
the past one month, however, many investors, particularly follow.
the new entrants, are beginning to lament the loss of
huge sums of money that could have been invested in Like immigration or globalisation, debating how Africa is
better yielding ventures. This suggests loss of confidence. reported is often a vexatious subject; it provides many people a
good chance to enjoy an argument with a closed mind. Its either
Investing in Nigeria capital market, to many, had been too you come to the debate feeling that in spite of scan MacBride’s
good to be true until sordid facts started to unfold in commission over two decades ago, all reports about Africa in
February, about the state of certain companies that had the western media will continuous to be about death, disease,
squeezed funds out of unsuspecting investors, especially despair and destruction; or you are asking whether this whole
via public offerings. What sparked off the fears was the business is about Africans wanting separate code of journalism
announcement by the securities and exchange that denies its own realities. Its black or white, and no room in
commission, that it was investigating some lending between for any shades of grey.
companies to ascertain the charge of price manipulation
suspected to be targeted at ripping off investors. Lack of context in reporting, which is perhaps the greatest
culprit at the moment, is just as much as problem among
Nevertheless, this was an eye opener for investors, who, African journalists covering Africa as it is among
many a times, blindly rush into the acquisition of shares European journalists or journalists anywhere for that
mainly of boost their ego, with neither a thorough matter – trying to understand why Josef Fritzl would lock
research as regards the financials of the companies, nor up his daughter in a cellar for 24 years and father seven
professional advice. children by her without anyone finding out.
(Taken from Azubuike Ishiekwo, view point – again the
1 Investing…………………..to many had been too good trouble with Africa)
to be true implies (a) optimism (b) hopelessness (c)
scepticism (d) belief
2 According to the passage the prevailing mood of new 1. According to the passage, a common problem for all
investors in the past month is one of (a) elation (b) journalist is (a) globalisation (b) immigration (c) lack
jubilation (c) regret (d) approbation of context in reporting (d) despair and destruction
3 This was an eye opener for investors means that (a) 2 The passage gives the impression that the
their eyes were closed (b) it was prevailing (c) they MacBride’s commission presented Africa in
were blind before (d) they could see their money ………….way
(a) an unfortunate (b) a favourable (c) a non-
In question 4 to 7 choose the option nearest in committal
meaning to the bold (d) confusing
and underlined word(s) of phrase 3 This passage is about (a) African journalism (b)
Josel Fritzl and his daughter (c) immigration and
4. He took exception to Ajuwa’s remark (a) was globalization (d) realities of report about Africa
delighted at (b) was excited by (c) got demoralized by
(d) objected to In each questions 4 to 7 choose the most appropriate
5 The new employee is so humble and friendly that he option nearest in meaning to the bold and underlined
will soon make himself popular. (a) welcome and word(s) or phrase
accepted (b) hated (c) estranged (d) rejected
6 The manager often quarrels with his staff because he 4 Our new vice-chancellor has stepped off on the
goes around with a chip on his shoulder. (a) wrong foot (a) injured his foot (b) stepped on the
deformed shoulder (b) readiness to be angered (c) wrong (c) made a costly mistake (d) started off badly
look of contempt (d) an air of superiority 5 Much to her chagrin, the bridegroom did not turn up
7 Telling Ali to break his habit of coming late to work is for the wedding (a) wonder (b) surprise (c)
like knocking your head against an brick wall. (a) disappointment (d) depression
asking him to beat you up (b) making a fool of 6 The chairman’s reaction was a storm in a tea cup
yourself (c) making him break down (d) trying the (a) suitable for the occasion (b) less serious than it
impossible appeared to be (c) more serious than necessary (d)
greatly diminished in scope
Choose the word(s) or phrase(s) which best fills 7 Do this job while I am away, but take your time. (a)
the gap(s) be careful (b) be thorough (c) be fast (d) don’t be in
8 Owing to the constant harassment of the populate by hurry
armed robbers, all night guards have been instructed
to shoot every moving thing. (a) at sight (b) by sight In question 8 to 10, choose the option opposite in
(c) in sight (d) off sight meaning to the
9 My wife and I were to celebrate our silver wedding underlined word or phrase
anniversary last Sunday unfortunately on that
…………..day (a) fruitless (b) faithful (c) in sight (d) 8 Good students can easily identified spurious
off sight arguments (a) genuine (b) interesting (c) false (d)
illogical

© 2014 www.examsguru.net | All Right Reserved Enquiries: call +234 (0) 802 652 9647 Page 25
Examsguru.Net – LASU POST UTME PAST QUESTIONS WITH SOLUTIONS 2005 - 2014
9 The athlete’s skin now flabby as a result of his 11 Anini was a mindless criminal (a) strong (b) mindful
changed circumstance (a) oily (b) neat (c) firm (d) (c) feeling (d) memorable
weak 12 The judge blamed the plaintiff for misleading the court
10 We watched in wonder as she rattled away in the (a) defendant (b) complainant (c) accused (d)
esoteric language (a) inscrutable (b) familiar (c) prosecution
secret (d) obscure 13 The thieves vandalized the valult’s lock (a) repaired
(b) arranged (c) serviced (d) actuated
ENGLISH LANGUAGE 2008/2009 14 The demonstrators have refused to call off local off
Read the passage carefully and answer the questions that their action (a) consolidate (b) start (c) resume (d)
follow. end
15 The angry waves swallowed the boat (a) gentle (b)
Delinquency describes actions that would not be if unruffled (c) cool (d) serene
performed by adults. If a young person performs one of
such actions then he has committed a crime. Reserves as necessary to promote a
Delinquency is one of several status offences that can be federal……………(9)………..rate
committed only by people in particular stations of life close to its target rate of 5.25 percent.
determine by age, profession or a person’s role in society.
For young people such offences include drinking, driving 6 A liquidity B money C cash D
and smoking under age. Usually they are offences only facility
to tile extent that they help top preserve some of the good 7 A lending B repurchase C credit D
things of life for the exclusive enjoyment of the adult borrowing
world. Delinquency is therefore a weapon forged in 8 A arrangement B issuance C purchase
adults minds and directed by adult hands against young D functioning
people. It is born out of envy, adult pride and intolerance. 9 A funds B money C liquidity D cash
If the world changed overnight and the responsibility to
make and enforce laws fell on juvenile shoulders, the In question 10, choose the option opposite in
adults should expect a raw deal in return. Delinquency meaning to the
would then certainly refer only to many of the adult now word underline
freely committed by them. 10 children at the age of ten usually voracious (a)
without appetite
1 The writer of the passage believes that delinquency (b) very weak (c) very intelligent (d) very troublesome
laws are (a) not relevant to human society (b) only fit
for young people (c) unfair to the juveniles (d) very In question 11, choose the option opposite in
unfair to the world meaning to the
2 Status offences those that can be committed by (a) all word underline
adults (b) juveniles only (c) delinquent juveniles only 11 There were sporadic occurrence of small pox in the
(d) specified classes of people village
3 In the view of the writer, drinking underage is an (a) widespread (b) occasional (c) sparse (d) regular
offence because (a) adults wants to have all he drink
to themselves (b) adults do not want juveniles to get Choose the option that gives the meaning nearest
drink (c) drunken juveniles can cause disorder in to that of the idioms underlined in the sentences
society (d) adults have to protect young persons in question 12
4 This world changed overnight (a) there would be
more delinquency laws (b) delinquency would refer to 12 He had his fingers in every pie (b) loved eating pie (c)
at adults action (c) delinquency would no more be a was involved in everything (d) baked all the pies
crime (d) delinquency would also change in meaning
5 When young people make and enforce laws (a) tile 13 She read the letter and her face fell (a) she read
responsibility will be too heavy on their shoulders (b) closely (b) she started crying (c) she was surprised
they would retaliate against the adults (c) the laws will (d) she looked suddenly disappointed
be juvenile (d) the world will be turned upside down
Choose the best option that completes the sentence
In question 6 to 15 choose the option opposite in in question 14
meaning to the
world in bold 14 To…………..Nigerians, corruption in high places is
not strange
6. Our government is making determined efforts to (a) we (b) us (c) ours (d) ourselves
eradicate illiteracy (a) compulsory (b) ineffective (c)
innocent (d) unreliable From the word letter A-D choose the word that best
7 The LAGOS STATE UNIVERSITY has a large intake completes the
of students each year (a) rejection (b) calmly (c) sentence in question 15
furiously (d) laboriously
8. Ngozi flogged the girl reluctantly (a) eagerly (b) 15 The quest is about rounding…………….his speech
calmly (c) furiously (d) laboriously (a) of (b) off (c) up (d) down
9 Sola’s car is badly damaged: he has to look for an
expert mechanic to fit it (a) uneducated (b) 1. The most important aspect of agriculture mentioned
amateurish (c) awkward (d) unscientific in the passage is (a) poultry (b) fishery (c) livestock
(d) farming
10 Coastal plains are often very densely populated (a)
weakly (b) badly (c) rarely (d) sparsely

© 2014 www.examsguru.net | All Right Reserved Enquiries: call +234 (0) 802 652 9647 Page 26
Examsguru.Net – LASU POST UTME PAST QUESTIONS WITH SOLUTIONS 2005 - 2014
2 Farming in many countries today is (a) an alternative 16 Disturb (a) comfort (b) affair (c) carry (c) search
to poultry (b) of great assistance to the economy (c) 17 Pause (A) country (b) sport (c) dual (d) pounce
for those who are out of job (d) for the illiterates
3 Some of the countries produce more and better crops Choose the word that has the same consonant sound
than other because the farmers in the former (a) are as the one
more educated (b) have greater manpower (c) have represented by the underlined letter.
more modern equipment (d) have more fertile land
4 In order to help improve the faring the government 18 Yam (a) fool (b) humble (c) spurious (d) crude
should (a) give all farmers enough money to work 19 Lake (a) would (b) talk (c) table (d) calm
well (b) well enough fertilizer to all farmers (c) find
ways of financing and modernizing the farming Select the word that has the same stress pattern as
system (d) help farmers with the storage of their the given word
crops 20 Salad (a) canoe (b) fifteen (c) distress (d) upright
5 A lot of crops harvested are wasted because farmers
(a) allow insect and pest to destroy their crops (b) do ENGLISH LANGUAGE 2005/2006
not have enough money to investors (c) do not have Read the passage carefully and answer the question that
good storage facilities (d) harvest too much at a time follow.

In each of the following question fill the gap with the In 1973, Japanese sericulture arrived in Malawi with a
appropriate batch of 40,000 silkworm eggs. They were taken
option from the list provided. Bvumbwe Agricultural research stallion in Thyolo District.
In this station, work is being done to determine favourable
6. There is no point getting upset by the problems I take silk worm raring conditions and areas where mulberry
them……………(a) in my stride (b) on the spur of the trees whose leaves the worms feed on, could grow well.
moment (c) by leaps and bounds (d) in a Jiffy According to the researchers, the silkworm which
7 I know that your friend will not accept the eventually develop into cocoons from which raw silk is
proposal…………..(a) and you neither (b( and neither produced-do well in areas with climatic conditions.
you (c) neither do you (d) neither will you
8 I have been trying to locate ………………(a) since Silk is one of tile strongest of fibres. Infact for thousands
five days (b) five days now (c) for five days (d) since of years, silk fabrics have been regarded as the most
five days now beautiful and durable materials woven by man. Many
9 The ………….of the participating countries will hold a people call silk the “cloth of kings and queens”
pre-conference on the eve of the conference (a)
auditor generals (b) auditor general (c) auditors The weaving of silk originated in china. An old Chinese
general (d) auditor generals believed to be written by Confucius, tells us that the write
10. It all depends on what……………….(a) does he want of emperor huangi-li was the first person to make fabrics
(b) he want (c) he does wanted (d) he wanted of silk. Around 2040 B.C, Emperor Huangi-ti asked his
Choose the word nearest in meaning to the wife his Ling-shih to study the worms that were destroyed
underlined word the mulberry trees in his garden. The empress took some
of the cocoons she picked up the gauzy mass shill had
11 I would not like you to do the work piecemeal discovered silk. She was so delighted with the discovery
(a) at meal time (b) bit by but (c) badly (d) as a and even wore a ceremonial robe for the Emperor out of
master place the cocoon threads. After that of the officials in the
12 I cannot understand how he suddenly became Emperor’s court wore brightly dyed silk robes on
audacious contrary to his nature (a) bold (b) pound important occasions.
(c) rude (d) hostile
People in other countries regarded the new fibres as
Choose the word opposite in meaning to the something rare and beautiful. A few traders went to china
underlined word to learn about making cloth from silk, but the Chinese
kept their silk worms a closely guarded secret.
13 Rich citizens are often niggardly in their ways (a)
pompous (b) generous (c) sordid (d) soft 1. Sericulture (a) carried out in china (b) the breeding of
14 The security recently enacted law on importation is silk worms of the production for silk (c) the research
quite nebulous done all silkworms (d) the breeding cloth from the
(a) vague (b) strategic (c) definitive (d) impressive cocoons of silkworms (d) the breeding of silkworms in
Malawi
Choose the option that best conveys the meaning of 2 It is implied in this passage that silk was discovered
the sentence (a) after years of hard work and research by the
Underlined empress (b) by accident (c) in the search for a more
durable fibre for making cloth (d) after some
15 My friend will never forgive his wife because she left experience carrier out by tile Japanese (e) by design
him in the lunch in his hour of need (a) abandoned 3 According to sericulture, silkworms (a) cannot survive
him (b) despised him (c) disrespected him (d) in a warm climate (b) may be reared on any tree (c)
disinherited him do well in areas with a climate (d) produce the longest
threads when they are fed leaves from the top of the
Choose the word that has the same vowel sound as mulberry tree (e) are destroyed by heat.
the 4 The work carried out the agricultural research station
represented by the underlined letter. in Malawi on the silkworm eggs was to (a) try to breed
cocoons which would produce more silk (b) determine

© 2014 www.examsguru.net | All Right Reserved Enquiries: call +234 (0) 802 652 9647 Page 27
Examsguru.Net – LASU POST UTME PAST QUESTIONS WITH SOLUTIONS 2005 - 2014
the survival rate of silkworms (c) find out the most 3 B
suitable areas and conditions search for rearing 4 D
silkworms (d) search for better method of rearing 5 A
silkworms (e) find out how cocoons becomes 6 B
silkworms 7 D
5 choose the meaning which best fits the underlined 8 A
phrase from the passage, closely guarded scret 9 D
(a) carefully hidden from the knowledge of others (b) 10 C
secretly processed the business with armed guards
(c) carefully hidden from the view of strangers (d) 2009/2010
secretly known (e) unknown 1 C
2 B
In each of the question 6 - 9, fill each group with the 3 A
most 4 B
appropriate option from the list provided 5 C
6 C
6 He not absent himself from lectures (a) dares (b) dare 7 D
(c) didn’t dare (d) do care 8 A
7 The conference turned out to the (a) five-days (b) five 9 C
day (c) five-day’s (d) five days’ 10 B
8 Ojo has an aversion ………….swimming (a) to (b) for
(c) in (d) with 2008/2009
9 The poor………….our attention and kindness (a) 1 C
deserves (b) does deserve (c) do deserves (d) 2 D
deserve 3 A
4 D
Choose the option nearest in meaning to the word(s) 5 B
or phrase in 6 D
italics 7 D
8 A
10 He will be livid if he finds out you are here (a) 9 B
extremely happy (b) extremely angry (c) very better 10 D
(d) very surprised 11 B
12 A
Choose the option opposite in meaning to the 13 A
word(s) or phrase in 14 C
italics 15 C

11 He meteoric rise to the fame surprised everyone 2007/2008 2006/2007


(a) tardy (b) swirl (c) gradual (d) surprised 1 D 1 D
2 B 2 B
Choose the option that best convey the meaning of 3 A 3 C
the sentence 4 B 4 C
5 A 5 C
12 An official has been appointed to show them the 6 B 6 A
ropes (a) assigned duties to the them (b) train them 7 C 7 D
(c) assist them (d) punish them 8 D 8 C
9 C 9 C
Select the word that has the same consonant sound 10 A 10 D
as the one 11 C 11 B
represented by the underlined letter 12 C 12 A
13 D 13 B
13 goal (a) face (b) boat (c) park (d) more 14 B 14 C
15 B 15 A
Select the word that has the same consonant sound 16 B 16 A
as the one 17 C 17 B
represented by the underlined letter 18 D 18 C
19 D 19 C
14 suggest (a) great (b( judge (c) angle (d) gather 20 C 20 D

Select the word that has the same pattern of stress as 2005/2006
given word. 1. B
15 Particular (a) indelible (b) respected (c) determine (d) 2 B
disrespectful 3 C
4 C
5 A
2010/2011 ENGLISH ANSWERS 6 C
1 C 7 A
2 C 8 A

© 2014 www.examsguru.net | All Right Reserved Enquiries: call +234 (0) 802 652 9647 Page 28
Examsguru.Net – LASU POST UTME PAST QUESTIONS WITH SOLUTIONS 2005 - 2014
3
9 A x – 2x2 – x 2
10 C (a) 1 (b) x – 1 (c) x 1 (d) 1
11 A x+2 x+1 x+2 x 2
12 B 9 Express 5x – 12 in partial fractions
13 B (-2) (x-3)
14 B (a) 2 3 (b) 2 3 ( c) 2 3 (d)
15 D 5 4
x -2 x-3 x -2 x-3 x -3 x-2 x
MATHEMATICS 2010/2011 -3 x-2
1. Calculate 33105 – 14425 (a) 1313 (b) 2131 (c) 4302 10 Which of the following binary operations is
(d) 1103 commutative in the set of integers? (a) a-b=a-2b (b)
2
2 Convert 3.1415926 to 5 decimal places a.b=a+b-ab (c) a-b=a +b
(a) 3.14160 (b) 3.14159 (c) 0.31415 (d) 3.14200 (c) a.b=a(b+1)
3 The length of a notebook 15cm, was measures as 2
16.8cm. Calculate the percentage error to 2
significant figures. MATHEMATICS 2008/2009
(a) 12.00% (b) 11.00% (c) 10.71% (d) 0.12% 1. The population of a village is 6538. Express this
4 A worker’s present salary is N24,000 per annum. number to three significant (a) 6540 (b) 6538 (c) 6530
His annual increment is 10% of his salary. What (d) 654
would be his annual salary at the beginning of the 2 What must be added to the express x2 – 18 to make
third year? (a) N28 800 (b) N29 040 (c) 31 200 (d) perfect significant (a) 9 (b) 36 (c) 72 (d) 81
N31 944 3 A bricklayer measured the length of a wall to be
5 Express the product of 0.0014 and 0.011 in standard 4.10m. if the actual length of the wall is 4.25m, find
form his percentage error.
4 -3 -4
(a) 1.54 x 10 (b) 1.54 x 10 (c) 1.54 x 10 (d) 1.54 (a) 39/17% (b) 327/41% (c) 15% (d) 355/17%
-2
x 10 4 A car is travelling at an speed of 80km/hr. its speed of
1
6 Evaluate (81)¾ - (27) /3 80km/hr. it speed in meters per second (m/s) is (a)
3
3 X 2 13.3m/s (b) 22.2m/s (c) 133.3m/s (d) 222.m/s
(a) 27 (b) 1 (c) 1/3 (d) 1 5 A cylinder of base radius 4cm is open at one end. If
7 Find the value of (16) 3/2 + log10 0.0001 + log2 32 the ratio of the area of its base to that of its curved
(a) 0.065 (b) 0.650 (c ) 5.500 (d) 65.000 surface is 1:4, calculate the height of the cylinder. (a)
1cm (b) 2cm (c) 4cm (d) 8cm
8 Simplify 12 – 3 6 The area of a parallelogram is 513cm2 and the height
12 + 3 (a) 1/3 (b) 0 (c) 2 (d_ 1 is 19cm. calculate the base (a) 13.5cm (b) 25cm (c)
9 Four members of a social first eleven cricket term are 27cm (d) 54cm
also members of the first fourteen rugby team. How 7 A headmaster contributes 7% of his income into a
many boys play for at least one of the two teams? (a) fund and his wife contributes 4% of her income. If the
25 (b) 21 (c) 16 (d) 3 husband earns N5,500 per annum and the wife earns
10. If S= (x:x2 = 9, x < 4), then S is equal to (a) 0 (b) (0) N4,000 per annum. Find the sum of their annual
(c) II (d) (%) contribution to the fund (a) N1,045 (b) N605 (c) 545
(d) 490
2
8 factorize 2x – 21x + 45 (a) (2x-9)(x-5) (b) (2x-15(x+3)
MATHEMATICS 2009/2010 (c) (2x+15) (d) (2x-15)(x-3)
o
9 A town 1 is on bearing 315 from town Q while town
3
1 If x – 1 and x + 1 are both factors of the equation x + R is south of town P and west of town Q. If town R is
2
px + qx + 6 = 0, evaluate p and q. (a) 6, -1 (b) 6, 1 60km away from Q how far is R from P?
(c) 1, -1 (d) 6, 6 10 Solve the simultaneous equations y=3x;4y-5x=14 (a)
2 Find a positive value of p is the expression 2x2 px + p -2,-6 (b) 2,6 (c) 2,-6 (d) -2, 6
leaves a reminder 6 when divided by x-p (a) 1 (b) 2 11 A cliff on the bank of a river is 300 meters high. If the
(c) 3 (d) 4 angle of depression of a point on the opposite site of
3 Find T in terms of K, Q and S if S = 2r (QT + K) the river is 60o, find the width of the river (a) 100m (b)
(a) S
2
K (b) S
2
K (c) S
2
K (d) 75 3m (c) 100 3m
S
2
(d) 200 3m
2 2 2
2r Q Q 4r Q Q 2r Q 12 The sum of the interior angles of a regular polygon is
2 30 right angles. How many sides has the polygon?
4r Q
2 2 (a) 17sides (b) 26sides (c) 30sides (d) 34sides
4 The graph of f(x) = x – 5x + 6 crosses the x-axis at
the points. 13 Three balls are drawn one after the other with
(a) (-g,0), (-1,0) (b) (-3,0), (-2,0) (c) (-6,0), (1,0) (d) replacement, from a bag containing 5 red, 9 white
((2,0), (3,0) and 4 blue identical balls. What is the probability that
5 Factorize completely the expression abx1 + 6y – 3ax they are one red, one white and one blue?
– 2byx (a) 5/102 (b) 5/136 (c) 5/162 (d) 5/204
(a) (ax-2y)(bx-3) (b) (bx + 3)(2y-ax) (c) (bx+3)(ax-2y) 14 The pilot of an aeroplane, flying 10km above the
(d) (ax-2y)(ax-b) ground in the direction of a landmark, views the
o
6 Solve the following inequality (x 3) (x 4) < 0 landmark to have an angle of depression of 35 and
o
(a) 3 < x < 4 (b) 3 < x < 4 (c) 3 < x <4 (d) 3<x<4 55 . Find the distance between the two points of
o o o
7
th th
The 4 term of an A.P is 13 while the 10 term is 31. observation. (a) 10(sin 35 – 5sin 55 ) (b) 10 (cos 35
o o o o
st
Find the 21 term (a) 175 (b) 85 (c) 64 (d) 45 – cos 55 ) (c) 10 (tan 35 – tan 55 ) (d) 10 (cot 35 –
o
cot 55 )
2
8 Simplify x – 1
© 2014 www.examsguru.net | All Right Reserved Enquiries: call +234 (0) 802 652 9647 Page 29
Examsguru.Net – LASU POST UTME PAST QUESTIONS WITH SOLUTIONS 2005 - 2014
15 In a family of 21 people, the average age is 14 years. 16
If the age of the grandfather is not counted, the Age in 37. 38. 39. 40. 41.
average age drops to 12 years. What is the age of
the grandfather? (a) 35 years (b) 40 years (c) 42 years 13 14 15 16 17
years (d) 54 years
No of 42. 3 43.10 44. 45. 46.15

MATHEMATICS 2007/2008 students 30 42


1 If a*b= + ab evaluate 2* (12*27)
(a) 12 (b) 9 (c) 6 (d) 2
2 Find the sum to infinity of the following sequence The frequency distribution above shows the ages of
1,9/10, (9/10)2, (9/10)3 students in a secondary school. In a pie chart
(a) 1/10 (b) 9/10 (c) 10/9 (d) 10 constructed to represent data, the angle
o o
3 Find the value of k if corresponding to the 15 years old is (a) 20 (b) 30
o o
-2 1 1 (c) 54 (d) 108
2 1 k = 23
1 3 -1 French, 90
Economics, 150
(a) 1 (b) 2 (c) (d) 4
4 If X = 1 2 and Y = 2 1 find XY
C.R.K, 30
0 3 4 3
History, 90

(a) 10 7 (b) 2 7 (c) 10 4 (d) 4 3


12 9 4 17 4 6 10 9 17 The pie chart above shows the distribution of
5
o
In a triangle XYZ, <YXZ = 44 and <XYZ = 112 ,
o students in a secondary school class. If 30 students
calculate the acute angle between internal bisectors offered French, how many offered C.R.K? (a) 25 (b)
o o
of <XYZ and <XZY (a) 12 (b) 56 (c) 68 (d) 78
o o 15 (c) 10 (d) 8
o o
6 Find the distance between two town P(45 N 30 W)
o o 18 The mean and the range of the set of number 1, 20,
and Q(15 S, 3 W) if the radius of the earth is
1.00, 0.90, 1.40, 0.80, 1.20 and 1.10 are in and r
7000km (?=22/7)
respectively. Find m + r (a) 1.11 (b) 1.65 (c) 1.85
(a) 1100/3km (b) 2200/3km (c) 22000/3km (d)
(d) 2.45
11000/3km
7 Two perpendicular lines PQ and QR intersect at (1, Class 1-3 4-6 7-9
-1), if the equation of PQ is x-2y + 4 = 0, find the
equation of QR Frequency 5 8 5
(a) x-2+1=0 (b) 2x+y-3=0 (c) x-2y-3=0 (d) 2x + y-
1=0
8 P is on the locus of points equidistant from two
19 Find the standard deviation of the data using the
given points X and Y, UV is a straight line through Y table above. (a) 5 (b) 6 (c) 5/3 (d) 5
o
parallel to the locus, if < PYU is 40 , find <XYP. 20 The variance of the scores 1, 2, 3, 4, 5 is
o o o o
(a) 100 (b) 80 (c) 50 (d) 40 (a) 1.2 (b) 1.4 (c) 2.0 (d) 3.0
9 The base diameter of a cylinder is 14cm while the
height is 12cm, calculate the total surface area if the
2
cylinder has both a base and a top (a) 836cm (b)
2 2 2 MATHEMATICS 2006/2007
528cm (c) 308cm (d) 154cm
1 If x,y and x are angles which lie between 90 and
10 A school boy lying on the ground 30, away from the 180 and x<y<z, then (a) sin x< siny<sinz (b)
foot of a water tank tower observes that the angle of sinx>siny>sinz
o
elevator of a top of the tank is 60 , calculate the (c) sin y < sin x < sin z (d) siny > sinx > sinz
height of the water tank. (a) 18.2m (b) 12.2m (c) 2 If the length of the side of a right angled triangle are
6.2m (d) 3.2m given as x-1, x and x + 1, then the hypotenuse is (a)
11 QRS is a triangle with QS=12m, <RQS=30 and
o 13 (b) 4 (c) 7 (d) 5
o 3 Two sectors of the same circle subtend angles of 75
<QRS=45 (a) 18.2m (b) 12.2m (c) 6.2m (d) 3.2m
and 30 at the centre of the circled. Their areas are in
12 The derivative of cosec x is (a) tan x cosec x (b) cot the ratio (a) 25:4 (b) 5:2 (c) 115:112 (d) 3:1
x cosec x (c) tan x sec x (d) cot x sec 4 A cylinder metal rod of radius 10cm and length 35cm
2
13 For what value of x is the tangent to the curve y=x - is to be melted down and recast into circular comes of
base radius 7cm and height 15cm. if  = /7 the
22
4x+3 parallel to the axis? (a) 3 (b) 2 (c) 1 (d) 0
14 Two variables x and y are such that dy/dx=4x-3 and examination.
y=5 when x=2, find y in terms of x. 5 The locus of point which is equidistant from the lines
2 2 2 x x = 0, and y + x = 0, for x > 0, is the (a) x-axis (b) y-
(a) 2x -3x+5 (b) 2x -3x+3 (c) 2x -3x (d) 4
2 axis (c) line x=1 (d) 80
15 Find the area bounded by the curve y=3x -2x+1, the 6 A binary operation is defined over the set of real
ordinates x=1 and x=3 and the x-axis numbers such as a*b=ax (b12). Find 3*(4*2) (a) 54
(a) 24 (b) 22 (c) 21 (d) 20 (b) 60 (c) 72 (d) 80

© 2014 www.examsguru.net | All Right Reserved Enquiries: call +234 (0) 802 652 9647 Page 30
Examsguru.Net – LASU POST UTME PAST QUESTIONS WITH SOLUTIONS 2005 - 2014
7 Which of the following binary operations is (are) NOT the education budget is (a) N1,250 (b) N1,500 (c)
closed over the set of odd integers? I Addition II. 1,800 (d) N,000
Subtraction III. Multiplication IV. Division (a) I and III 20 The, standard deviation of the set of numbers -3, -2, -
only (b) II and IV only (c) IV only (d) I, II and IV only 1, 0, 1, 2, 3, is (a) 4 (b) 22 (c) 2 (d) 4
3
8 Limit x – 1 is equal to
2
X –1 (a) 0 (b) 1 (c) 1½ (d)
Infinity

9 Differentiate y = y 1 with respect to x


X
MATHEMATICS 2005/2006
(a) 1 1 (b) 1 1 (c) x - 1 1 If Paul can paint a fence in 2 hours and Fred can print
the same fence in 3 hours, Paul and Fred working
2 x + 2x x x - 2x x together can paint the fence in (a) 2.5hours (b) 1.2
x hours (c) 5.0 hours (d) 1.0 hour
2 If two angles of a triangle are acute angles, the third
3
(d) x-½ + x- /2 angle (a) is less than the sum of two given angles (b)
is an acute angle (c) is the largest angle of the
2 2
10 If y = x sin x, then d y equals triangle (d) may be an obtuse angle
2
dx 3 DCCXLIX in roman numerals represents the number
2 2 2
(a) 2xsin x x cos x (b) 4xcosx + (2-x )sin x (c) (2-x ) (a) 749 (b) 764 (c) 1249 (d) 1264
2
sin x 4xcosx (d) 2sin x-x sin x 4 The sum of an odd number and even number is (a)
11 The value of (1-2x)dx is (a) 0 (b) 1/8 (c) -1/4 (d) ¼ sometimes an even number (b) always divisible by 3
12 The coordinator of point X and Y on the line XY are or 5 or 7 (c) always an old number (d) always a prime
XY are X (-2,-1) and Y-(2,1). This equation of the number
perpendicular bisector of XY is (a) y-2x=0 (b) y-1-2x0 5 If one angle of a triangle is three times a second
(c) y0-1-2x=5 (d) y-2x-3 angle and the third angle is 20 degrees more than the
o o
13 The point at which the tangent to the curve y=x2 – second angle, the second angle is (a) 32 (b) 34 (c)
o o
5x14 makes an angle of 135 with the positive x—axis 40 (d) 50
2 2
is (a) (1,0) (b) (3,2) (c) 5,9 (d) (2,2) 6 The factors of a b are
2 2 2
14 If P – 2 3 and the inverse of P is 2 x then values (a) (a-b), (a+b) (b) (a-b), (a-b) (c)a , -b (d) (a -1), (1
2
of x and y are b)
1 2 y 2 7 A bank manager received 8 times as much salary as
one of the messengers. If the sum of their salaries is
(a) x-3,y-1 (b) x-3,y-1 (c) x=3,y=1 (d) x=1,y=3 N135,000, the manager’s salary is (a) N15,000 (b)
15 In attest conducted on the three arms of a class, 100,000 (c) 120,000
class A with 30 students scored and average of 5.5 (d) N125, 000
marks, class B with 25 students scored an average of 8 Given a rectangle hexagon, calculate each interior
o o o o
6.4 marks, and class C got an average of 6.25 marks. angle of the hexagon (a) 60 (b) 30 (c) 120 (d) 45
If the average mark for the whole class is 6.0, the 9 The lengths of the sides of a right-angled triangle at
number of students in class C is (a) 35 (b) 30 (c) 25 (3x__)cm, (3x___)cm, find the value (a) 2 (b) 6 (c) 18
(d) 20 (d) 12
10. Thirty boys and x girls sat for a test. The means of
The distribution of marks scored by students in a the boy’s score and that of the girls were respectively
class test is given below. 6 and 8 . Find x if the total score was 468. (a) 38 (b)
24 (c) 36 (d) 22
Mark 2 3 4 5 6 7 8 9 11 Ade and Uche can do a piece of work in 18 days.
No. of 1 2 3 4 5 3 1 1 Ade can do it alone in x days, whilst Uche takes 15
days longer to do it alone. Which of the following
Students equations describes x?
2 2 2
(a) x 5x 18 = 0 (b) x – 20x + 360 = 0 (c) x 21x 270 =
16 From the table, the mean anti the modal marks are 0
2
respectively (d) 2x + 42x 190 = 0
(a) 5,6 (b) 5,4,6, (c) 5,5,5 (d) 5,4,5,5 12 What is the probability that a number chosen at
random from the integers between 1 and 10 inclusive
The figure above shows the give drawn for the marks is either a primer or a multiple of 3? (a) 7/10 (b) 3/5
scored by 500 candidates in a matriculation (c) 4/5 (d) ½
examination. Use the information in the figure to 13 A bags contains 4 white balls and 6 red balls. Two
answer questions 216 and 21 red balls are taken from the bag without replacement.
What is the probability that they are both red? (a) 1/5
17 The median and the upper quartile are respectively (b) 1/3 (c) 2/15 (d) 2/9
(a) 60,65 (b) 150,751 (c) 50,70 (d) 55,75 14 If y=x/(x-3) + x(x + 4) find y when x = -2
18 The score of 50% corresponds to a percentile of (a) -3/5 (b) 3/5 (c) -7/5 (d) 7/5
(a) 60.0 (b) 50.0 (c) 18.8 (d) 15.5 15 A man invests a sum of money at 4% per annum
19 When the monthly budget for a household is simple interest. After 3 years, the principal amount to
displayed on a pie chart; the sectorial angles for the N7,000.00. Find the sum invested (a) N7,840.00 (b)
various items of expenditure are as follows: Food N6,250.00 (c) N6,160.00 (d) N5,833.33
(1650). Education (90) budgeted for clothing, then
MATHEMATICS ANSWERS

© 2014 www.examsguru.net | All Right Reserved Enquiries: call +234 (0) 802 652 9647 Page 31
Examsguru.Net – LASU POST UTME PAST QUESTIONS WITH SOLUTIONS 2005 - 2014
4 C
2010/2011 2009/2010 5 A
1 B 1 D 6 A
2 A 2 A 7 A
3 D 3 B 8 C
4 C 4 A 9 D
5 C 5 B 10 C
6 D 6 C 11 C
7 C 7 C 12 B
8 C 8 A 13 B
9 D 9 B 14 A
10 B 10 B 15 B

2008/2009 2007/2008 PHYSICS 2010/2011


1 A 1 D 1 A man walks 1km due east and then 1km due north.
o o
2 A 2 B His displacement is (a) 1km N15 E (b) 1km N30 E
(c) 2kmN45 E
o
3 B 3 A
(d) 2kmN60 E
o
4 B 4 B
5 D 5 A 2 An air force jet flying with a speed of 335m/s went
6 C 6 B past an anti-aircraft gun. How far is the aircraft 5s
7 C 7 C later when the gun was fired? (a) 838m (b) 3350m
8 D 8 D (c) 670m (d) 16.75m
9 D 9 C 3 A palm fruit dropped to the ground from the top of a
10 B 10 A tree 45m tall. How long does it take to reach the
11 C 11 C ground? (a) 9s (b) 4.5s (c) 3s (d) 7.5s
12 A 12 C 4 A piece of rubber 10cm long stretches 6mm when a
13 A 13 D load of 100N is hung from it. What is the strain? (a)
-2 -3
14 C 14 B 60 (b) 6 (c) 6 x 10 (d) 6 x 10
15 D 15 B 5 A machine gun with a mass of 5kg fires a 50g bullet
16 B at a speed of 100m/s. the recoil speed of the gun is
17 C (a) 0.5m/s (b) 1.5m/s (c) 1m/s (d) 0.2m/s
18 D 6 The internationally agreed system of units (SI) for
19 D physical measurements are (a) Ib, ft, sec (b) g, m, se
20 C (c) kg, m, sec
(d) cm, g, sec
7 The force required to make an object of mass m,
travelling with velocity v, turn in a circle of radius r is
2 2 2
(a) mv /r (b) mr /v (c) mr/v (d) mv/r
8 A few grains of table salt were put in a cup of cold
water kept at constant temperature and left
undisturbed. Eventually all the water lasted salty.
This action is due to (a) convection (b) osmosis (c)
2006/2007 capillarity (d) diffusion
o
1 A 9 A temperature of 20oC is the same as (a) 36 F (b)
o o o
2 B 68 F (c) 11.1 F (d) 43.1 F
-1
3 B 10. The latent heat of fusion of ice is 80 calg . How
o
4 B much heat is required to change 10g of ice at 0 C into
5 A water, at the same temperature? (a) 80cal (b) 10cal
6 C (c) 9cal (d) 800cal
7 D
8 C PHYSICS 2009/2010
9 A 1. In a D.C. circuit a 10microfarad (mF) capacitor is
10 B placed in series with a 10 ohm resistor. The total
11 A resistance of the combination is
12 B (a) 10 ohms (b) 1 ohms (c) zero (d) 20 ohms (e(
13 B infinite
14 C 2 On an a.c. generator which of the following does not
15 C apply? (a) field magnet (b) commutator (c) armature
16 C (d) slip rings (e) carbon brushes
2 o
17 D 3 A gas at pressure P N/m and temperature 27 C is
o
18 B heated to 77 C at constant volume T. The new
2
19 A pressure is (a) 0.85 PN/m2 (b) 0.86 PN/m (c)
2 2 2
20 C 1.16PN/m (d) 1.18PN/m (e) 2.85Pn/m
2
4 The following are some units. I. Ns II. Nm III. N/m IV.
-1 -1
2005/2006 JoK .Jkg . What are the units of latent heat? (a) I (b)
1 B II (c) III (d) IV (e) V
2 C 5 The heat from the sun reaches the earth by the
3 A process of

© 2014 www.examsguru.net | All Right Reserved Enquiries: call +234 (0) 802 652 9647 Page 32
Examsguru.Net – LASU POST UTME PAST QUESTIONS WITH SOLUTIONS 2005 - 2014
(a) insulation (b0 precipation (c) conduction (d) 10 Which of the following is NOT a fundamental S.I
convection unit? (a) Radian (b) Amper (c) Kelvin (d) Second (e)
(e) radiation Kilogram
6 The amount of hat required to raise the temperature 11 A simple pendulum with a period of 2.0s has its
of a body is length doubled. Its new period is (a) 1.00s (b) 1.41xs
(a) thermal energy (b) thermal conduction (c) heat (c) 0.35s (d) 2.83s (e) 4.00s
lost (d) specific heat capacity (e) thermal capacity 12 The mode of heat transfer which does not require
7 Two lamps rated 40W and 220W each are connected material medium is (a) conduction (b) radiation (c)
in series. The total power dissipated in both lamps is convection (d) propagation
(a) 10W (b) 20W (c) 40W (d) 80W (e) none of the 13 If the refractive index of glass is 1, 5, the critical angel
-1
above at the air-glass interface will be (a) Sin 0.6667 (b)
-1
8 Three 5 ohms resistors connected in parallel have a sin 1.5 (c) cos+ 2/3
-1
potential difference of 60V applied across the (d) cos 1.5
combination. The current in each resistor is (a) 4A 14 What is the effect of the increase in the size, of the
(b) 36A (c) 12A (d) 24A (e) 10A hole of a pin-hole camera on the image? It (a) gives a
9 The PHCN charges 5k per KWh, what is the cost of blurred image (b) corrects for chromatic aberration (c)
operating for 24 hours a lamp requiring 1A on a 200V magnifies the image (d) brings the image into sharper
line? (a) 24k (b) 55k (c) 40K (d) 26k (e) 32k focus
10 A magnet is moved through a coil of wire. The e.m.f
produced in the wire depends on (a) the number of
turns in the coil (b) the strength of the magnet (c) the PHYSICS 2007/2008
speed at which the magnet is moved (d) all of the 1 An electric charge could be transmitted through (a)
above (e) none of the above wood, rubber and stone (b) paper, clay and plastic (c)
glass, acid and cloth (d) the human body, water and
PHYSICS 2008/2009 metals (e) all of the above
1. A man of mass 50kg ascends a flight of stairs 5m 2 A guitar string is 75cm long. The wavelength of its
high in 5 seconds. If acceleration due to gravity is fundamental note is (a) 75cm (b) 150cm (c) 37.5cm
2,
10m/s , the power expanded is (a) 100W (b) 300W (d) 112.5cm (e) 50cm
(c) 250W (d) 400W (e) 500W 3 A herdsman yelling out to a fellow herdsman heard
2 Which of the following arrangements in the sequence his voice reflected by a cliff 4s later. What is the
shown can be used to obtain a pure spectrum of velocity of sound in air if the cliff is 680m away? (a)
white light? 170m/s (b) 136m/s (c) 680m/s (d) 340m/s (e) 510m/s
(a) source, slit, converging lens, prism, converging 4 An object 1 cm high placed on the axis 15cm from a
lens, screen converging lens form an image 30cm from the lens.
(b) source, slit, diverging lens, screen (c) source, The size of the image is (a) 0.5cm (b) 1.5cm (c) 3cm
converging lens, prism, diverging lens, screen (d) (d) 2.5cm (e) 2cm
source, slit, prism, diverging lens, screen 5 The angular dispersion of a prism depends on (a) the
3 Ripples on water are similar to light waves in that they index of refraction only (b) the angle of incidence as
both (a) have the same wave length (b) are well as the index of refraction (c) angle of incidence
longitudinal (c) cannot be reflected (d) travel at the (d) the thickness of the prism (e) prism angle
same speed (e) can be refracted and diffracted 6 correction of myopic defects in the human eye we
4 A piece of wood is floating on water. The force acting require (a) convex lens (b) a concave lens (c) a
on the wood are (a) upthrust and reaction (b) weight combination of concave and convex (d) a prism (e) a
and reaction (c) weight and upthrust (d) upthrust and concave mirror
viscosity (e) weight and viscosity 7 Following which is different from the others? (a) rays
5 the angle between the direction of the earth’s (b) gamma rays (c) cathrode rays (d) ultraviolet rays
magnetic field and the horizontal is called the (a) (e) stared rays
3 -3
angle of deviation (b) magnetic declination (c) 8 Density of 400cm of palm oil was 0.9gcm before
magnetic meridian (d) angle of dip frying. If the density of the oil is 6gm-3 after frying,
6 A force of 16N applied to a 4.0kg block that is at rest assuming no loss of oil due to spilling, its new volume
3 3 3
on a smooth horizontal surface. What is the velocity (a) 860cm (b) 600cm (c) 240cm (d) 800cm3 (e)
3
of the block at t=5 seconds? 450cm
7 1,000 identical drops of oil of density 5000kg/m have 9 10kg car which was initially at rest travelled with an
2
a total mass of 5 x 10kg. one of the drops forms a thin acceleration 5m/s , its energy after 4s was (a) 2.5 x
2 5 3
film of area 0.5m on water. The thickness of the film 10 J (b) 2.5 x 10 J
-8 -9 3 4 5
is (a) 2 x 10 m (b) 2 x 10 m (c) 2 x 10 J (d) 5 x 103 J (e) 5 x 10 J
-9 -9 -8
(c) 2 x 10 m (d) 3 x 10 m (e) 2.8 x 10 m 10 The temperature at which the water vapour present in
8 An electric current of 3A flowing through an electric the air saturates the air and condense is known as (a)
healing element of resistance 20 embedded in 1kg boiling point (b) melting point (c) triple point (c) dew
o
of an oil raises at the temperature of the oil by 10 in point (d) tidal temperature
10 seconds, then the specific heat capacity of the oil 11 Which one of the following types of waves cannot
o o o
is.(a) 1.8J/g C (b) 6J/g C (c) .18Kj/kg C (d) travel through a vacuum? (a) sound wave (b) light
o o
1.8kJ/kg C (e) .06J/g C waves (c) infra-red waves (d) x-radiation (e) radio
9 The difference of potential between the terminals of a waves
cell is 2.2V. When a 4 resistor is connected across 12 A simple pendulum experiement the length of the
the terminals of this cell, the potential difference is 2 inextensible string is led by a factor of four, its period
volts. What is the internal resistance of the cell? (a) is increased by a factor of (a) 4 (b) /2 (c) ¼ (d)
0.10 (b) 0.25 (c) 0.40 (d) 2.50 (e) 4.00 2 (e) 2

© 2014 www.examsguru.net | All Right Reserved Enquiries: call +234 (0) 802 652 9647 Page 33
Examsguru.Net – LASU POST UTME PAST QUESTIONS WITH SOLUTIONS 2005 - 2014
13 What range of temperature is the expansion of water difference across the terminals of the capacitor is (a)
o o
anomalous? (a) + 208 C to + 212 C (b) -80oC to - twice the e.m.f of the accumulator (b) less than the
o o o
76 C (c) -4 C to 0 C em.f. by the potential drop across the resistor (c) zero
o o o o
(d) + 96 C to + 100 C (e) 0 C to +4 C (d) the same as the e.m.f. (e) greater than the e.m.f
14 When the vapour of a substance is in equilibrium with 9 Which of the following statements is true? The
its own liquid, it is said to (a) gaseous (b) unsaturated frequency of a vibrating string is (a) proportional to its
(c) liquefied (d) diffused (e) saturated length (b) inversely proportional to the tension within
it (c) proportional to the amplitude of vibration (d)
15 If the wavelength of a wave travelling with a velocity inversely proportional to its cross-section (e)
of 360m/s is 60cm, the period of the wave is (a) 6s (b) proportional to the square root of the tension within it
3.6s (c) 0.17s (d) 0.61 (e) 3s 10 An air bubble rises from the bottom to the top of a
16 The heights of the mercury thread in a mercury-in- water dam which is 40m deep. The volume of the
3
glass thermometer when in selling ice and then in bubble just below the surface is 2.5cm . Find its
steam are 3cm and 18cm respectively. At a volume at the bottom of the dam, if atmospheric
o 3
temperature of 60 C the height would be (a) 7.5cm pressure is equivalent to 10m of water (a) 10.cm (b)
3 3 3 3
(b) 9cm (c) 10.8cm (d) 12cm (e) 12.6cm 2.0cm (c) 1.6cm (d) 0.625cm (e) 0.5m
17 An object is placed 30cm from a concave mirror of 11 Which of the following electromagnetic waves has the
focal length 15cm. the linear magnification to the shortest wavelength? (a) radio waves (b) x-rays (c)
image produced is (a) 0 (b) 2/3 (c) 1 (d) 2 (e) 3 infra-red (d) blue light (e) ultraviolet
18 If a given mass of gas at constant pressure obeys the 12 Al the heat generated in a 50hm resistor by 2A
relation… volume is proportional to the absolute flowing for 30seconds is used to evaporate 5g of a
temperature… the gas is said to obey (a) Hooke’s law liquid at its boiling point. Which of the following is the
(b) Boyle’s law (c) Dalton’s law (d) Charles’ law (e) correct value of the specific latent heat of the liquid?
Newton’s law (a) 120J (b) 60J/g (c) 120J/g (d) 1500J (e) 1500J/g
19 A man standing between two parallel mirror in a 13 The speed of light in air is 3 x 108m/s. if the
barber’s shop will see the following number of his refractive index of light from air-to-water is 4/3, then
own image (a) eight (b) two (c) four (d) one (e) infinite which of the following is the correct value of the
20 A landlord has eight 40W electric light bulbs, four speed of light in water? (a) 4 x 108m/s (b)
8 -8 8
60W bulbs and two 100W bulbs in his house. If he 2.25x10 m/s (c) 4/9x10 m/s (d) 2.25x10 m/s
has all the points on for five hours daily and if PHCN 14 On which of the following is the operation of a moving
charges 5k per unit, his bill for 30 days is (a) N5.70 coil galvanometer based? (a) magnetic effect of
(b) N7.23 (c) N3.65 (d) N8.05 (e) N4050 electric current (b) electromagnetic induction (c)
chemical reaction produced by electricity (d) force on
PHYSICS 2006/2007 a current carrying conductor in a magnetic field (e)
1 Which of the following statements is correct? The electrical energy and power
reading of pressure on a mercury barometer is 15 A body is released from rest and allowed to fall freely
independent of (a) the cross-sectional area of the from a given height under gravity. The kinetic energy
tube (b) the atmosphere pressure (c) the acceleration at its halfway point is (a) a little above half of its initial
due to gravity (d) the temperature of mercury energy (b) a little below half of its initial energy (c) a
2
2 What are unit of thermal conductivity? (a) kg.m.s (b) half of its initial energy (d) a little above its initial
2 -1 -4 -1 -1 -1 -4 3
Js- m k (c) JK D, Ns m K (d) M energy (e) a little below its initial energy
3 Which of the following statements is correct? The 16 It is know that a neutron exists in a light atomic
magnetized state of an iron bar can be destroyed nucleus. Which of the following also exists in the
completely by (a) heating the bar to a high nucleus? (a) an electron
temperature (b) breaking the bar (c) connecting (b)  particle (c) an particle (d) a proton
opposite poles with a small piece of iron (d) placing 17 Radio carrier waves are generated by causing
the bar inside a strong magnetic field (e) none of the electrons to oscillate rapidly in (a) a microphone (b) a
above television camera (c) coaxial cables (d) an antenna
4 .An electric cell has an internal resistance of 20hns. A (e) a transmitter
current of 0.5A is found to flow when a resistor of 18 Which of the following statements is correct? Silvered
50hns resistance is connected across it. What is the walls of a vacuum flask are used to prevent (a) heat
electromotive force of the cell? loss due to conduction (b) vacuum loss (c) electric
(a) 5volts (b) 3.5volts (c) 2.5volts (d) 1 volt (e) 10volts into the inner flask due to convection (d) heat loss
5 The bulb of a motor-cycle headlamp is marked 40W, due to radiation (e) heat loss due to convection
6V. The resistance of the filament when it is switched 19 Which of the following is true of an electrical charge?
2 2 2
on is (a) 6 /40 ohm (b) 40/6 ohms (c) 40/6ohm (d) (a) positive charge means deficit of electrons (b)
2
6 x 40ohms (e) 6 /40ohms negative charge means excess of electrons (c)
6 In an electrolysis experiment, a cathode of mass 5g is electric current means movement of electrons (d) all
found to weight 5.01g after a current of 5a flows for of the above (e) A and B only
50s. what is the electrochemical equivalent of the 20 Which of the following statements are true in the
deposited substance? (a) 0.00004g/C (b) 0.00002g/C structure of atoms? (a) the atoms of the different
(c) 60/7cm (e) none of the above elements differ only in the number of protons and
7 A concave lens of focal length 20cm forms an image electrons (c) atoms are constructed according to the
¼ the size of the object. The object distance is (a) plan of our solar system (d) all of the above (e) A and
100cm (b) 100/9cm (c) 60cm (d) 60/7cm (e) none of C only
the above
8 A capacitor and a resistor are connected in series PHYSICS 2005/2006
with each other and in series with an accumulator of 1 The water at the bottom of a waterfall is slightly
negligible internal resistance. The potential warmer than that at the top because part of the (a)

© 2014 www.examsguru.net | All Right Reserved Enquiries: call +234 (0) 802 652 9647 Page 34
Examsguru.Net – LASU POST UTME PAST QUESTIONS WITH SOLUTIONS 2005 - 2014
potential energy raises the water temperature (b) (a) the temperature rises and the ice expands (b) the
water absorbs latent heat from the atmosphere (c) temperature remins constant and the volume of ice
kinetic energy is changed into internet capacity (d) decreases (c) the ice give soul heat as it melts (d) the
dropping mass of water increases the heat capacity mass remains constant and the temperature
(e) A and C above increases
2 A pressure cooker saves both time and fuel in (e) the mass decreases because some heat is lost to
cooking because inside the cooker the (a) boiling the atmosphere
point of water is raised (b) pressure is constant (c) hat 15 Four lenses are being considered for use as a
Is completely trapped (d) temperature is evenly microscope objective. Which of the following focal
distributed (e) volume of steam varies lengths is mot suitable?
3 Which of the following statements ins NOT true of the (a) -5mm (b) +5mm (c) -5cm (d) +5cm
image in a plane mirror? The image is (a) the same
size as the object (b) laterally inverted (c) virtual (d) CHEMISTRY 2010/2011
magnified (e) the same distance behind the mirror as 1 Which of the following substances is the most
the object is in front abundant in universe? (a) air (b) carbon (c) water (d)
4 Any line or section through an advancing wave in oxygen
which all the particles are in the same phase is called 2 The energy change (H) for the reaction;
the (a) wave length (b) wave crest (c) wave trough (d) CO(g) + 1/202(g)…..> CO2(g) is (a) -282.6Kj (b)
wave amplitude (e) wave front +282.6Kj (c) -503.7Kj (d) +393.3Kj
5 …………..to the characteristic behaviour substances 3 Which of the following group of physical properties
which set with their length, perpendicular to a strong increases from left to right of the periodic table? I.
magnetic field instead of in line with it is (a) lionization energy II. Atomic radius III.
magnetization (b) paramagnetism (c) diamagnetism Electronegative (a) I and II (b) I, II and III (c) III and
(d) dermagnetization (e) ferrormagnetism IV (d) I, II, III and IV
6 Which of the following statement on the use of X-rays 4 Which of the following solution will conduct the least
is incorrect? X-rays are used (a) in hospital to obtain amount of electricity? (a) 0.01maqueous solution of
photographs of tissue and bones in the body (b) for sugar (b) 0.01maqueous solution of NaOH (c)
the treatment of malignant growths like cancer cells 0.01maqueous solution of hexaonic acid (d)
(c) in detecting finger prints (d) to reveal hidden flaws 2.00maqueous solution of NaOH
in metal castings and welded joints (e) to direct
alterations which have been made to works of art. PHYSICS ANSWERS
7 What is the relative permittivity of a capacity if its 2010/2011 2008/2009 2009/2010
capacitance with a medium as dielectric is 16 farads 1 C 1 E 1 E
and its capacitance with vacuum as dielectric is 2 2 D 2 D 2 B
farads? (a) 1/8 (b) ½ (c) 2 (d) B (e) 32 3 C 3 E 3 C
8 If the magnificent of a virtual, image formed, plan 4 C 4 C 4 E
object 10cm from a convex lens is 3, the then focal 5 C 5 5 E
length of the lens. (a) 10cm (b) 15cm (c) 16cm (d) 6 C 6 C 6 E
10cm (e) 20cm 7 A 7 7 D
9 In the study of physics temperature and heat are 8 D 8 C 8 C
often confused with each other ? Which of the 9 B 9 9 A
following statement correctly defined these two 10 D 10 B 10 D
element? (a) temperature is a measure of beverage 11 D
kinetic energy of the molecules of a substance (b) 12 B
heat is measure of the total kinetic energy of the 2007/2008 13 A
molecules in a system (c) different materials required 1 D 14 D
different amounts of heat because a given charge in 2 B 15 2005/2006
temperature (d) all of the above (e) A and B only 3 C 1 C
10 A stand-by generators connected, to fifteen 40W 4 E 2 A
lamps and a musician’s 600W amplifying system. 5 A 2006/2007 3 D
How much energy is used if the generator runs for 6 B 1 A 4 E
6hours? (a) 3.87o (b) 7.20Wh (c) 8.40Wh (c) 7 C 2 C 5 C
12.00kWh (e) 14.56kWh 8 B 3 A 6 C
11 A tugboat is travelling from Asaba to Onitsha across 9 A 4 B 7 D
the river Niger with a resultant velocity of 20knots. If 10 D 5 E 8 B
the river flows at 12 knots, the direction of motion of 11 A 6 S 9 D
the boat relative to the direction of water flow is (a) 12 E 7 C 10 B
o o o o o
36.87 (b) 53.13 (c) 90 (d) 136.87 (e) 143.13 13 C 8 B 11 B
12 A milliammeter of resistance 5 and full scale 14 E 9 E 12 D
deflection of 50mA is to be used to measure a 15 C 10 E 13 C
potential difference of 50V. the resistance of the 16 D 11 B 14 B
multiplier should be (a) 950 (b) 1100 (c) 5050 (d) 17 C 12 C 15 B
9050 (e) 1005 18 D 13 B
13 An object of mass m and volume v is totally immersed 19 E 14 D
in a liquid of density p The tension of a string holding 20 A 15 C
it is (a) mg (b) (m+pv)g (c) (m-pv)g (d) pvg (e) mg/(m- 16 D
pv) 17 E
o 18 D
14 Heat is supplied to a block of ice at 0 C until half of
has melted. Which of the following statement is true? 19 D

© 2014 www.examsguru.net | All Right Reserved Enquiries: call +234 (0) 802 652 9647 Page 35
Examsguru.Net – LASU POST UTME PAST QUESTIONS WITH SOLUTIONS 2005 - 2014
20 B (b) heated gold (c) heated magnesium (d) calcium
hydroxide
5 Which of the following is true or sulphur? It (a) forms
two alkaline salts (b) is spontaneously flammable (c)
conducts electricity in the molten form (d) burns with
a blue flame
CHEMISTRY 2010/2011 6 In the Haber process for the manufacture of
1 Which of the following substances is the most ammonia, finely divided iron is used as (a) a
abundant in universe? (a) air (b) carbon (c) water (d) reducing agent (b) a catalyst (c) an ionizing agent (d)
oxygen an oxidizing agent
2 The energy change (H) for the reaction; 7 A molar solution of caustic soda is prepared by
CO(g) + 1/202(g)…..> CO2(g) is (a) -282.6Kj (b) dissolving
+282.6Kj (c) -503.7Kj (d) +393.3Kj (a) 40g of NaOH in 100g of water (b) 20g of NaOH in
3 Which of the following group of physical properties 500g of water (c)40g of NaOH in 1000g of water (d)
increases from left to right of the periodic table? I. 20g of NaoH in 1000g of water
lionization energy II. Atomic radius III. 8 The ratio of the number of molecules 16g of oxygen
Electronegative (a) I and II (b) I, II and III (c) III and to that of hydrogen is (a) 1:1 (b) 2:1) (c) 1:4 (d) 1:2
IV (d) I, II, III and IV 9 Sodum chloride may be obtained from brine by (a)
4 Which of the following solution will conduct the least sublimation (b) decantation (c) evaporation (d)
amount of electricity? (a) 0.01maqueous solution of distillation
sugar (b) 0.01maqueous solution of NaOH (c) 10. Which of the following acts as both a reducing and
0.01maqueous solution of hexaonic acid (d) an oxidizing agent (a) SO2 (b) HO2 (b) H2S (c) H2
2.00maqueous solution of NaOH (d) Co2
5 How will a metal X. which react explosively with air
and dilute acids be best extracted from its ores? (a) CHEMISTRY 2008/2009
electrolysis of used salt (b) decomposition of its 1 An emulsion paint consists of (a) gas or liquid
oxide (c) electrolysis of the solution of its salt (d) particles dispersed in liquid (b) liquid particle in quid
displacement from solution by an alkali metal (c) solid particles dispersed in liquid (d) solid particles
6 Which of the following sulphides is insoluble in dilute dispersed in solid
HCI? 2 A sample of orange juice is found to have pH3.80.
-4
(a) Na2S (b) Zn (c) FeS (d) Cus What is the concentration of (a) 1.6 x 10 (b) 6.3 x
-11 4 -11
7 Hydrogen is not librated when trioxonitrate(V) acid 10 (c) 6.3 x 10 (d) 1.6 x 10
reacts with zinc because (a) trioxonitrate V acid is a
strong acid (b) all nitrates are soluble in water (c) is 3 Which of these is an acid salt?
2
rendered passive by the acid (d) hydrogen produced (a) K SO4AI2(SO4)3.24H2O
is oxidized to water 4 Copper (II) tetraoxosulphate(IV) solution is
8 Given the molecular mass of iron is 56 and that of electrolyzed using carbon electrodes. Which of the
oxygen is 16, how many moles of iron (III) oxide will following are produced at the anode and cathode
be contained in 1kg of the compound. (a) 6.25moles respectively?
3
(b) 265.0 moles (c) 12.5moles (d) 3.125moles 5 A given mass of gas occupies 2dm at 3000K. At
9. The boiling points of water, ethanol, toluene and what temperature will its volume be doubled keeping
butan-2-ol are 373.0k,351, 383,6k and 372.5k the pressure constant? (a) 400K (b) 480K (c) 550K
respectively, which liquid has the highest vapour (d) 600K
4
pressure at 323.0k (a butan-2-ol (b) toluene (c) water 6 If 100cm of oxygen pass through a porous plug in
(d) ethanol 50seconds, the time taken for the same volume of
10 A man suspected of being drunk is made to pass his hydrogen to pass through the same porous plug is (a)
breath into acidified potassium dichromate solution. If 10.0s (b) 12.5s (c) 17.7s (d) 32.0s (O=16,H+I)
his breath carries a significant level of ethanol, the 7 Which of the following is a measure of the average
final colour of the solution is (a) pink (b) green (c) kinetic energy of the molecules of a substance (a)
purple (d) orange volume (b) mass (c) pressure (d) temperature
8 An increase in temperature causes in the pressure of
a gas in a fixed volume due to an increase in the (a)
number of molecules of the gas (b) density of the gas
CHEMISTRY 2009/2010 molecules (c) number of collisions between the gas
1 When pollen grains are suspended in water and (d) number of collision between the molecules and
viewed through a microscope, they appear to be in a the walls of the container
state of constant but erratic motion. This is due to (a) 9 Calcuate the mass in kilograms of magnesium
the bombardment of the pollen grain by molecules of produced by the electrolysis of magnesiums (II)
water (b) convection currents (c) small changes in chloride in a cell operating for 24 hours at 500
-1
pressure (d) small changes in temperature amperes (Faraday=96, 500C mmol , Mg=24)
2 sulphate(IV)ions are finally tested using (a) acidified (a) 2.7 9b) 5.4 (c) 10.8 (d) 21.7
barium chloride (b) acidified silver nitrate (c) acidified 10 Calaculate the volume of carbon(IV) oxide measure
lead nitrate (d) lime-water at s.t.p, produced when 1kg of potassium hydrogen
3 Sodium hydroxide (NaOH) pellets are trixocarbonate(IV) is totally decomposed by heat (a)
3 3 3 3
(a) hygroscopic (b) hydrated (c) deliquescent (d) 28dm (b) 56m (c) 112m (d) 196m
efflorescent
4 Nitrate can best be obtained from a mixture of oxygen [G.M/ V at S.T.P = 22.4dm3, K=39.O=16,
and nitrogen by passing the mixture over (a) heated C=12199=1]
phosphorus

© 2014 www.examsguru.net | All Right Reserved Enquiries: call +234 (0) 802 652 9647 Page 36
Examsguru.Net – LASU POST UTME PAST QUESTIONS WITH SOLUTIONS 2005 - 2014
st
11 The electronic configuration of an 1 1s22p62p63s3. 15 The reaction of potash with palm-oil to produce foam
how many unpaired electron are then in the element is referred to as (a) neutralization (b) saponification
(a) 5 (b) 4 (c) 3 (d) 2 (d) etherification (d) salting-out
12 Which of the following represents the type of bonding 16 The alkanol obtained from the production of soap is
present in ammonium chloride molecule? (a) ionic (a) ethanol (b) glycerol (c) methanol (d) propanol
only (b) covalent only (c) ionic and dative covalent (d) 17 The flame used by welders in cutting metals is (a)
dative covalent only butane gas flame (b) acetylene gas flame (c)
13 Which of the following is arranged in order to kerosene flame (d) acetylene flame
increasing electronegativity (a) chlorine, aluminium, 18 Consecutive members of an alkane homologous
magnesium, phosphorus, sodium (b) sodium, series differ by (a) CH (b) CH2 (c) CH3 (d) CnHn
magnesium, aluminium phosphorus, chloride (c) 19 Stainless steel is an alloy of (a) carbon, iron and lead
sodium, chlorine, phosphorus, magnesium, alumina, (b) carbon, iron and chromium (c) carbon, iron and
14 A quantity of air was passed through a weighted copper (d) carbon, iron and silver
mount of alkaline of the pyrogallol. An increase in the 20 The mass of an atom is determined by (a) its
weight of the pyrogallol would result from the ionization potential (b) its electrochemical potential
absorption of (a) nitrogen (b) neon (c) argon (d) (c) the number of protons (d) the number of neutrons
oxygen and protons
15 Which of the following gases can be collected by
upward displacement of air (a) NO (b) H2 (c) NH3 (c)
CI2 CHEMISTRY 2006/2007
1 Which of the following is physical change?(a) burning
CHEMISTRY 2007/2008 of kerosene (b) melting of ice cream (c) exposing
1 The cracking process is very important in the white phosphorus to air (d) dissolving calcium in
petroleum industry because it (a) gives purer water
products (b) yields more lubricants (c) yields more 2 Hardness of water is caused by the presence of the
engine fuels (d) yields more asphalt ions of (a) calcium and magnesium (d) sodium and
2 A gas that can behave as reducing agent toward potassium
chlorine and as an oxidizing agent toward hydrogen 3 Catalytic hydrogenation of benzene produces (a)
sulphide is (a) O2 (b) NO (c) SO2 (d) NH3 cyclohexene (b) oil (c) margarine (d) cylohexane
3 Sodium chloride can be obtained from brine by (a) 4 The shape of ammonia molecule is (a) trigonal planar
titration (b) decantation (c) distillation (d) evaporation (b) octahedral (c) square planar (d) tetrahedral
4 Which of the following conducts electricity? (a) 5 Which of the following substances can be used
sulphur (b) graphite (c) diamond (d) red phosphorus effectively as a water softener? (a) sodium
5 The movement of liquid molecules from the surface tetraborate (b) calcium bicarbonate (c) ferric chloride
of the liquid gaseous phase above it is known as (a) (d) calcuium silicate
Brownian movement (b) condensation (c) 6 Calcium hydroxide is added in the treatment of town
evaporation (d) liquefaction water supply to (a) kill bacteria in the water (b)
6 In the Haber process for the manufacture of facilitate coagulation of organic particles (c) facilitate
ammonia, finely divided iron is used as (a) an sedimentation (d) improve the taste of the water
oxidizing agent (b) a reducing agent (c) a catalyst (d) 7 Which of the following is used in the fire
a dehydrating agent extinguishers? (a) carbon (II) oxide (b) carbon (IV) (c)
7 An example of a neutral oxide is (a) AL2O3 (b) NO2 Sulphur (V) oxide (d) Ammonia
(c) CO2 (d) CO 8 What reaction takes place when palm-oil is added to
8 All alkalines (a) are all gases (b) have the general potash and foams are observed? (a) neutralization
formula (c) are usually soluble in water (c) four times (b) saponification (c) etherification (d) Salting-out
as fast as oxygen 9 The following the reagent frequently used in gas
analysis to absorb carbon monoxide is (a) cuprous
9 Hydrogen diffuses through a porous plug (a) at the chloride (b) calcium chloride (c|) pyrogallic acid (d)
same rate as oxygen (b) at a slower rate than oxygen sodium peroxide
(c) twice as fast as oxygen (d) four times as fast as 10 Of the following a metallic compound which is
oxygen increasing in importance as a lubricant is (a) graphite
10 The neutralization reaction between NaOH solution (b) iron oxide (c) molybdenum disulfide (d) lungstem
and nitrogen(IV) oxide(NO3) produces water and (a) oxide
NaNO2 and NaNO3 (b) NaNO3 and HNO3 11 The shape of the s-orbitat is (a) spherical (b) elliptical
11 A correct electrochemical series can be obtained (c) spiral (d) circular
from K, Na, Ca, AI, Zn, Fe, Pb, H, Cu, H, Cu, Hg, Ag, 12 Which of the following mixtures of gases is likely to
Au by interchanging born in flame? (a) neon and nitrogen (b) Helium and
12 When marble is heated to 1473K, another whiter noon (c) neon and hydrogen (d) nitrogen and helium
solid is obtained which reacts vigorously with water to 13 Pollution of underground water by metal ions is very
give an alkaline solution. The solution contains (a) likely in a soil that has (a) nitrate content (b) alkalinity
2
Ca(OH) (b) KOH (c) Mg(OH)2 (c) acidity (d) chloride content
13 Which of the following will react further with oxygen 14 In the production of soap, concentrated sodium
to form a higher oxide (a) NO and H2O (b) CO and chloride is added to (a) saponity the soap (b) increase
CO2 (d) CO2 and H2O the solubility of the soap (c) decrease the solubility of
14 When chlorine is passed into water and subsequently the soap (d) emulsify the soap
exposed to sunlight, the gas evolved is (a) HCL (b) 15 According to Charles law, the volume of a gas
HOCL (c) O2 (d) CI2O2 becomes zero at (a) -3730c (b) emulsify the soap (c) -
100oC (d) )oC

© 2014 www.examsguru.net | All Right Reserved Enquiries: call +234 (0) 802 652 9647 Page 37
Examsguru.Net – LASU POST UTME PAST QUESTIONS WITH SOLUTIONS 2005 - 2014
16 Carbohydrates are compounds containing carbon 14 The fitter in a cigarette reduces the nicotine contents
hydrogen oxygen in the ratio (a) 1:2:1 (b) 3:1:1 (c) by
2:1;1 (d) 1:1:1 (a) absorption (b) burning (c) absorption (c)
17 The modern process of manufacturing steel from iron evaporation
is by 15 A liquid that will dissolve fat is (a) hydrochloric acid
(a) blast reduction (b) oxidation (c) treatment with (b) kerosene ((c) water(d) calcium dioxides
acids (d) calcium tetraoxocarrbonate (IV)
18 The furring of kettles is cause the presence in water
of (a) 1calcium tetraoxicarbonate (b) calcium
hydroxide (c) calcium tetraoxosulphate (d) calcium
hydrogentrioxicarbonate (IV)
19 In the reaction E+ F <-> G + H, the background
reaction is favoured if the concentration of (a) G is
reduced reduce (b) F is increase (d) E is increased
(d) is reduced
20 When H2S is passed into a solution of iron(III)
chloride, the solution turns (a) pale green (b) plate red
,,
(c) colourless (d) brown

CHEMISTRY 2005/2006
1 Aluminum hydroxide is used the dyeing industrial as a
(a) dye (b) salt (c) dispersant (d) mordant
2 Carbon is tetravalent because (a) the 2s and 2p
atomic orbital are hybridized (b) dispersant (c) the
electrons in all the orbital of carbon are equivalent (c)
the electrons in all the oribital of carbon are
equivalent (d) all the atomic originals of carbonate
hybridized
3 atoms are best prepared by (a) cooling a molten
mixture of the metals (b) reducing a mixture of their
metallic oxides
4 Which of the following is found in cotton? (a) cellulose
(b) starch (c) fat (d) oil
5 The process by which atoms are arranged into
different molecular structure in the petroleum refining
process is referred to as (a) hydrocracking (b)
polymerization
(c) catalytic cracking (d) reforming
6 How many isomers does pentane have? (a) 4 (b) 6
(c) 5 (d) 3
7 Which of the following is an electrolyte? (a) alcohol
(b) solid potassium hydroxide (c) sodium acetate
solution (d) mercury
8 When sugar is dissolved in tea, the reaction is always
accompanied by (a) no entropy change (b) ositive
entropy change (c) negative entropy change (d) a
minimum entropy change
9 The commonest feature of reaction at the mode is
that (a) dissolved solute and the solvent (b) solvent
and undissolved solute (c) electrod dissolves (d)
oxidation is involved
10 A change in temperature of a saturated solution
disturbs the equilibrium between the (a) dissolved
and solved solvent (b) solvent and dissolved solute
(c) dissolved solute and undissolved solutes solvent
(d) dissolved solute and the solution
11 A consequence of global warming is (a) water
pollution (b) increased humidity (c) air pollution (d)
flooding
12 When cathode rays are deflected onto the electrode
of an electrometer, the instrument becomes (a)
positively charged (b) neutral (c) bipolar (d) negatively
charged
13 A gas exerts pressure on its container because (a)
the molecules of the gas collide with the walls of the
container (b) some of the molecules are moving
faster than others 9c) of the collision at the molecules
with each other (d) of the mass to the molecules of
gas.

© 2014 www.examsguru.net | All Right Reserved Enquiries: call +234 (0) 802 652 9647 Page 38
Examsguru.Net – LASU POST UTME PAST QUESTIONS WITH SOLUTIONS 2005 - 2014
CHEMISTRYANSWERS 23 A rhizome is an underground (a) root (b) shoot (c)
2010/2011 2007/2008 2006/2007 food (d) stem
1 A 1 C 1 B 24 Gymnosperms are commonly called (a) conifers (b)
2 A 2 C 2 A flowerless plants (c) seedless plants (d) cork bearing
3 C 3 D 3 D plants
4 A 4 B 4 A 25 Siliqua can be described as a (a) long narrow many
5 A 5 C 5 C seed developing from a superior bicarpellary ovary
6 D 6 C 6 D (b) dry monocarpellary, superior one-chambered fruit
7 A ` 7 D 7 B (c) dry monocarpellary multilocular fruits developing
8 A 8 C 8 B from a superior way (d) many seeded, unilocular fruit
9 D 9 C 9 C developing from a superior ovary
10 B 10 A 10 A 26 Blood platelets are important because they (a)
11 B 11 A produce antitoxins (b) antibodies (c) digest harmful
2009/2010 12 A 12 C bacteria (d) release
1 A 13 C 13 B 27 A tapeworm has no alimentary canal because (a) it
2 A 14 C 14 C does not feed (b) it sucks blood (c) its body adsorbs
3 C 15 B 15 D digested food (d) only liquid food is taken.
4 A 16 B 16 A 28 Cypsela is (a) dry, dehiscent, monocarpellary,
5 D 17 D 17 A developed from superior ovary (b) dry, dehiscent,
6 B 18 B 18 D multilocular, one chambered fruit (c) dry indehiscent,
7 C 19 B 19 D monocarpellary from superior ovary (d) dry,
8 D 20 D 20 A indehiscent, bicarpellary developed from inferior
9 C ovary one seeded fruit.
10 A 29 What is osmosis? (a) movement of solvent from high
concentration of solute (b) process of selective
2008/2009 2005/2006 transmission of solvent in preference to solute
1 B 1 C through semi-permeable membrane (c) movement of
2 B 2 B solute from high concentration of solvent to low
3 C 3 A concentration of solvent through semi-permeable
4 B 4 A membrane (d) movement of solvent from low
5 D 5 D concentration of solvent to high concentration of
6 B 6 A solvent through semi-permeable membrane.
7 D 7 C 30 In positive phototropism auxin (a) is produced more in
8 D 8 A the shade side than the illuminated side of the shoal
9 B 9 D )b) as an hormone circle on all the side of them.
10 C 10 A (c) is produced less in the shaded side than
11 C 11 C illuminated side of shoot (d) is produce on the plant
12 C 12 D side near the root so that it can grow longer.
13 B
14 D SECTION F: ECONOMICS
15 D 1 Economic can be defined as a science which deals
basically with (a) the demand and supply of goods
and services. (c) allocation of scale resources among
SECTION B – BIOLOGY competing ends (d) the determine or individual wants
16 The organelle in Amoeba which performs a similar 2 To the economist all things are said to be scarce,
function with that of the kidneys in vertebrate is the since by |scarce| simply means (a) expensive to
(a) nucleus (b) contractile vacuole (c) mitochondrion purchase (b) beyond the reach of the common man
(d) pseudopodium (d) limited in supply (e) not plentiful
17 Steady spread of the unpleasant odour from dead rat 3 The concept of choice in economics arises because
is a physical process called (a) cyclosis (b) (a) means exist only for satisfying some of people’s
diffusion(c) osmosis (d) plasmolysis wants (c) one has the opportunity to attain high
18 De-oxygenated blood from the head region first enter degree of cost minimization (d) it is always necessary
the heart through the (a) right auricle (b) right to make a choice
ventricle (c) left auricle (d) left ventricle 4 Give that servers draught affected an area where
19 Which of the following is not a characteristic of groundnut is grown. The effect of this on groundnut
enzymes? (a) they are proteins which are activated oil would be (a) a decrease in the demand for
by co-enzymes (b) they are specific and can act on groundnut oil (b) a decrease in the quantity supplied
specific substrates (c) they are organic catalysis (d) of groundnut oil (c) all increase in the quality
their reactions are irreversible demanded of groundnut oil substitute (d) a fall ill the
20 The best way of protecting natural resources is by (a) price of substitute
creating games and forest reserves (b) enforcing 5 Which of the following countries is NOT a member of
conservation laws (c) embarking on tree planting (d) the economics community of West African? (a)
prohibiting lumbering in forest reserves Nigeria (b) Benin (c) Libya (d) Togo
21 One basic characteristics feature of viruses is that 6 Which of the following are direct taxes? (a) sales (b)
they grow (a) as colonics (b) as jelly like materials (c) excise duties (c) income and company taxes (d) tariff
inside living cells (d) in culture media duties
22 Which of these is the function of pneumatopores in 7 Which of the following is a function of money? It is (a)
white mangrove plants? (a) anchorage (b) breathing PORTABLE (b) a standard for deferred payment (c)
(c) storage (d) excretion relatively stable in value (d) generally acceptable

© 2014 www.examsguru.net | All Right Reserved Enquiries: call +234 (0) 802 652 9647 Page 39
Examsguru.Net – LASU POST UTME PAST QUESTIONS WITH SOLUTIONS 2005 - 2014
8 The largest employment section in a typical West
Africa country is (a) construction (b) transportation (c) SECTION G – GEOGRAPHY
petroleum (d) agriculture 31 Lines joining places of equal sunshine duration on
9 The forgone alternative in economics is simply the (a) maps are known as (a) isohyet (b) isoeph (c) isobar
marginal costs (b) cost of forgone product (c) total (d) isohel
cost (d) opportunity 32 Which of the following countries is the leading
10 All the following are different form of money EXCEPT producer of petroleum (a) Nigeria (b) Libya (c) Egypt
(a) bank notes (b) demand deposits (c) coins (d) bank (d) Gabon
tellers 33 All the following constitute environmental hazards
11 The labour force in Nigeria falls between the ages of except (a) urbanization (b) mass wasting (c)
(a) 18 and 25 (b) 18 and 55 (c) 18 and 60 (d) 18 and earthquakes (d) drought
30 34 Which of the following is a feature of river deposition?
12 The difference between birth and death rates of a (a) Levee (b) screen (c) barcharn (d) morning
given population is referred to as the (a) mortality rate 35 Which of the following is not a metamorphic rock?
(b) fertility (c) natural grow rate (d) linear grow rate (a) grit (b) sdust (c) marble
13 A country’s import index by 1995 was 50 and her
index of export price was 70 calculate the terms of SECTION H – GOVERNMENT
trade (a) 20% (b) 8.71% (c) 120% (d) 140% 36 Fascism was a type of dictatorial government
14 The main handicap of sole proprietorships is (a) established in (a) Germany (b) Russia (c) Israel (d)
limited liability (b) lack of technical know-how (c) low Italy
profit margin (d) inadequate capital 37 Public opinion can be expressed through
15 More money charging for goods brings about (a) (a) census (b) general strike (c) socialization (d)
inflation (b) retail prices (c) deflation (d) speculative compromise
motive 38 power is different from authority because power lacks
(a) ability (b( sovereignty (c) probity (d) legitimacy
39 The activities of Nigeria in the international
SECTION G – GEOGRAPHY community are primarily influenced by (a) diplomacy
1. The largest producer of gold in the world is (a) (b) propaganda (c) national interest (d) military power
Gambia (b) Nigeria (c) South Africa (d) Zaire 40 The act of removing an elected official by the
2 The earth (a) rotates on its axis around the moon (b) electorate is referred to as (a) impeachment (b)
is inclined 66 on its orbital plane (c) is a satellite on consensus (c) recall (d) plebiscite
the sun (d) revolves round the sun in 365 days
3 Which of the following is NOT a product of river SECTION I – HISTORY
deposition? 41 Shake began his career as a warrior with the army of
a) food plain (b) Delta (c) levac (d) gorge (a) Dingiswayo (b) Ndwandwe (c) Zwide (d) Sobhuza
4 One of the effect of most urban ignition on the source 42 The last state of the Maghren to be conquered by the
region is (a) rural depopulation (b) urban Europeans was (a) Libya (b) Moroco (c) Algeria (d)
depopulation (c) rural congestion (d) urban Tunisa
congestion 43 The traditional rulers of Tunisia at the French
5 In which of the following …………….(a) igneous (b) colonization were called (a) Mai (b) Sultans (c)
plutonic (c) sedimentary (d) metamorphic Sheikhs (d) Bey
6. What would be the length of a two kilometre 44 Which of the following Yoruba states, did not
……………….. welcome the christian missionaries? (a) Egba (b)
(a) season pattern of rainfall (b) low nutrient value of Ijebu-Ode (c) Ibadan (d) Oyo
soils (c) high mean annual temperature (d) how 45 Which of the following place was a trading station in
temperature Benin Kingdom? (a) Udhoton (b) Wam (c) Ishan (d)
7 The work done in a gravitational ………….. Uzere
(a) the path taken (b) mass (c) acceleration due to
gravity (d) difference in height SECTION J – LITERATURE IN ENGLISH
46 “Pleasure is moth” is an example of (a) simile (b)
SECTION F – ECONOMICS irony
26 The sum of N80,000 is deposited in a bank and the (c) metaphor (d) hyperbole
cash ratio of the banking system is 10%. Calculate 47 Pick out the odd item from the following (a) plot (b)
the total sum of money the bank can create from the assonance (c) alliteration (d) rhythm
deposit (a) N100,000 (b) N800,000 (c) N700,000 (d) 48 One of the following is a narrative technique in novel
N850,000 (a) character (b) flashback (c) logical conclusion (d)
27 If budget deficits are financed by following, the interesting story
crowding out effect can be offset by an increase in (a) 49 Poetry involve one of the following (a) ideas only (b)
savings (b) interest rates (c) government expenditure beauty only (c) diction only (d) emotion only
(d) exchange 50 A sonnel is made up of a rhyming sestel and two
28 A tax that takes an interesting fraction of income goes hyming
down is called (a) conditional (B) regressive (c) (a) tercels (b) couplets (c) stanzas (d) quatrains
progressive (d) proportional
29 Arising short run average cost is a result of (a) failing SECTION K – ACCOUNTING
marginal cost (b) rising fixed costs (c) economics of 51 The group of users of accounting information
scale (d) diminishing return responsible for achieving the goals of the business is
30 The only factor of production that plays an active role it (a) employers (b) management (c) accountants (d)
in the production on process is (a) capital (b) labour investors
(c) entrepreneur (d) land

© 2014 www.examsguru.net | All Right Reserved Enquiries: call +234 (0) 802 652 9647 Page 40
Examsguru.Net – LASU POST UTME PAST QUESTIONS WITH SOLUTIONS 2005 - 2014
52 Assets with short lives which change form in the SECTION I
course of business is called (a) good assets (b) HISTORY
fictitious assets (c) current assets (d) fixed assets 41 -
53 Assets with short lives and which often change John 42 -
in the course of business is called (a) permanent 43 -
capital (b) owner’s capital (c) element assets (d) fixed 44 -
assets 45 -
54 A person to whom an entity is indebted is called (a) a
debtor (b) an investor (c) a liability (d( a creditor SECTION L
55 Given that opening Debtors figure is N23,000 sales COMMERCE
N96,000 and receipt from debtors N90,000 the 56 D
closing debtors should be (a) N17,000 (b) 29,000 (c) 57 C
119,000 (d) N209,000 58 A
59 D
SECTION L – COMMERCE 60 A
56 Before bill of exchange has any value, it must first be
(a) endorsed (b) discounted (c) documented (d) SECTION G
accepted GEOGRAPH
57 Who calculates the risks of losses for insurers? (a) 31 -
syndicates (b) jobbers (c) assessors (d) ucluaries 32 -
58 The difference between the jobber’s buying and 33 -
selling prices is known as his (a) turn (b) cut (c) profit 34 -
(d) premium 35 -
59 The value of the materials used in making notes and
coins is known as the (a) market value (b) face value SECTION J
(c) legal value (d) intrinsic value LIT. IN ENGLISH
60 What is the maximum number of peoples who can 46 C
loan a partnership? (a) 20 (b) 7 (c) 50 (d) unlimited 47 A
48 B
SECTION M - CHRISTIAN RELIGIOUS KNOWLEDGE 49 D
61 Gehazi’s request for Naaman’s talent silver and feslal 50 D
garments exposed his (a) evil plan to discredit his
maser (b) sense objectivity and fair play (c) greed and SECTION M
lust for materialism (d) consideration for rights C.R.K
62 The scroll which Ezekiel ate was (a) honey comb (b) 61 C
God’s covenant (c) God’s commandment (d) 62 D
Message of God 63 B
63 The companion of David in the Spy of King Saul in 64 B
the wilderness of Ziph was (a) Abner (b) Abishai (c) 65 A
John
(d) Caleb SECTION B
64 Paul argued in Galatians that the Jews were, until the BIOLOGY
coming of Jesus slaves to (a) the ignorance of their 16 B
parents (b) sin (c) death (d) the elemental spirits of 17 B
the universe 18 A
65 In 1 Corinthians, Paul says he is least of the Apostles 19 D
because he (a) was not one of the twelve (b) he 20 B
persecuted the church (c) was the last to be called by 21 C
Jesus (d) was an apostle of the gentiles 22 B
23 D
ANSWERS 24 A
25 C
SECTION B SECTION H SECTION K 26 D
BIOLOGY GOVERNMENT ACCOUNTING 27 C
6 D 36 D 51 B 28 D
7 C 37 B 52 C 29 B
8 B 38 D 53 C 30 A
9 B 39 A 54 C
10 - 40 C 55 B

SECTION F
ECONOMICS
26 B
27 C
28 B
29 D
30 B

© 2014 www.examsguru.net | All Right Reserved Enquiries: call +234 (0) 802 652 9647 Page 41
Examsguru.Net – LASU POST UTME PAST QUESTIONS WITH SOLUTIONS 2005 - 2014
SECTION B – BIOLOGY 5 Which of the following advantages of localization of
1 Blood clotting is initiated by (a) leucotytes (b) platelets industries may also be disadvantages? (a) external
(c) beamolymph (d) haemoglobin economies (b) concentration of industries and people
2 Anaerobic respiration in yeast produces (a) carbon (c) development of organized markets (e) reduced
dioxide and ethanol (b) carbon dioxide and water (c) cost of research
carbon dioxide and oxygen (d) carbon dioxide and 6 The Nigerian Bank for commercial and industry is (a)
glucose a commercial bank (b) development bank (c) an
3 Which of the following is NOT regarded as a pollution industrial bank (d) a merchant bank
on hand and in air? (a) noise (b) smoke (c) sulphur 7 To control inflation, the monetary authorless of a
(d) carbon monoxide country call (a) reduce taxes (b) advise government
4 Denitrifying bacteria in nature liberate gaseous to increase its expenditure (c) engage in expansive
nitrogen directly from (a) ammonium salts (b) soil monetary policy (d) engage in restrictive monetary
nitrates (c) thunderstorms (d) soil nitrites policy
5 The appendicular skeletons is made up of the (a) 8 Which of the following is used by the central bank to
limbs (b) skull and limbs (c) phalange (d) girdles and control the rate of interest (a) bill of exchange (b)
limbs banker’s order (c) fixed deposit account (d) open
6 Germination which results in the cotyledon’s being market operation
brought about ground is called (a) hypocotyls (b)
epicotyis 9 Gresham’s law in economics shows that
(c) epigeal (d) hypogcal (a) bad money drives good money out of circulation
7 A relationship between living organisms which is of (b) good money drives bad money out of circulation
mutual benefit is (a) parasitizing (b) saprophytism (c) (c) gold must be available to maintain the value of
ecosystem (d) symbiosis paper money
8 In a mammal stimulus is transferred from the receptor (d) the price level varies directly with the quantity of
from the receptor muscle to the central nervous money
system through the (a) motor neurons (b) effectors 10 Given that the cash reserve ratio is to percent what is
muscles (c) dendrites (d) sensory enormous the maximum amount of money that the banking
9 An onion is a bulb because it (a) has a tuberous stem system can create from an initial cash deposit of
(b) has a reduced stem and thick fleshy leaves (c) N10,000.00 (a) N100.00 (b) 1,000.00 (c) N9,000.00
has adventitious roots (d) bears many buds at the (d) 10,000.00
nodes 11 The most important economic characteristic
10 The main function of blood in mammals is to (a) differentiating commercial banks from other financial
excretory materials from tissues (b) carbon dioxide institution is their
from lungs to tissues (c) oxygen to the lungs (d) (a) function as a safe and profitable store place for
digested food from all the body tissues savings (b) role in financing balance of payments
11 Meiosis occurs during the (a) abnormal division of deficits (c) status as an intermediary between savers
cancer cells (b) animal cell division (c) plant cell and borrowers (d) ability to create and destroy money
division (d) formation of haploid cells 12 Deflation Isa persistent fall in the general price level
12 Of the following, the blood cells which are not and is usually caused by (a) reduction in total
manufactured in the red bone narrow are the demand (b) an increase in government spending (c)
……………….. an increase in the money supply (d) an increase in
aggregate demand
ECONOMICS 13 Which of the following causes of inflation is related to
1 In a public company, shares are (a) sold to one demand pull inflation? (a) low productivity on farms
person only (b) distributed freely (c) advertised to and in firms (b) poor storage facilities (c) poor
members of the public for subscription (d) disposed of distribution system (d) increase in government
the chief executive expenditure on construction
2 Which of the following is a characteristic of a private 14 Which of the following is NOT a goal of modern
limited liability company? (a) 115 shares can be sold budgets? (a) the control of inflation (b) the reduction
to the public (b) the number of share-holders ranges in come inequality (c) the shift of all resources from
from fifty to one hundred (c) all the share holders the private to the public sector (d) economic
have equal powers and responsibilities (d) the development
number of shareholders ranges from two to fifty 15 The best technical description of an progressive tax is
3 One of the advantages of a partnership over a sole a (a) tax which takes money from the rich than from
proprietorship is that (a) it is the most popular form of the poor (b) more equitable tax than a regressive one
business organization (b) the partner can easily (c) tax which takes higher proportion of extra naira
withdraw from the business (c) II make an increase in received as income rises (d) tax which falls directly on
the capital of the business possible (d) there is no those in the high income brackets
limit to the number of people, who may bring in
capital. CHRISTIAN RELIGIOUS KNOWLEDGE
th
4 Which of the following is an external economy derived 1 According to genesis chapter 1, on the 6 day, God
by a firm? (a) low cost opportunities enjoyed by being created (a) the firmament (b) light (c) dry land and
in a place where other producers concentrate (b) sea (d) man and living creature (e) the sun, moon and
technical economics enjoyed by varying the factors of stars
production (c) economies of management by putting 2 The flood lasted while Noah was in the ark for (a) two
administrators where they are most efficient (d) weeks (b) ten weeks (c) ten days (d) a hundred days
marketing advantages attained through preferential (e) a hundred and fifty days
treatment in the purchase and distribution

© 2014 www.examsguru.net | All Right Reserved Enquiries: call +234 (0) 802 652 9647 Page 42
Examsguru.Net – LASU POST UTME PAST QUESTIONS WITH SOLUTIONS 2005 - 2014
3 Walk before me and be blameless. How old was the parties, the executive and the judiciary
addresses when this voice came? (a) 75years (b) 91 (c)government and the legislative
years (c) 99years (d) 105 years 9e) 120 years 12 The three fundamental rights of citizens are
4 You would bring down my grey hairs with sorrow to (a)salvation, properly, freedom of thoughts (b)
sheol, Jacob said this to his children because (a) employment, property and social security (c) life,
Joseph was unjustly imprisoned (b) Joseph bad been liberty and property (c) life, liberty and property (d)
reported killed (c) the famine was very sever (d) the free education peaceable assembly and freedom of
Ishmaeliles had brought Joseph and had resold him thought (e) freedom of movement, associating and
to Potiphar religion
5 And the magician said to Pharaoh. This is the finger 13 In a democracy, franchise is given to all (a) resident
of God. This statement was made after the plague of adults (b) citizens (c) citizens except members of the
(a) frogs (b) blood (c) locust (d) boils (e) gnals armed forces (d) loyal party member (e) qualified
adult citizens
GOVERNMENT 14 The citizenship of a country may be acquired by an
1 In a presidential system of government the executive individual through (a) decolonization (b) nomination
(a) executes its own laws only (b) legislates all (c) nationalization (d) neutralization (e) naturalization
binding laws (c) make laws for the national assembly 15 Political authority is vested in the (a) state (b)
(d) forms the government (e) executes all legal judiciary
government plotters (c) government (d) armed forces
2 A constitution is a legal document
(a) drawn up by lawyers (b) enacted by military COMMERCE
decree 1. The rate at which a country’s exports exchange for its
(c) forming the basis upon which a government rules imports is called (a) balance of payments (b) balance
the country (d) which must not be altered by any of trade (c) terms of payment (d) terms of trade
succeeding government (e) only likely to succeeded 2 The difference between the total payments for
in a country where there is union government imports and the receipts from exports within a given
3 One features of a totalitarian state is the existence of period is referred to as (a) balance of payments (b)
(a) a single recognized party (b) pressure groups (c) balance of trade (c) comparative cost advantage (d)
opposition groups (d) a colonial power (e) fierce comparative advantage
political rivalry 3 A pro forma invoices is NOT required when (a)
4 A system in which a row powerful and rich nobles quoting for the supply of goods (b) goods are sent on
own land which is hired out to that poor people to approval (c) dealing regularly with a customer (d) final
farm is called prices are uncertain
(a) feudalism (b) co-operatives (c) socialism 4 Which of the following statements is TRUE about sea
(d) communism transport? (a) all ships have specific routes to ply
5 The principal checks and balances is necessary (b) all ships must be insured at every journey
because it (a) prevents government from becoming (c) All ships are insured separately from the crew
dictatorial (b) prevents the executives from (d) all ships normally arrive at their destination with
functioning (c) makes the executives stronger than goods
the other organs (d) make the three organs hate each 5 The postal organization which proceeded in Nigerian
other (e) leave each organ of government postal service is (a) post and telecommunications (b)
independent of the judiciary post and telephone (c) post and teleprinter (d) post
6 When a constitution is difficult to amend we say it is and telegraph
(a) federal (b) unity (c) written (d) fragile (e) rigid 6 Goods discharged from ships on which import duties
7 A political authority which maintains sovereign power have not been paid are kept in the (a) ordinary
over a specific geographical area is termed (a) the warehouse (b) public warehouse (c) private
nation (b) the nation-state (c) the state (d) nationalism warehouse (d) bonded warehouse
(e) imperialism 7 A sole proprietor insured his good worth N200,000 for
8 A government in which control of ultimate power is by half the value. There was a fire incident in which
a few who rule in their own selfish interest is goods valued at N800,000 were destroyed. The
classified as (a) a democracy (b) a dictatorship (c) an value of compensation expected from the insurer is
aristocracy (d) a monarchy (e an oligarchy (a) N40,000 (b) N80,000 (c) 100,000 (d) 200,000
9 The most basic property of pressure to groups which 8 One of the effects of the devaluation of the naira is
differentiates them from political parties is that they that the (a) goods that can be imported with the Naira
(a) are not as interested in politics (b) do have have become cheaper (b) value of other currencies
permanent organizations (c) do not seek to influence relative to the naira have become cheaper (c) goods
public opinion (d) do not support candidates in that can be exported from Nigeria have become
elections (e) do not nominate candidates as their own costlier (d) imports that can be bought with the naira
official representatives have become costlier
10 In a presidential system of government ministries are 9 Drawing two parallel lines across a cheque means
(a) collectively responsible to the senate (b) that is has to be paid into the account of the (a) payee
collectively responsible president (c) individual (b) drawer (c) drawee (d) payer
responsible to the president (d) individually 10 The financial institution established to boost Nigeria’s
responsible to no one trade with the rest of the world is called (a) Nigerian
(e) individually and collectively responsible to the deposit insurance corporation (b) Nigerian economist
electorate reconstruction fund (c) Nigerian industrial
11 The three principal organs of government are the (a) development bank (d) Nigerian export import bank
legislature, the public service and judiciary (b) political 11 Which of the following is both direct and indirect credit
enterprises? (a) co-operative and thrifts society (b)

© 2014 www.examsguru.net | All Right Reserved Enquiries: call +234 (0) 802 652 9647 Page 43
Examsguru.Net – LASU POST UTME PAST QUESTIONS WITH SOLUTIONS 2005 - 2014
retail-cooperation society (c) consumer cooperative 3 Which of the following protein may not be associated
society (d) wholesaler cooperative society with transport of oxygen in organisms? (a)
12 The business in the stock exchange is characterized Haemoglobin (b) Haemocrythin (c) chlorocruorin (d)
essentially by (a) dealing (b) brokerage (c) Prothrombin
speculation (d) transactions 4 Which of the following is not required for primary
13 The second –tier securities market is (a) an production to take lace? (a) sugar (b) chlorophyll (c)
appendage to the Nigerian stock exchange (b) a water (d) light
member of the Nigeria stock exchange (c) meant for 5 In normal blood smear which of the following is
the shares of the quoted private companies (d) a expected to be least in number (a) moncytes (b)
regulating arm of the Nigeria stock exchange erythrocytes (c) platelets (d) basophils
14 The elements of the marketing mixture are (a) 6 Which one of the following is not a detritivore in a
product, promotion, place and price (b) personal strict sense? (a) woodlice (b) locust (c) tubeworm (d)
selling, advertising and research (c) promotion, earthworm
production, place and price 7 Which one of these is not a direct effect of osmosis
(d) procurement, product and place (a) haemolysis (b) crenation (c) homeostasis (d)
15 Which of the following items has the shortest effect turgor
on the consumer? (a) fashion (b) innovation (c) fad 8 The blood vessel labelled D in figure I is?
(d) attribute (a) superior vena cava (b) inferior vena cava (c) aorta
(d) renal artery
ACCOUNTING
1 The act establishing the Institute of Chartered
Accountants of Nigeria(ICAN) came into force on (a)
st st st
1 September, 1960 (b) 1 October, 1960 (c) 1
st
October, 1963 (d) 1 September, 1965
2 Cost reports for attention of management should
reflect (a) as much details as possible (b) summary Figure 1. The block arrows show direction of
figure only (c) details of non-controllable expenses (d) blood flow.
cost and comparable data useful in decision making 9 If a flower is considered as the modified system of the
3 If the inventory at the end of the current is understand plant (a) pedical (b) receplacle (c) floral parts (d)
and the error is not caught during the following year, anther
the effect is to (a) understate income this year and 10 Which of the following activities may bring along heat
understate in next year (b) overstate income this year losses in an animal? (a) liver metabolism (b) muscle
and overstate in next year (c) understate this year’s activity (c) cold food ingestion (d) aerobic respiration
income with no effect on next year’s (d) overstate the 11 All members of the same species found in a particular
income for the two year period community can best be described as a (a) Niche (b)
4 The total of the discounts received column in the cash Ecosystem (c) population (d) family
book is posted to the (a) credit of the discounts 12 How many of the 46 chromosomes found in the adult
received account (b) debit of the discounts allowed human male body cells will be called sex
account (c) credit of the discounts allowed account chromosomes
(d) debit of the discounts received account (a) none (b) 2 (c) 44 (d) 1
Use the information blow to answer question 5 and 6.
st
The initial imprest as at July 1 was N500. Petly cash
th
vouchers with the custodian by July 15 added up to
N394. an IOU slip received from a coworker was N65 and
there was a shortage of N5 cash. The coworker made 13 Which of the following ABO blood genotypes show
th A o A B o o B B
refund on July 18 before the imprest was replenished co-dominance (a) 1 1 (b) 1 1 (c) 1 1 (d) 1 1
th
14 The diagram (figure 2) shows a young plant cell.
5 What was the actual cash in the till as at 15 July? Which of the membranes in the diagram is freely
(a) 106 (b) 1010 (c) N70 (d) N30 permeable?
6 The amount required to replenish the payment made (a) D only (b) B and D (c) D and E (d) none of the
from the imprest was (a) N399 (b) N394 (c) 358 (d) above
N101 15 During insect metamorphosis most growth is seen in
7 State bank collected a note for Al-Makura company. the (a) egg (b) larva (c) imago (d) pupa
This collection, not yet recorded in AL-Makura’s
books, appears on the bank reconciliation as (a) an HISTORY
addition to balance per books (b) a deduction from 1 The Governor of the Western Region who died in a
balance per bank statement military coup with the visiting Head of state in 1966
(c) an addition to balance per bank statement (d) a was (a) Lil. Col B.S. Dimka (b) Col. Shittu Alao (c) U.
deduction from balance per books Col. Adekunle Fajuyi (d) Col Ibrahim Taiwo
2 The symbols of Asante unity in pre-colonial time was
BIOLOGY (a) the Goklen stool (b) the sacred python (c) Efua
1 The ribosomes for protein synthesis are rock (d) the Ho River
manufactured by? (a) nucleus (b) nucleolus (c) 3 Which of these was not in the Western Sudan? (a)
nucleoplasm (d) cytoplasm Gao (b) Timbuku (c) sankore (d) Dares salam
2 Which of the following colours of precipitates is not 4 The legitimate trade introduced in the nineteen
associated with positive responses of reducing century mainly involved (a) trade in agriculture
sugar? products (b) trade in animal products (c) slave barter
(a) Green (b) yellow (c) red (d) purple (d) domestic trade

© 2014 www.examsguru.net | All Right Reserved Enquiries: call +234 (0) 802 652 9647 Page 44
Examsguru.Net – LASU POST UTME PAST QUESTIONS WITH SOLUTIONS 2005 - 2014
5 The popular means of transport during the trans- 5 A requiem is …………………(a) suite (b) organ work
Saharan trade was the (a) donkey (b) horse (c) camel (c) mass for the dead (d) worship song (e) piano
(d) mule pieces
6 A Aro became famous far andbeyong Igboland 6 ,,,,,,,,,,,,,is composed the Nigeria national anthem
because (a) Olusoji (b) Odujobi (c) Odiase (d) Oikiolme (e)
(a) they were gold traders (b) their kings were the Ajani
most famous in all Igboland (c) they medicated in all 7 ………..composed the water music
disputes through their long juju (d) none of the above (a) Mozart (b) Hayden (c) Gluck (d) Handel (e) Purcel
7 The firs ruling dynasty in Benin was the (a) Ogiso 8 Oliver de Coque was a ……………musician
dynasty (b) Achadu dynasty (c) Eweka dynasty (d) (a) Funk (b) R & B (c) Highlife (d) Goje (e) Juju
Onogie dynasty 9 Kakaki was mostly found in the …………….of Nigeria
8 Which of these were involved in the battle of Adowa? (a) East (b) Niger (c) Middle Belt (d) North (e) River
(a) Ethiopia and Italy (b) Sudan and Egypt (c) Sudan Belt
and Fanco (d) Egypt and Ethiopia 10 Atilogwu is of the (a) Kannri (b) Ebira (c) Karaje (d)
9 Which was the general purpose currency in pre- Igbo (e) Ishan
colonial Nigeria? (a) Cloth (b) Salt (c) Copper (d) 11 The Ayans are families of ………………..(
Cowry (a) drummers (b) healers (c) flute makers (d)
10 General Murtala Muhammed was assassinated in a blacksmiths (e) instrument repairers
coup led by (a) Lt. Co. Kaduna Nzeoguw (b) Col. Joe 12 Baba drums are used by the ……………
Garba (a) Hausas (b) Binis (c) Tapas (d) Nupes (e) Yorubas
(c) Lt. Col. B. S. Dimka (d) Major Gideon Orkar 13 God save the Queen is the national anthem of
11 Abuja became the federal capital of Nigeria in (a) (a) Queensland (b) Scotland (c) United Kingdom (d)
1907 (b) 1991 (c) 1993 (d) 1995 the Victorian Era (e) none of the above
12 The first ruler of the Mali empire was (a) Saif (b) 14 Salawe Abeni is a …………..Musician
Sumanguru Kanite (c) Sundiate (d) Sunni Ali (a) Juju (b) Fuji (c) Pankeke (d) Sewele (e) Waka
13 The twenty one state structure came into being in 15 “If music be the rood of love, then play on” is from
Nigeria during the rule of (a) general Murtala (a) Shakespeare (b) Bible (c) Koran (d) Hardy chase
Muhammad (b) Major General Aguiy Ironsi (c) (e) Wordsmith
General Olusegun Obansanjo (d) General Ibrahim
Babnagida LITERATURE IN ENGLISH
14 The main cause of the 1962/63 census crisis was that 1 A poem rhyming in five times is called
(a) inexperienced officers were used for the (a) Alogomachy (b) Limerick (c) Monostich (d)
enumeration (b) the army participated in the exercise Monody
(c) the police prevented the people from being 2 Which of the following is a likely source of traditional
counted (d) there was widespread in inflation of oral poetry(a) theatre (b) songs (c) Novels (d) drams
census figures in many parts of the country 3 Mime is the art of acting that involves the use of (a)
15 The Jamiya Murtanem Arewa Cultural group evolved songs only (b) speeches only (c) gestures only (d)
into (a) Northern Elements Progressive Union dances only
(NEPU) Read the following and answer the question that
(b) United Middle Belt Congress (UMBC) follow
(c) Northern People’s Congress (NPC) Look!
(d) Nigerian National Democratic Party (NNDP) Look our there
In the bucket
VISUAL ART The rusty bucket
1 Which of these is true about primary colours With water unclean
(a) the cannot be gotten by mixing any other colour Look
(b) they can be gotten by mixing 3 different colours Aluminous plate is floating
(c) they can be gotten by adding water to the colours The moon dancing to the gentle night wind
that are to be mixed (d) they are not primary colours Look! All you who shout across the wall with a million
2 One of these is a chief exponent of the Art movement hates look
known as cubism (a) Michelangelo (b) Paul Zezanne at the dancing
(c) Henri Matisse (d) Pablo Picasso (c) Georges It is peace unsoiled by the murk
Braque And dirt of this bucket war
3 The word ‘torso’ means (A) …………………
4 The wall in line 9 refers to (a) the speaker’s feelings
MUSIC (b) barrier of hatred erected b y people (c) a physical
1 The guitar is a ………………..instrument (a) string (b) wall (d) the wall of love
wind (c) percussive (d) horn 5 The beginning of this poem is (a) monotonous (b)
2 The surprise symphony was composed by (a) dramatic (c) reflects the moon (d) repetitive
Beethoven (b) handel (c) hayden (d) chopin (e) field 6 The poem demonstration the poet’s keen sense of
3 Mime is the art of acting that involves the use of (a) (a) observation and descriptive ability (b) reflection
songs only (b) speeches only (c) gestures only (d) the moon (c) reflection on the peace around him (d)
dances only love for nature
4 “I know my Redeemer liveth” is an extract from 7 The poet uses the moon as a symbol of (a) light (b)
………… peace (c) love (d) hatred
(a) symphony (b) organ work (c) mythology (d)poems 8 The omniscient narrator (a) uses flashback (b) has a
(e) opera scientific knowledge of the story (c) knows all about
characters and events (d) refers to futures event

© 2014 www.examsguru.net | All Right Reserved Enquiries: call +234 (0) 802 652 9647 Page 45
Examsguru.Net – LASU POST UTME PAST QUESTIONS WITH SOLUTIONS 2005 - 2014
9 Penny wise, pound foolish is an example of (a) 6 Which of the following will not help solve the problem
paradox (b) metaphor (c) analogy (d) oxymoron of overpopulation? (a) birth control (b) reduction in
area of agricultural land (c) increase in food
Read the following extract and answer question 10-15 production (d) limiting the size of towns
Never has the death of a poet 7 Which of the following explains why the USA has
Been tolled by all the world more large towns than West Africa? (a) the USA is
God’s work on the earth through more highly industrialized than West Africa (b) the
Has its universal funeral in the west USA has temperate climate while West Africa has hot
Recurrent grave of day’s almighty soul climate (c) the soil in the USA are richer than those in
West Africa (d) there are more mining centres in the
Never was victory so trumpeted USA than in Africa
As that of the sun scorching his fiery way 8 Synthetic rubber is made from chemical and is
And then in gorgeous colours failing becoming more important especially in (a) Britain (b)
Trailing stars USA (c) Malaysia (d) Iran
Life, death, water and aridity 9 Which of the following group of industries consists
Bow to his morning ray only of light industries? (a) ship building, locomotives,
chemical (b) locomotives, petroleum, textiles (d)
With his passing, death, stirs in the thicket paper furniture clothing (d) printing flour milling, ship
In church the tells tolled building
In barracks at the last bugle note 10 Which of the following does not contribute to the
Soldiers like and file world production of wine? (a) France (b) California
Drunkards like sick dogs retch homewards (c) Norway (c) Australia
The night voce is a harsh guitar 11 Which of the following is not essential for the intense
cultivation of wet rice? (a) fertile soils (b) dependable
But on a hill among musizi trees supply of water (c) level land (d) tropical monsoon
Sweet nuns sing the litanies climate
Of the origin whose sun we know 12 Which one of the following best as described the type
Priest like lamp posts in a graveyard of rainfall that occurs in West Africa? (a) convectional
Stoop over the breviary (b) frontal (c) cyelogenic (d) orographic
13 All the crops cotton, sugarcane, rubber, oil palm and
There’s piping of crickets in the bush coffee are usually associated with (a) estate
And bellowing of frogs agriculture (b) mechanical cultivation (c) intensive
All sing the ancient elegy farming (d) rotational cultivation
For the sum that has died in the west 14 The readings on a wet and dry bulb thermometer will
be the same when (a) there is a lot of moisture in air
10 The opening of the poem is a tribute to (a) the sun (b) (b) there is little or no moisture in air (c) the air
the dead (c) the poet (d) crickets in the bush contains no moisture (d) the air is saturated with
11 The poem can be regarded as (a) an elegy (b) a moisture
nature poem (c) poem about death of a poet (c) poem 15 Dairy farming in Denmark is very successful mainly
about the world (d) poem about the world because (a) Denmark has good soils (b) the climate
12 Death in the poem is (a) eulogized (b) personified (c) is suitable for growing grass (d) Denmark has
scolded (d) extolled excellent dairy cattle
13 The first two stanzas are concerned with (a) the
sound of a guitar (b) the sounds at night (c) the sound
of a bell (d) the sound of music
14 In line 18 image of a harsh guitar refers to (a) the SECTION E
sound of a guitar (b) the sounds at night (c) the sound MUSIC LIT. IN ENGLISH
of a bell (d) the sound of music 1 A 1 B
15 Day’s almighty soul in line 5 refers to (a) a person’s 2 A 2 B
soul (b) the sun (c) the time of great activities in a day 3 D 3 C
(d) the most important time of the day 4 A 4 D
5 C 5 C
GEOGRAPHY 6 A 6 A
1. Basement rocks refer to rocks that (a) form core fold 7 C 7 A
mountains (b) are never seen on the surface (c) form 8 C 8 C
a continental platform on which other rocks have 9 D 9 A
formed (d) form at the base 10 B 10 C
2 Exactly half of the earth’s surface has light at any 11 A 11 A
particular time, because the (a) sun is overhead at 12 E 12 C
the equator (b) earth’s axis is tilled at an angle of 66½ 13 B 13 D
to the earth’s orbital (c) earth rotates on its axis 14 C 14 B
3 The rotation of the earth is NOT the cause of (a) the 15 A 15 A
seasons (b) day and night (c) the deflection of ocean
currents (d) the deflection of the winds
4 An intrusion of magma along a bedding plain is called
(a) an overfold (b) sill (c) dyke (d) volcanic plug
5 Which of the following features occurs in a glaciated
low-land region? (a) Esker (b) Arcete (c) Hanging
valley (d) crique

© 2014 www.examsguru.net | All Right Reserved Enquiries: call +234 (0) 802 652 9647 Page 46
Examsguru.Net – LASU POST UTME PAST QUESTIONS WITH SOLUTIONS 2005 - 2014
6 A
7 A
2008 ANSWERS 8 B
SECTION B SECTION G 9 C
ECONOMICS BIOLOGY 10 D
1 C 1 D 11 D
2 D 2 A 12 B
3 C 3 D 13 C
4 A 4 A 14 C
5 B 5 A 15 D
6 C 6 B
7 D 7 B
8 D 8 C
9 - 9 C
10 C 10 C
11 C 11 C
12 A 12 B
13 D 13 B
14 C 14 A SECTION C – COMMERCE
15 A 15 C 31 What will ships be unloaded on to if the number of
berths at a poll is inadequate? (a) steamers (b)
SECTION C SECTION H tramps (c) lighters
C.R.S VISUAL ART (d) containers ships
1 D 1 A 32 What would a bull do? (a) sell shares in anticipation
2 D 2 - of a fall in price (b) subscribe for new issues (c) buy
3 C 3 D shares in anticipation of a price rise (d) buy and sell
4 C shares on commission for other people
SECTION D 5 A 33 A delcredere agent (a) has unrestricted powers (b)
GOVERNMENT 6 D arranges credit terms (c) guarantees payment by the
1 D 7 D buyer (d) arranges insurance
2 C 8 D 34 The ratio of capital owned to other liabilities in a
3 A 9 D business is known as (a) efficiency ratio (b) liquidity
4 A 10 E ratio (c) gearing ratio (d) rate of return
5 A 11 E 35 Mr. Chukwuma took up a fire insurance on a properly
6 E 12 E valued at N1000 and the amount insured is N800. the
7 C 13 C properly caught fire causing a loss of N400, if the
8 E 14 - insurance was taken with the clause with average,
9 E 15 D what is the amount to be paid by the insurance
10 B company? (a) N420 (b) 320 (c) N460 (d) N570
11 D 36 The computer using linear integrated circuit
12 C technology coupled with quantification of data in term
13 E of length and distance is known as (a) analogue
14 E computer (b) hybrid computer (c) digital computer (d)
15 C mainframe computer
37 if goods or “returnable” are returned the seller issues
SECTION E a (a) debit note (b) consignment note (c) advice note
COMMERCE (d) credit not
1 D 38 The Jobber’s Turn is (a) his commission for arranging
2 A dealings in sell share (b) the price at which he is
3 C prepared to sell shares (c) the different between his
4 A buying and selling prices (d) the price at which he is
5 A prepared to buy shares
6 D 39 If duty has been paid on goods, and they are later
7 A exported, what can the exporter claim? (a) drawback
8 D (b) tax exemption (c) demurrage (d) a bond
9 C 40 The capital employed is (a) W20,000 (b) W19,000 (c)
10 D W28,000 (d) W31,000
11 A
12 C
13 A
14 A
15 D
SECTION F
ACCOUNTING
1. A
2 D
3 -
4 -
5 D

© 2014 www.examsguru.net | All Right Reserved Enquiries: call +234 (0) 802 652 9647 Page 47
Examsguru.Net – LASU POST UTME PAST QUESTIONS WITH SOLUTIONS 2005 - 2014
Use the information below to answer question 41 & consumption increase, if investment increases by
42 N300? (a) W100 (b) W200 (c) W600 (d) W900
The following information was obtained from the books of 51 Which of the following is a normative statement? (a) a
st
Johnson on 31 December, year 2000. more equal distribution of income would increase the
N rate of economic growth (b) a more equal distribution
Stock 31/12/2000 2,500 of income would raise the average propensity to
Rent 500 consume (c) a more equal distribution of income
Insurance 300 would be inflationary (d) more equal distribution of
Lighting expenses 500 income would increase national welfare.
Motor expenses 500 52 An employer currently employs 200 workers. The
Discount allowed 1,000 marginal cost of hiring an additional worker is N502.
Stock 01/01/2000 2,400 if the employees current wage bill is N20,000 by how
Salaries and wages 2,000 much will the wage paid per worker need to be
Purchases 30,000 increased in order to attract one more worker? (a)
Sales 35,000.00 N1.50 (b) N2.00 (c) N2.52 (d) N3.00
Return outwards 600 53 Given the cost function C =160 + 36q, what is the
Return inwards 900 average cost of 20 units of output? (a) N44 (b) N216
Debtors 6,800 (c) N880 (d) N720
Furniture 5,000 54 When the important of a commodity is limited to a
Discount received 1,300 definite quantity, the control means used is known as
Drawings 2,500 (a) Quotas (b) tax relief (c) devaluation (d) exchange
Capital 39,900 rate
55 How do commercial banks create money?? (a) by
41 The cost of goods available for sale is cashing cheques for their customers (b) when its
(a) N2,500 (b) 42,400 (c) N31,800 (d) N34,100 output is at its lowest (c) when the firm is maximizing
42 The net profit is (a) 2,500 (b) 1,300 (c) N2,800 (d) total revenue (d) when there is no tendency to
N1,700 change output
43 The process whereby a business organization 56 When is a profit maximizing firm in equilibrium? (a)
regularly compares physical events with the standard when average profit is maximized (b) when its output
set and takes corrective action is (a) organizing (b) is at its lowest (c) when the firm is maximizing total
coordinating (c) controlling (d) planning revenue (d) when there is no tendency to change
44 A speculators who applies for new stocks and shares output
with the intention of selling them at a higher price is a 57 Which of the following will be likely to result from an
(a) bull (b) stag (c) bear (d) jobber open market purchase of securities by the central
45 The cost of transporting goods to customers is called bank?
(a) carriage forward (b) carriage inward (c) carriage (a) a full in short-term capital flows (b) a full in the
outward (d) carriage paid quantity of money (c) a rise in the level of
unemployment ((d) arise in the rate of interest
58 Which of the following explains how changes in
national income may be brought about by changes in
SECTION D – ECONOMICS investment (a) the acceleration principle () the
46 The consumption function of an economy with no investment function (c) the marginal efficiency of
government sector and no foreign trade is C=W120 + capital (d) the theory of multiplier
0.6Y 59 Average earning in manufacturing increase by 8%
(C=aggregate consumption and Y national income) while hourly wage rates increase by only 6%. What
If investment expenditure is autonomous and equal to could account for this (a) an increase in overtime (b)
W280, what is the equilibrium level of income? (a) an increase in the level of employment (c) an
N400 (b) N600 (c) N800 (d) N1,000 increase in labour productivity (d) an increase in
47 The winner of a.T.V quiz show is offered a choice of unemployment
their N1000 or the contents of one or other of two 60 The diagram “DD is a consumer’s demand curve for
sealed boxes. She’s told that one box contains ac certain commodity
N2,500 and that the other box is empty. What is the
opportunity cost to the individual of opting for the
N1000 price D
(a) a 50% chance of winning W2500 (b) 50% chance
of winning W1,500 (c) a 50% chance of winning
W1500 (d) a 50% chance of losing W1,000 QUANTITY
48 Which schedule shows the maximum price at which a
firm would be able to sell any given level of output.
(a) demand schedule (b) marginal revenue schedule Q
(c) production schedule (d) supply schedule O
49 In which circumstance will an indirect tax fall entirely
upon -----------(a) when the demand curve is perfectly At price OP the consumer spends the amount OPRO,
(b) when the elasticity of demand is unit at all points there is possibility that this commodity will be withdrawn
on demand curve (c) when the elasticity of supply is from the market what is the most she would be willing to
unit at all points on the supply curve (d) when the pay for the right to continue top buy quantity OQ at price
supply curve is perfect inelastic OP?
50 In a closed economy with no government, the value (a) OPHD (b) DPR (c) OPHQ (d) QRD
of the investment multiplier is 3 by how much will

© 2014 www.examsguru.net | All Right Reserved Enquiries: call +234 (0) 802 652 9647 Page 48
Examsguru.Net – LASU POST UTME PAST QUESTIONS WITH SOLUTIONS 2005 - 2014
SECTION E – GEOGRAPHY 78 The key word for explaining the term authority is (a)
right (b) ability (c) control (d) power
61 The map of a farm land on a scale of 1:50,000 79 Gabriel Almond and Sydney verbal are scholars
measures 10.2cm by 12.6cm. the actual area of the known with the concept of (a) democracy (b) Zionism
farm to the nearest Km2 is? (a) 33 (b) 31 (c) 31.5 (d) (c) political culture (d) political propaganda
32. 80 The most appropriate form of government for a
62 The points on a river are 10.4km apart and one homogenous society is a (a) unitary system (b) one
stands 400 meters above the other what is the party system (c) totalitarian system (d) socialist
gradient along the river between the points (a) I in 24 system
(b) 1 in 20 (c) 1 in 26 (d) 1 in 36 81 A scholar that is known with the principle of right to
63 The wearing away of the sides and bottom of a river’s life, right to personal liberty/property is (a) Thomas
channel by the load carried by a river is called? (a) Hobbes (b) Jean Bodin (c) Harold Laxwell (d) John
corrosion (b) corrosion (c) attrition (d) hydraulic action Locke
64 In which of the following vegetation types would you 82 A deliberate tampering with electoral boundaries for
expect to find the baobab, locusts bean and acacia the purpose of winning future election is called (a)
as important trees? (a) tropical rainforest (b) politicking (b) Gerrymandering (c) lobbying (d)
savannah woodland (c) coniferous (d) temperature electoral trafficking
deciduous 83 1976 local government elections in Nigeria was
65 Which of the following group’s of features is normally based on (a) multi-party system (b) two party system
associated with the old age of a river? (a) sandbar, (c) zero-party system (d) one-party system
kick points and natural levees (b) meander terraces, 84 The National Youth Service Corps (NYSC) was
point bars and natural levees (c) point-bars, U- established in (a) 1960 (b) 1963 (c) 1964 (d) 1975
shaped valleys and ox-bow lakes (d) meander 85 Removal by impeachment applies to the position of a
terraces point bars and knick point (a) prime minister (b) cabinet minister (c) president
66 remnants of folded structures are found in the (a) (d) chief judge
Atwapin hills and Enugu scarp (a) Atwapin hills and 86 The membership of pressure groups is limited
Enugu scarp (b) Adamawa Highlands and Enugu because they (a) pursue specific and narrow
scarp (c) Adamawa Highland Atwapim Hills (d) Fouta objectives (b) are mainly concerned with public
Djallon Highlands and Adamawa Highland interest (c) promote other people’s interest (d) are in
67 Which of the following river basins has the highest patriotic
population concentration? The (a) Indus (b) Niger 87 The Emir’s treasurer in the pre-colonial Hausa/Fulani
(c) Nile (d) mississippi system was (a) Waziri (b) Dagari (c) Maaji (d)
68 The rain forest of South America is often referred to Madawakin
as (a) pampas (b) illanos (c) selvas (d) campos 88 Cattle Tax in the Fulani Emirate is called (a) Jigali
69 The major export products of Namibia are (a) coffee (b) Isakole (c) Alkali (d) Hakimi
and rubber (b) ranium ad diamond (c) copper and iron 89 The first political party in Nigeria was established in
ore (d) petroleum and coal (a) 1923 (b) 1922 (c) 1960 (c) 1944
70 A mature soil with distinct horizons, occurring in a 90 The majority of OPEC members are located in (a) the
particular climatic zone but influenced strongly by middle belt (b) Asia (c) Africa (d) Latin America
topography is best described as (a) intrazonal (b)
zonal (c) alluvial (d) azonal SECTION G – BIOLOGY

71 In karst region, when several dolines are joined 91 The following show three processes that contribute to
together to form a large depression, it is known as? transport across cell surface membranes which
(a) cave (b) uvala (c) stalaclite (d) calcite pallar processes
72 Most of the word’s principal fishing regions are are active and which are passive?
located on continental shelves where (a) cold and Diffusion Endocytosis Osmosis
warm current meet (b) there are no noticeable ocean A a A a
(c) there are cold ocean currents (d) there are warm B b B b
ocean currents C p p p
73 Rural settlements can best be distinguished from A p p p
urban settlements by their? (a) function and
population (b) site and function (c) demography and Key: a = active process
morphology (d) morphology and location P = passive keys
74 The scientist who propounded the theory of
continental drift was? (a) Francis Bacon (b) Alfred 92 Which statement is the best explanation of why
Wegener (c) Authur Holmes (d) William Davis sucrose rather than glucose is transported by
75 Blamtyre, Rio de Janeiro, New York and Lagos are phloem? (a) sucrose can pass through plant cell
similar in that they all serve as their countries? (a) surface membranes more easily (b) sucrose is a
major seaport (b) political headquarter (c) commercial disaccharide and is more easily converted to starch
centers (d) cultural centre (c) Sucrose is a non-reducing sugar, so is less
reactive (d) sucrose synthesis requires less energy
SECTION F – GOVERNMENT
76 Classical democracy is associated with one of the
following countries (a) France (b) Britain (c) USA (d)
Greece
77 Which of the following is not necessarily the feature of
a Nation? (a) government (b) population (c)
sovereignty (d) territory

© 2014 www.examsguru.net | All Right Reserved Enquiries: call +234 (0) 802 652 9647 Page 49
Examsguru.Net – LASU POST UTME PAST QUESTIONS WITH SOLUTIONS 2005 - 2014
93 The diagram represents part of the nitrogen cycle Below are names of groups used when classifying
which process is carried out by nitrifying bacteria? organisms
i. phylum ii. Kingdom vii order
Nitrogen in atmosphere iii class iv. Family
A v. species vi Genus

B Use them to answer question 101


ammonia
C 101The correct order when grouping organism is
D (a) i, ii, iii, iv, v, vi and vii (b) i, ii, iv, vi, v and vii
(c) ii, I, iii, vii, iv, vi and v (d) ii, iv, I, vi, vii, iii and v
nitrogen compounds nitrogen compounds nitrogen 102 During sexual reproduction the mega nucleus in
compounds paramecium (a) divide into 2 parts (b) divide into 4
in soil in plants in animals parts (c) breaks into 4 parts (d) fuses with the
micronucleus
103 Which of the following tissues represents supporting
94 From which cell organelle are nucleic acids absent? tissue? (a) columnar epithelium (b) cartilage (c)
(a) Chloroplast (b) Golgi body (c) Mitochondrio striated muscle (d) nervous tissue
(d) Ribosome 104 The creamy semi-fluid of food in the stomach after
95 What will break an ionic bond between amino acids? some hours is known as (a) Chile (b) chyme (c) chyle
(a) condensation (b) high temperature (c) hydrolysis (d) chiline
(d) pH change 105 Which of the following structures is not in the male
96 The drawing shown stages of the mitotic cell cycle reproductive parts? (a) uniferous tubule (b) prostate
gland (c) epidymis (d) seminal vesicles

ANSWERS
SECTION A SECTION D SECTION F
ENGLISH LANGUAGE ECONOMICS GOVERNMENT
1 D 46 D 76 C
2 B 47 A 77 C
3 D 48 D 78 A
4 C 49 B 79 C
5 C 50 - 80 A
In which order do the stages occur?
6 B 51 D 81 C
First 7 B 52 - 82 C
Last
8 D 53 - 83 C
A 2 1 3 5 4 9 A 54 A 84 C
B 2 4 1 5 3 10 B 55 A 85 C
C 4 2 1 3 3 11 B 56 D 86 A
D 4 2 3 1 5 12 D 57 B 87 C
13 - 58 D 88 A
14 - 59 C 89 A
97 The first breath milk produced by the mother for a 15 A 60 B 90 B
new born baby contains antibodies? (a) what do
these antibodies provide? (a) artificial active immunity SECTION C SECTION E SECTION G
(b) artificial passive immunity (c) natural active COMMERCE GEOGRAPHY BIOLOGY
immunity (d) natural passive immunity 31 A 61 91 -
98 Which protein has a fibrous structure? (a) Amylase 32 C 62 92 C
(b) collagen (c) Haemoglobin (d) Insulin 33 C 63 93 C
99 A girl has blood group A and her brother has blood 34 B 64 94 B
group B which combination of genotype cannot 35 A 65 95 B
belong to their parents? 36 C 66 96 D
37 D 67 97 B
Mother Father 38 C 68 98 B
A A B O
A 1 1 1 1 39 A 69 99 A
B
A
1 1
B A B
1 40 - 70 100 B
O O A A
41 C 71 101 C
C 1 1 1 1 42 B 72 102 -
D 1
O
1
O A
1 1
O 43 D 73 103 B
44 B 74 104 B
45 C 75 105 A
100 In which form is carbondioxide mainly transported in
blood? (a) as carbammo-haemoglobin (b) as carbonic 31 One of the following is not a type of drama
acid (c) as hydrogencarbonate (d) in solution (a) Burlesque (b) Resolution (c) Tragedy (d)
Pantomime
32 The analysis of the metrical pattern of a poem is (a)
prosody (b) appreciation (c) scansion (d) style

© 2014 www.examsguru.net | All Right Reserved Enquiries: call +234 (0) 802 652 9647 Page 50
Examsguru.Net – LASU POST UTME PAST QUESTIONS WITH SOLUTIONS 2005 - 2014
33 One of the following is correct (a) dramatization 52 The very first cargoes of slave from Nigeria shores in
occurs only in plays (b) only prose and drama use European ships were destined for: (a) England (b)
dramatization (c) dramatization is used in poetry (d) The Americans (c) the Gold Coast (d) Egypt
dramatization is found only in prose 53 The great Benin Monarch reputed to have
34 Both tragic and comic play have (a) happy ending (b) transformed the state from a small kingdom into an
sad ending (c) temper (d) climax aggressively expansive empire was (a) Ewuare (b)
35 The dramatic interaction between the protagonist and Eweka 1 (c) Esigie (d) Ozolua
the antagonist in a play is known as (a) dialogue (b) 54 Which of the following is NOT associated with the
climax (c) understanding (d) conflict origin of the Hausa? (a) Queen Daura (b) Agadu (c)
36 The English sonnel has ………….syllables (a) 110 (b) Bawo (d) Bayajidda
96 (c) 140 (d) 120 55 The basis for Nigeria’s membership of the
37 A symbol could be: (a) universal or private (b) general commonwealth is (a) she was a former colon of
of particular (c) literary or non-literary (d) dramatic or Britain (b) She was a leading opponent of a gathered
poetic in South Africa (c) She is the most populous black
38 Melodrama and tragic-comedy have this in common: nation (d) she provides athletes for the
(a) both end unhappily (b) both are comic plays (c) commonwealth games
tragedies occur in both (d) both are prose narrative 56 Berber nationalism and resistance against the French
39 A prose narrative having independent plots loosely invasion of Algeria revolved around a man called (a)
linked together is said to have plot (a) linear (b) Abdel Rahman (b) E! Hassan III (c) Col Urabi (d)
simple (c) episodic (d) difficult Abdel Kader
40 …………..is used when humour is achieved through 57 Khedive Ismail of Egypt established a parliamentary
unexpected and pointed manipulation words: (a) wit system of government when he created the assembly
(b) sarcasm (c) onnuendo (d) parody of delegated in (a) 1863 (b) 1879 (c) 1866 (d) 1873
41 “Peripetia” occurs in (a) prose (b) comedy (c) drama 58 The signing of the Treaty of Marza in 1883
(d) tragedy established full colonial control of the French over (a)
42 Which of the following is Chaucer noted for? Moroco (b) Libya (c) Tunisia (d) Algeria
(a) the Midsummer Night Dream (b) the Mayor of 59 Mohammed Ali Pasha who dominated the history of
th
Canterbury (c) the Canterbury tales (d) none of the Egypt for the first half of the 19 century was (a)
above Persian Prince (b) an Albanian by origin (c) a Turkish
43 The occurrence of similar themes, characters, events Sultan (d) a Mamluk by birth
and concepts in works of art is known as: (a) 60 Mohammed All’s main concern was to: (a) encourage
repetition (b) plagiarism (c) imitation (d) leit mouf Egyptian nationalism (b) establish Turkish control
44 When a character in a literary work psses personal over the whole of North Africa (a) encourage Egyptian
traits opposite to that of another in it same text, he is nationalism (b) establish Turkish control over the
said to be a(n) to Him/Her (a) antagonist (b) foil (c) whole of North African (c) discourage foreign trade (d)
protagonist (d) villain make Egypt a modern state on the European modei

SECTION H – C.R.S
45 Versification include the use of (a) rhythm, style and 106 The first stories of the patriarchs form the great
form (b) meter, rhyme and stanza form (c) length of theological history of Israel’s origin. In which of the
line, paragraph and chapter (d) characterization, following can this be found? (a) the first six books of
rhythm and verse the Bible (b) the first last books of the Bible (c)
Genesis and Exodus (d) the Poetic
SECTION D – HISTORY 107 The first six books of the Bible tells us that (a)
46 The disastrous drought which affected the saheliani centuries before Israel settled in canaan (b) the
zone of Africa south of the Sahara occurred in the patriarchs worshippied Canaanite gods (c) the
year ancestors of Israel came from Mesopotamia (d)
(a) 1953 (b) 1963 (c) 1973 (d) 1983 centuries before Israel took passion of Canaan her
47 The agents of external influence in the eastern Niger ancestors had come from Mesopotamia and as semi
th
Delta region of Nigeria in the 19 century included all nomads had roamed through the land supported by
but one of the following: (a) the merchants of various the promises of their God that I would one day belong
European nations (b) the heavy military presence of to their posterity.
the Americans (c) the missionaries beginning 108 It is not historically accurate to say that the god of the
operations in Calabar (d) the official British presence Patriarchs was Yahweh. Yahwism begain with (a)
represented by naval officers Joshua (b) Moses (c) Abraham (d) Isaac
48 The first British consul appointed over the bights of 109 The patriarchal migration was time conditioned none-
Benin and Biafra in 1849 was: (a) Richard Lander (b) the-less it was in real sense an act of (b) Grace (c)
Macgregor Laird (c) Hugh Clapperton (d) John Patience (d) Courage
Beecroft 110 According to the book of Numbers Israel upon leaving
49 The first of the rulers of the ancient Kanuri State to Sinai had it local point for sometimes at (a) Kadesh a
become a Muslim was: (Umme Jilmi (b) Dunama great oasis some fifty miles south of Beer-sheba (b)
Dlobatemi (c) Idris Alooma (d) Alh. Ghaji the Amorite Kingdom of Hesbon (c) the ancient city of
50 “Nsibidi”, the reduction into writing of the sign Shechem (d) Ancient shrine of Hebron
language was used to keep records of the past by 111 From the beginning, Israel’s faith forbade the worship
the: (a) Ijo (b) Kslabari (c) Gwari (d) Ibibio of any god but Yahweh. This prohibition is classically
51 The Alafin of Oyo credited with the creation of the expressed in the (a) first commandment (b) second
Are-Ona Kakanfo title was (a) Obalokun (b) Ajagbo commandment (c) third commandment (d) eight
(c) Sango (d) Awole commandment

© 2014 www.examsguru.net | All Right Reserved Enquiries: call +234 (0) 802 652 9647 Page 51
Examsguru.Net – LASU POST UTME PAST QUESTIONS WITH SOLUTIONS 2005 - 2014
112 Saul’s kinsman Abner, who had surved the slaughter 140 Which of these economic activities – LEAST
at Gilboa made the only suriving son of Saul King at characteristics of typical urban centre? (a) commerce
Mahanamin in Transjordan who was this soon? (a) (b) quarrying (c) transportation (d) manufacturing
Jonathan (b) Solomon (c) Eshbaal (d) Abijah 141 In Karst region, when several dolines are joined
113 David became King over Judah in Hebron, that did together to form a large depression, it is know as (a)
with the Philistine consent is certain. WHY? (a) David cave (b) uvala
was an army commander in Ziklag (b) David paid (c) stalacitite (d) calcite pallar
tributes to the Philistine 142 Rural settlements can best be distinguished from
(c) David killed Goliath (d) David was their vassal and urban settlements by their? (a) function and
could hardly have taken such a step without their population (b) site and function (c) demography and
approval morphology (d) morphology and location
114 The people of Judah welcomed David as king in 143 The scientist who propounded the theory of continent
Hehron. WHY? (a) He was one of them (b) He was a drift was? (a) Francis Bacon (b) Aifted Wegener (c)
strong leader who could see to their defense (c) He Author Holmes (d) William Davis
was acclaimed King by popular consent (d) he was 144 The major world exporters of wool are (a) Britain,
one of them, a strong leader who could see to their Australia, Mediterranean Europe and South Africa (b)
defense and was also in the position to mediate New Zealand, Uruguay Australia and Argentina (c)
between them and their philistine masters. Australia, India, Paraguay and Argentina (d) New
115 After the murder of Eshbaal by two of his officers, the Zealand, India, Uruguay and Peru
people flocked to David in Hebron and there in a 145 Bjantyre, Rio de Janeiro, New York and Lagos are
solemn covenant acclaimed King over all Israel. The similar in that they all serve as their countries? (a)
whole incident illustrates. (a) the tribal confederacy major seaport (b) political headquartr (c) commercial
(b) the tenacity of the charismatic tradition (c) the centers (d) cultural centres
theocracy of Israel (d) the monarchy of Israel 146 The dawn is a temperate grassland found in? (a)
116 Which of the synoptic Gospel writers recorded thin Australia (b) South America (c) Eurasia (d) South
would have prevented Jesus from being baptized by Africa
himself (a) Matthew (b) Matthew and Luke (c) Luke 147 Commercial grape cultivation is associated with?
(d) John (a) cool temperature climate (b) morsnon region
117 According to the Gospel of Matthew who saw the (c) Mediterranean regions (d) Montane climate
Holy Spirit descending on Jesus when Jesus was 148 The highest volume of shipping across the Atlantic
being baptized? (a) John Himself (b) Jesus Himself ocean is (a) between Europe and Africa (b) between
(c) Jesus and John (d) Jesus, John and the other North America and Europe (c) between Africa and
people South America (d) between North and America and
118 Which of the following does not record the temptation South America
of Jesus? (a) Luke (b) John (c) Matthew (d) Mark 149 The Ukraine (in the USSR) is famous with the
119 The synoptic writers record that ether the baptism following pairs of economic activities (a) commercial
Jesus was tempted by the devil. The third temptation grain agriculture and livestock ranching (b) plantation
in Matthew was aimed at minding out (a) whether agriculture and nomadic herding (c) commercial grain
Jesus would succumb to the Jewish expectation of a farming and industry (d) fishing and lumbering
political and conquering messiah. (b) testing his 150 The following countries in West Africa have economic
father and see whether he will measure up to the deposits of iron ore (b) Liberia, Sierra Leone and
needs of one who has wholly dedicated himself to Togo (c) Nigeria, Guinea and Sierra Leone (d) Niger,
doing God’s will (c) testing him to employ his power Mali and Mauritania
as the son of God in satisfying his own needs and 151 Adelcredere agent (a) has unrestricted powers (b)
other human physical needs. arranges credit terms (c) guarantees payment by the
120 “Those who are well have no need of a physician but buyer (d) arranges insurance
those who are sick”. The statement was made in the 152 The ratio of capital owned to other liabilities in a
house of (a) Levi (b) Peter’s mother in-law (c) business is known as: (a) efficiency ratio (b) liquidity
Zacchaeus (d) Zebedee (e) the latter is appropriate in ratio (c) gearing ratio (d) gearing ratio (c) gearing ratio
an enlighten environment (d) ratio of return
153 The computer using linear integrated circuit
technology coupled with qualification of data in term
SECTION J - GEOGRAPHY of length and distance is known as (a) analogue
136 Which of the following features produced by wave computer (b) hybrid computer (c) digital computer (d)
deposition (a) caves (b) stack (c) tombolo (d) blow mainframe computer
holes 154 A package contained in a wrapper bound with rope,
137 Which of the following is the major factor responsible wire or iron strips is known as alan (a) Bail (b) berth
for Japan’s unparalleled industrial growth in the last (c) bale (d) bait
few decades (a) Japan’s proximity to mainland (b) 155 An agreement to hire a ship which gives the chatterer
Japan’s cargo population (c) availability of abundant compete control of the skip is known as (a)
coal and petroleum (d) technological proficiency demurrage (b) voyage charted (c) charter by demise
138 Which of the following limber frees are found in the (d) shipping conference
tropical rain forest of Africa? (a) obeche and teak (b) 156 The term subrogation relates to (a) insurance (b)
mahogany and teak (c) obeche and mahogany (d) finance
Iroko and Eucalyptus (c) advertising (d) premium
139 Which of the following river basins has the highest 157 Organizing (b) coordinating (c) controlling (d)
population concentration? The (a) Indus (b) Niger (c) planning
Nile (d) Mississippi

© 2014 www.examsguru.net | All Right Reserved Enquiries: call +234 (0) 802 652 9647 Page 52
Examsguru.Net – LASU POST UTME PAST QUESTIONS WITH SOLUTIONS 2005 - 2014
158 A speculator who applies for new stocks and shares could account for this?(a) an increase in over time (b)
with the intention of selling them at high price is a (a) an increase in the level of employment (c) an
bull (b) stag (c) bear (d) jobber increase in labour productivity (d) an increase in
159 At Lioyds, groups of insurers form themselves into unemployment
a) chambers (b) syndicate (c) associations (d) cartels 174 In which circumstances would a tax on alcohol be
160 The cost of transporting goods to customers is called most effective in reducing drinking? (a) high income
(a) carriage forward (b) carriage inward (c) carriage elasticity of demand for alcohol drinks (b) High price
outward (d) carriage paid elasticity of demand for alcoholic drinks (c) low
161 Drawing two parallel lines across a cheque means income elasticity of demand for alcoholic drinks (d)
that it has to be paid into the account of the (b) low price elasticity of demand for alcoholic drinks
drawer (b) payee (c) drawee (d) payer 175 The basic economic problems of then society are (a)
162 The parties to a cheque are the (a) drawer, payee what to produce, how and for whom (b) how to
and endorse (b) payer, drawer and payee (c) bank produce and sell (c) scarcity, when to produce and
drawer and drawee (d) drawer, payee and drawee how (d) means used in different ways
163 The system of parting with authority but not 176 If income rises from 200 to 250 and the amount spent
responsibility is (a) controlling (b) planning (c) on good X falls from 30 too 28 then good X is (a) an
delegating (d) coordinating income elastic good (b) a normal good (c) an inferior
164 Which of the following documents contains the good (d) demand elastic good
complete information on goods available in the 177 Both the short run and in the long run, a firm
wholesaler’s warehouse? (a) catalogue (b) order (c) maximizes its profit when (a) MC=MR (b) AC=MC (c)
consignment AVC=AC
(d) invoice (d) MC=AV
165 The rebate given to retailer by the wholesaler to 178 Which of the following is not a form of business
enable him make profit is known as (a) debit note (b) organization? (a) sole proprietorship (b) cartel (c)
cash discount (c) trade discount (d) credit guarantee partnership (d) Joint Stock
179 Which of the following measures is; directly aimed at
SECTION L – ECONOMICS redistributing wealth? (a) inheritance taxes (b) profits
166 The price elasticity of demand for a good is unity. taxes (c) progressive income taxes (d) unemployment
What will increase as a resale of a reduction in the benefits
price of the good? 180 A rational consumer allocates his expenditure so as
(a) expenditure on the good (b) expenditure on to (a) equate the marginal utilities obtained from each
substitutes (c) marginal revenue (d) quantity good (b) equate the total utilities obtained from each
demanded good (c) maximise the marginal utility from each good
167 Under which conditions will the economic rent (d) maximize total utility from his expenditure as a
received for a given type of labour decrease? (a) the whole.
demand curve for the labour shift to the right (b) the
opportunities for training and mobility increase (c) the SECTION M – GOVERNMENT
skills possessed by such labourers become more 181 A systematic effort to manipulate the beliefs, altitudes
specialised (d) the supply curve for that labour and actions of the public through the mass media and
becomes more inelastic other means is called (a) political enlightenment (b)
168 Which of the following defines the minimum efficient opinion poll
scale of output? (a) where a plant of a given size is (c) propaganda (d) mass mobilization
used to full capacity (b) where the firm can avoid
making a loss (c) where the long run average cross a
firm stop falling 182 Those who hold public offices and am actively
169 An employer currently employs 200 workers. The involved in politics are referred as (a) politicians (b)
marginal cost of hiring an additional worker of N502. political gladiators (c) political spectators (d) political
if the employees current wage bill is N20,000 by how activists
much will the wage paid per worked need to be 183 What differentiates referendum from plebiscite is that
increased in order to attract one more worker? (a) (a) the former emphasizes yes or no voting
1.50 (b) 2.00 (c) 2.52 (d) 3.00 (b) the letter L commonly used in Europe America
170 A single country within a monetary union experiences (c) the farmer is more popular than the latter
a current account balance of payment deficit. Which 184 A deliberate lampering with electoral boundaries for
policy could the country use reduce the deficit? (a) the purpose of winning future elections is called (a)
exchange rate policy (b) monetary policy (c) fiscal politicking (b) Gerrymandering (c) lobbying (d)
policy (d) britt policy electoral trafficking
171 When is a profit maximizing firm in equilibrium? (a) 185 Which of the following is not an interest group?
when average profit is maximized (b) when its output (a) Christian Association of Nigeria (b) Nigerian
is at its lowest (c) when the firm is maximising total Medical Association (c) Catholic Church (d)
revenue (d) when thee is no tendency to change Pentecostal Fellowship of Nigeria
output 186 The membership of pressure groups is limited
172 Which condition makes price discrimination possible? because they (a) pursue specific and narrow
(a) a large number of firms in the market (b) identical objectives (b) are mainly concerned with public
elasticity of demand in the market in which the interest (c) promote other people’s interest (d) are not
product is solid (c) the existence of product patriotic
differentiation (d) the separability of markets in which 187 Cattle tax in the Fulani Emirate is called
the product is sold (a) Jigali (b) Isakole (c) Alkali (d) Hakimi
173 Average earnings in manufacturing increase by 8% 188 The first political party in Nigeria was established in
while hourly wage rates increase by only 6% what (a) 1923 (b) 1922 (c) 1960 (d) 1944

© 2014 www.examsguru.net | All Right Reserved Enquiries: call +234 (0) 802 652 9647 Page 53
Examsguru.Net – LASU POST UTME PAST QUESTIONS WITH SOLUTIONS 2005 - 2014
189 A legislative debate or proceeding which is attended 111 B
by all members of the house is called (a) plenary 112 -
session (b) recess session (c) committee 113 D
stage/session (d) third reading session 114 D
190 The majority of OPEC members are located ii (a) the 115 A
middle belt (b) Asia (c) Africa (d) Latin America 116 A
191 The Vice Presisdent in Nigeria between 19789-1983 117 D
is 118 -
(a) Dr. Joseph Wayas (b) Dr. Alex Ekwueme (c) Alhaji 119 A
Bashiru Tofa (c) Chief A. M. Akinloye 120 A
192 A principle that advocates total equality of members
of a society (a) communalism (b) egalitarianism (c) SECTION K
cotalitarianism (d) oligarchy COMMERCE
193 The Highest Court of Appeal in Nigeria in 1962 was in 151 C
the 152 C
(a) Privy Council, Judicial Committee, House of 153 C
Lords, Britain (b) Supreme Court (c) Court of Appeal 154 B
(d) High Court 155 C
194 The ending of a session of the legislature that has not 156 A
exhausted its tenure is called (a) dissolution (b) recall 157 D
(c) prorogation (d) quorum 158 B
195 A group of creditor nations to which most third world 159 B
countries are indebted is called (a) Pairs Club (b) 160 C
Accra Club (c) World Bank (d) Economic Commission 161 C
for Africa 162 D
163 C
ANSWERS 164 A
GEOGRAPHY 165 C
136
137
138
139
Notes
140 -----------------------------------------------------------------------------
141
142 -----------------------------------------------------------------------------
143
144 -----------------------------------------------------------------------------
145
145 -----------------------------------------------------------------------------
147
148 -----------------------------------------------------------------------------
149
150 -----------------------------------------------------------------------------

SECTION C -----------------------------------------------------------------------------
LIT. IN-ENGLISH
31 B -----------------------------------------------------------------------------
32 C
33 B -----------------------------------------------------------------------------
34 D
35 A -----------------------------------------------------------------------------
36 C
37 B -----------------------------------------------------------------------------
38 B
39 A -----------------------------------------------------------------------------
40 B
41 C -----------------------------------------------------------------------------
42 A
43 D -----------------------------------------------------------------------------
44 A
45 B -----------------------------------------------------------------------------

SECTION H -----------------------------------------------------------------------------
C. R. S.
106 C -----------------------------------------------------------------------------
107 D
108 B -----------------------------------------------------------------------------
109 A
110 - -----------------------------------------------------------------------------

© 2014 www.examsguru.net | All Right Reserved Enquiries: call +234 (0) 802 652 9647 Page 54

You might also like